старшая группа, высшая лига

advertisement
XXXVII УРАЛЬСКИЙ
(XIX КИРОВСКИЙ) ТУРНИР
ЮНЫХ МАТЕМАТИКОВ
14–20 ФЕВРАЛЯ 2011 ГОДА
КИРОВ 2011
АБАКА-СЕНЬОРЫ
Комбинаторика
0
0
0
0
0
Целые числа
Логика
Сегодня 14.02.2011.
Рассмотрим эту дату как 4
двузначных или однозначных числа 14, 02, 20, 11.
Например, 14 делится на 2,
но 14 и 11 не делятся одно на
другое. Найдите две ближайшие даты (одну в прошлом, другую в будущем),
чтобы НОД любых двух из
получившихся четырёх чисел был больше 3.
В комнате было несколько
человек. Один сказал: «Нас тут пятеро» – и ушёл. После этого каждую минуту кто-то уходил, сказав
на прощание: «Все, кто ушел до меня, перед уходом солгали», пока
комната не опустела. Сколько человек, уходя, могли сказать правду?
Учитель хочет расставить по кругу 10 мальчиков и
несколько девочек так, чтобы
рядом с каждым мальчиком
2
стояли мальчик и девочка, а
через одного от каждой девочки стояли тоже мальчик и девочка. Сколько девочек можно
поставить в этот круг?
На плоскости нарисован правильный пятиугольник
ABCDE. Лёня хочет раскрасить
его стороны фломастерами
трех3 цветов так, чтобы каждая
сторона была бы одного цвета,
а любые две стороны одного
цвета не имели бы общих вершин. Сколькими способами
Лёня сможет это сделать?
Найдите целые
положительные числа
a, b, c, для которых
НОК(a,b)
=
210,
НОД(a,b)
=
10,
НОК(a,c)
=
110,
НОД(a,c) = 2.
В школе прошел забег с
участием 10 спортсменов, и все заняли разные места. На следующий
день каждого из них спросили, какое место он занял, и каждый, естественно, назвал одно из чисел от 1
до 10. Сумма их ответов оказалась
равна 37. Какое наименьшее число
врунишек могло быть?
Каждую
минуту
написанное на доске натуральное число x заменяют
на число x+k, где k –
наибольший делитель x, не
равный ему. Вначале на
доске написано чётное число N, не кратное 4. Какое
число будет написано после
10 замен?
Отметили центр
каждого из квадратиков
прямоугольника 3×2011.
4
Сколько
существует
прямых, которые проходят ровно через три отмеченные точки?
Найдите
все
простые числа p, для
которых 6p+1  точная пятая степень, если известно, что такие числа существуют.
Сколькими способами можно расставить
в клетках таблицы 510
5
числа 1 и 1 так, чтобы
произведение чисел в
любой строке и любом
столбце равнялось бы 1?
Рассматриваются все девятизначные числа, у которых
цифры идут в порядке
невозрастания.
Найдите остаток от
деления суммы всех
этих чисел на 9.
Саня дружит с Алексеем, Верой,
Глебом, Дашей и Евгением. У него День
Рождения. Если пригласить Алексея, то
тогда надо пригласить и Веру. Обязательно
нужно пригласить либо Глеба, либо Веру,
но приглашать их вместе не стоит. Также
нужно пригласить либо Дашу, либо Евгения, либо обоих вместе. Дашу можно приглашать либо вместе с Глебом, либо никого
из них. А если пригласить Евгения, то тогда
необходимо пригласить Алексея и Дашу.
Кого же всё-таки надо пригласить?
Петя, Дима, Миша, Саша и Илья
играют в мафию. Среди них два мафиози,
два мирных жителя и комиссар. Мафиози
знают только друг друга, комиссар знает
роль каждого, мирные жители не знают
роли других игроков. Мафиози всегда лгут.
Комиссар и мирные жители говорят правду.
Мальчики сделали следующие заявления:
Петя: «Я не знаю, кто Дима»;
Дима: «Я знаю, кто комиссар»;
Миша: «Я знаю, кто Петя»;
Саша: «Я знаю, что Миша – комиссар».
Кто Илья?
В компании из 100 друзей, среди которых
есть рыцари, лжецы и нормальные люди
были произнесены следующие высказывания: «Среди моих друзей 0 лжецов»; «Среди
моих друзей 1 лжец»; и т.д. до фразы «Среди моих друзей 99 лжецов». Через минуту в
той же компании прозвучали те же фразы,
но каждую фразу произнёс не тот человек,
что в первый раз. Еще через минуту в той
же компании прозвучали те же фразы, но ни
один человек не повторил уже произнесенных им фраз, и так далее. Каждый произнёс
99 различных фраз. Сколько лжецов могло
находиться в этой компании?
Солдаты выстроились в
колонну по одному, причём
среди солдат есть только брюнеты, блондины и рыжие. Есть
ровно
1 17 брюнетов, которым в
затылок дышат блондины, а
еще есть ровно 23 блондина, за
которыми сразу стоят брюнеты. Какое наименьшее число
рыжих могло быть в строю?
1
0
На дорогу вышла толпа
из 4096 человек, и сразу ровно
половина из них пошла направо,
а половина – налево. Через полчаса каждая группа разделилась
пополам, половина пошла дальше, а половина повернула об6
ратно. Еще через полчаса каждая
из групп снова разделилась ровно пополам и так далее. Найдите, сколько человек окажется в
исходной точке после десяти
таких «делений». Все люди идут
с одной и той же постоянной
скоростью.
АБАКА-СЕНЬОРЫ
Деятельная
0
0
0
0
Найдите количество таких натуральных чисел n, что
n = [n/2]+[n/3]+[n/5].
[k] – целая
часть числа k, то есть
наибольшее
целое
число, не превосходящее k.
В одной компании
было x человек (каждый –
рыцарь или лжец). Каждый
произнес фразу: «Среди
всех остальных (не считая
меня) не больше n рыцарей
и не больше k лжецов». Чему мог быть равен x?
Геометрическая
Алгебра
Методкомиссия в неполном
составе собралась на турнир. Через
день приехал Дмитрий Валерьевич, и
Игорь Соломонович записал, во
сколько раз увеличилась методкомиссия. Потом приехал еще Александр Сергеевич, и Игорь Соломонович1 опять записал, во сколько раз
увеличилась методкомиссия. То же
самое он сделал после приезда Алексея Владимировича, и, наконец, после приезда Леонида Михайловича,
который приехал последним. Произведение четырёх записанных чисел
оказалось равным 4/3. Сколько человек входит в методкомиссию?
Биссектрисы треугольника ABC пересекаются в точке
I. Через точку I проходят две
прямые, которые параллельны
прямым AB и AC и пересекаются с BC в точках D и E. Найдите
периметр треугольника IED,
если AB=10, BC=11, CA=12.
Известно,
что
–
3  x  7,
–2  y  5, –5  z  3.
Найдите наименьшее
возможное значение выражения
xyz–z2.
За столом сидят несколько
мальчиков и 5 девочек, а на тарелке
лежат 60 булочек. Каждая девочка
дала по булочке (с тарелки) каждому
2
знакомому ей мальчику, а каждый
мальчик дал по булочке (с тарелки)
каждой незнакомой ему девочке. После этого оказалось, что все булочки
розданы. Сколько было мальчиков?
Дан
равнобедренный
треугольник ABC с основанием
AC и углом B=700. На отрезках
AB и AC взяты точки D и E соответственно
такие,
что
DA+AE=AC. На отрезках AC и BC
взяты точки F и G соответственно такие, что FC+CG=AC. Точка
E лежит между A и F. Найдите
угол между прямыми DF и EG.
Известно, что для
попарно различных чисел
x, y, z, t верно, что
x  z y  z  2011

. Укажите
Четверо подружек поделили между собой 1001 конфету, при
этом каждой девочке досталось
конфет
или столько же, сколько у
3
какой-то из ее подружек, или ровно
в два раза меньше, чем у одной из
них. Как могли распределиться
конфеты?
В полдень черепаха начала
ползти домой, каждый час поворачивая или налево, или направо (между
поворотами
она ползла прямолиней4
но). За первый час она проползла 50
метров, а за каждый последующий
час  на 50 метров больше, чем в
предыдущий. На каком наименьшем
В треугольнике ABC на
стороне BC отмечена точка D
так, что CD/BD =
. Известно, что ABC = 45, DAB =15.
Найдите угол ACB.
Разрежьте фигуру на две
части, из которых можно составить квадрат (черные клетки –
это дырки). Части можно поворачивать, но не переворачивать.
2
x t
y  t  2011
все возможные значения
разности x–y.
Введём новую операцию:
ab  наибольшее из чисел
2a и a+b.
Решите уравнение:
2 (x (–2))=2011.
Найдите
сумму:
расстоянии от места старта она могла
оказаться в 7 часов вечера?
1
1
5
Имеется 8 внешне неразличимых монет, веса которых — различные натуральные числа. Известно, что если на левую и правую чаши
исправных
двухчашечных весов про5
извольным образом положить по две
монеты, то перевесит та чаша, на которой лежит самая тяжелая монета из
этих четырёх. Какой наименьший вес
может иметь самая тяжелая монета?
Вдоль дороги расставлены
светофоры на расстоянии 10 км. Они
работают так: последние 5 минут
каждого часа там горит красный
свет, остальное время – зеленый.
6
Машина ехала по этой дороге 10 часов с постоянной скоростью, при
этом ни разу не остановившись на
светофорах на красный свет. Какое
наибольшее расстояние она могла
проехать?
0
0
АБАКА-ЮНИОРЫ
Комбинаторика
0
0
В посёлке 2011 жителей.
Однажды каждый житель переехал в другой домик, причём в
каждый домик переехал ровно
один житель. Какого минималь1
ного количества цветов всегда
будет достаточно, чтобы покрасить домики (каждый домик в
один цвет) так, что каждый житель переехал из домика одного
цвета в домик другого цвета?
Площадь прямоугольника ABCD равна 1. На стороне
AB отмечены точки G и F так,
что BG = GF= AF. Точка E –
середина CD. Диагональ AC
пересекает отрезок GE в точке
K, а отрезок FE в точке H.
Найдите площадь треугольника
KHE.
В узлах клетчатой бумаги
живут садовники, а вокруг них
повсюду растут цветы. За любым
цветком должны ухаживать три
ближайших к нему садовника.
(Иногда несколько садовников
находятся на одинаковом расстоянии - в таком случае за цветком
ухаживают все они.) Нарисуйте
участок, за которым должен ухаживать каждый садовник.
Алгебра
Известно, что
–3  x  7,
–
2 y  5,
–5  z  3.
Найдите наименьшее
возможное значение
выражения xy–z2.
Солдаты выстроились в
колонну по одному, причем среИзвестно, что для чеди солдат есть только брюнеты, тырёх чисел x, y, z, t выполблондины и рыжие. Есть ровно нено равенство
217 брюнетов, которым в затылок
x z x  z  10
 
.
дышат блондины, а еще есть
y t y  t  10
ровно 23 блондина, за которыми
Укажите все возсразу стоят брюнеты. Какое
можные
значения дроби x/y.
наименьшее число рыжих могло
быть в строю?
3
1
4
1
4
1
3
1
3
1
2
1
1
1

1
1
5
1
1
6
100
6
1
100
Долго-долго
Марьванна
выписывала все произведения
восьми попарно различных натуральных сомножителей, каждый из
которых меньше 17, и никакие два
из которых не дают в сумме 17.
Затем она сложила все полученные
произведения. Какое число у нее
получилось?
Известно, что
(2x2–x+1)(y2+4y+8)(z2–
2z+3)= 7 .
Найдите x+y+z.
Логика
В семье есть Иван Сидорович, Сидор Иванович,
Сидор Петрович, Петр Сидорович, Петр Петрович. Один
из них сейчас смотрит телевизор, его отец дремлет, брат
читает газету, а дети ушли
гулять. Как зовут того, кто
смотрит телевизор?
В комнате было несколько человек. Один сказал: «Нас тут пятеро» – и
ушел. После этого каждую
минуту кто-то уходил, сказав
на прощание: «Все, кто ушел
до меня, перед уходом солгали», пока комната не опустела. Сколько человек, уходя,
могли сказать правду?
0
0
0
0
Учитель пригласил
30 мальчиков и 40 девочек и расставил их по
кругу так, что напротив
рядом стоящей пары
3«мальчик-девочка» тоже
стоит пара «мальчикдевочка» (а дети в этих
парах могут стоять как
угодно). Сколько девочек
стоят точно напротив девочек?
На плоскости нарисован
правильный
пятиугольник
ABCDE. Лёня хочет раскрасить
его стороны фломастерами трех
4цветов так, чтобы каждая сторона была бы одного цвета, а любые две стороны одного цвета не
имели бы общих вершин.
Сколькими способами Лёня
сможет это сделать?
В некоторых ячейках
стеклянной коробки 3×3×3 лежит по одной конфете. Дима,
Сережа и Лена смотрят на эту
5
коробку с трех сторон: Дима –
спереди, Сережа – сверху, а Лена – сбоку. Сколько конфет может лежать в коробке, если все
они видят по 9 конфет?
Сколькими способами можно расставить в
клетках таблицы 510
6
числа 1 и 1 так, чтобы
произведение чисел в
любой строке и в любом
столбце равнялось бы 1?
Введём новую
операцию:
ab

наибольшее из чисел
2a и a+b.
Решите уравнение:
2 (x (–
2))=2011.
Сколько среди
чисел 2х+у, х  у, х 
2у, у  2х может быть
положительных?
(Укажите все варианты.)
Саня дружит с Алексеем, Верой,
Глебом, Дашей и Евгением. У него День
Рождения. Если пригласить Алексея, то
тогда надо пригласить и Веру. Обязательно нужно пригласить либо Глеба,
либо Веру, но приглашать их вместе не
стоит. Также нужно пригласить либо Дашу, либо Евгения, либо обоих вместе.
Дашу можно приглашать либо вместе с
Глебом, либо никого из них. А если пригласить Евгения, то тогда необходимо
пригласить Алексея и Дашу. Кого же всётаки надо пригласить?
Петя, Дима, Миша, Саша и Илья играют в мафию. Среди них два мафиози, два
мирных жителя и комиссар. Мафиози знают
только друг друга, комиссар знает роль каждого, мирные жители не знают роли других
игроков. Мафиози всегда лгут. Комиссар и
мирные жители говорят правду. Мальчики
сделали следующие заявления:
Петя: «Я не знаю, кто Дима»;
Дима: «Я знаю, кто комиссар»;
Миша: «Я знаю, кто Петя»;
Саша: «Я знаю, что Миша – комиссар».
Кто Илья?
Найдите сумму:
1
1
1
1
1
1
3
1
4
1
5
6
1
100
Очень умный Знайка пришел в
гости к братьям-близнецам Винтику и
Шпунтику, зная, что один из них никогда
не говорит правду, и спросил одного из
них: «Ты Винтик?» «Да»,-ответил тот.
1

Когда Знайка спросил «Ты Винтик?» вто1
2
1 рого брата, то получил ответ и сразу же
3
1
4
1определил, кто есть кто. Определите, как
5
1
звали
первого брата и второго брата, и
6
100
что
сказал каждый из них: правду или
ложь.
Долго-долго Марьванна выписывала все произведения восьми попарно
различных
натуральных
сомножителей, каждый из
которых меньше 17, и никакие два из которых не дают в
сумме 17. Затем она сложила
все полученные произведения. Какое число у нее получилось?
4
В компании из 100 друзей, среди которых есть рыцари, лжецы и нормальные люди были произнесены
следующие высказывания: «Среди моих друзей 0
лжецов»; «Среди моих друзей 1 лжец»; и т.д. до
фразы «Среди моих друзей 99 лжецов». Через минуту в той же компании прозвучали те же фразы, но
каждую фразу произнёс не тот человек, что в первый раз. Еще через минуту в той же компании прозвучали те же фразы, но ни один человек не повторил уже произнесенных им фраз, и так далее. Каждый произнёс 99 различных фраз. Сколько лжецов
могло находиться в этой компании?
АБАКА-ЮНИОРЫ
Деятельная
0
0
0
0
Геометрическая
Целые числа
По дороге из леса в поселок спешат Красная Шапочка и Волк, а навстречу им
идут Бабушка и Охотник.
Скорость
сближения Волка и
1
Бабушки 18 км/ч, Красной
Шапочки и Бабушки  8 км/ч,
Охотника и Красной Шапочки
 12 км/ч. С какой скоростью
сближаются Волк и Охотник?
Сегодня 14.02.2011.
Разрежьте фигуру на Рассмотрим эту дату как 4
рисунке на две равные части.
двузначных или однозначных
числа 14, 02, 20, 11. Например, 14 делится на 2, но 14 и
11 не делятся одно на другое.
Найдите две ближайшие даты (одну в прошлом, другую
в будущем), чтобы НОД любых двух из получившихся
четырёх чисел был больше 1.
Методкомиссия в неполном составе собралась на турнир. Через день
приехал Дмитрий Валерьевич, и Игорь
Соломонович записал, во сколько раз увеличилась методкомиссия. Потом приехал
еще Александр Сергеевич, и Игорь Соломонович
опять записал, во сколько раз
2
увеличилась методкомиссия. То же самое
он сделал после приезда Алексея Владимировича, и, наконец, после приезда Леонида Михайловича, который приехал последним. Произведение четырёх записанных чисел оказалось равным 4/3. Сколько
человек входит в методкомиссию?
Проведите из одной точки на плоскости
пять лучей так, чтобы
среди образованных ими
углов было ровно четыре
острых. Рассматриваются углы не только между
соседними, но и между
любыми двумя лучами.
Сколько существует
натуральных
чисел, у которых самый большой делитель, отличный от этого числа, равен 323?
За столом сидят несколько
мальчиков и 5 девочек, а на тарелке
лежат 60 булочек. Каждая девочка дала по булочке (с тарелки) каждому
3
знакомому ей мальчику, а каждый
мальчик дал по булочке (с тарелки)
каждой незнакомой ему девочке. После этого оказалось, что все булочки
розданы. Сколько было мальчиков?
Биссектрисы треугольника ABC пересекаются в точке
I. Через точку I проходят две
прямые, которые параллельны
прямым AB и AC и пересекаются с BC в точках D и E. Найдите
периметр треугольника IED,
если AB=10, BC=11, CA=12.
Найдите целые положительные числа a, b, c, для
которых
НОК(a,b)=210,
НОД(a,b) = 10,
НОК(a,c)=110, НОД(a,c) = 2.
Четверо подружек поделили между собой 1001 конфету, при этом каждой девочке досталось конфет или
4
столько
же, сколько у какойто из ее подружек, или ровно
в два раза меньше, чем у одной из них. Как могли распределиться конфеты?
Дан
равнобедренный
треугольник ABC с основанием
AC и углом B=700. На отрезках
AB и AC даны точки D и E соответственно
так,
что
DA+AE=AC. На отрезках AC и
BC даны точки F и G соответственно так, что FC+CG=AC.
Точка E лежит строго между A
и F. Найдите угол между прямыми DF и EG.
Каждую
минуту
написанное на доске натуральное число x заменяют на
число x+k, где k – наибольший делитель x, не равный
ему. Вначале на доске написано чётное число N, не
кратное 4. Какое число будет
написано после 10 замен?
5
0
0
В полдень черепаха начала
ползти домой, каждый час поворачивая или налево, или направо (между
поворотами она ползла прямолинейно).5 За первый час она проползла 50
метров, а за каждый последующий час
 на 50 метров больше, чем в предыдущий. На каком наименьшем расстоянии от места старта она могла оказаться в 7 часов вечера?
Вдоль дороги расставлены светофоры на расстоянии 10 км. Они работают так: последние 5 минут каждого часа там горит красный свет,
остальное
время – зеленый. Машина
6
ехала по этой дороге 10 часов с постоянной скоростью, при этом ни разу не
остановившись на светофорах на
красный свет. Какое наибольшее расстояние она могла проехать?
Разрежьте по клеточкам
фигуру на две части, из которых
можно составить квадрат (черные
клетки – это
дырки). Части
можно поворачивать, но
не переворачивать.
В узлах клетчатой бумаги
живут садовники, а вокруг них
повсюду растут цветы. За любым
цветком должны ухаживать три
ближайших к нему садовника.
(Иногда несколько садовников
находятся на одинаковом расстоянии - в таком случае за цветком
ухаживают все они.) Нарисуйте
участок, за которым должен ухаживать каждый садовник.
6
На
какое
наибольшее
количество нулей может
оканчиваться произведение трёх натуральных чисел, сумма которых равна 407?
Рассматриваются
все
девятизначные
числа, у которых цифры идут в порядке невозрастания. Найдите
остаток от деления
суммы всех этих чисел
на 9.
АБАКА-СТАРТ
Комбинаторика
0
0
0
0
0
Логика
Арифметика
Разрежьте квадрат
на две равные (и по
форме, и по числу клеточек) части так, чтобы
сумма чисел в каждой
части равнялась 13.
8
2
6 4
В посёлке 2011 жителей. Однажды каждый житель
переехал в другой домик, причём в каждый домик переехал
ровно один житель. Какого ми1
нимального количества цветов
всегда будет достаточно, чтобы
покрасить домики
так, что
каждый житель переехал из домика одного цвета в домик другого цвета?
В турнире участвовало пятеро девочек – две пятиклассницы
и три четвероклассницы. Сначала
Анна выиграла у Марии, Софья с
Ириной сыграли вничью, Анна выиграла у Ольги, а Ольга – у Софии.
В каждом туре встречались девочки из разных параллелей. Как зовут
четвероклассниц?
На канале АбакаТВ проходит игра. Ведущая задает вопрос, сколько прямоугольников
можно увидеть на полоске 1×n. Но
изображение
на экране пропало, и
2
ведущая подсказала, что верный
ответ больше 2011, но меньше
2020. Чему равно может равняться
n? (Например, на полоске 1×3
можно видеть 6 прямоугольников).
Шпион отправил короткое шифрованное сообщение. Два контрразведчика
перехватили и попытались расшифровать
его. Первый получил расшифровку «вариант», второй – «неумёха». Известно, что в
исходном сообщении не было подряд
идущих гласных. Кроме того, установлено,
что каждый символ правильно расшифрован хотя бы одним из контрразведчиком,
но ни один из них не расшифровал верно
два подряд идущих символа. Каким могло
быть исходное сообщение?
Какое наименьшее
число жильцов можно
вселить в 30-квартирный
3
дом
так, чтобы в любых
трёх произвольно взятых
квартирах жило не менее
7 человек?
В семье есть Иван
Сидорович, Сидор Иванович, Сидор Петрович, Петр
Сидорович, Петр Петрович.
Один из них сейчас смотрит телевизор, его отец
дремлет, брат читает газету, а дети ушли гулять. Как
зовут того, кто смотрит телевизор?
На плоскости нарисован
правильный
пятиугольник
ABCDE. Лёня хочет раскрасить
его стороны фломастерами трех
цветов
так, чтобы каждая сто4
рона была бы одного цвета, а
любые две стороны одного цвета не имели бы общих вершин.
Сколькими способами Лёня
сможет это сделать?
В комнате было несколько
человек (не меньше трёх). Один сказал: «Нас тут не больше пяти» – и
ушел. Следующий сказал: «И теперь
нас не больше пяти», и тоже ушёл.
После этого каждую минуту кто-то
уходил, сказав на прощание: «Тот,
кто ушёл до меня, перед уходом солгал», пока комната не опустела.
Сколько человек, уходя, могли сказать правду?
Использовав пять
двоек, знаки арифметических операций и скобки, получите число 17.
Например,
10
можно получить поразному:
10 = 2+2+2+2+2 =
2
2 +22+2 = 2(22+2:2).
В мешке у Деда Мороза
лежат конфеты нескольких видов (более двух), причем их не
5
обязательно поровну. Дед Мороз знает, что если вынуть любые 100 конфет из мешка, то
среди них обязательно найдутся
В школе прошёл забег с
участием 10 спортсменов, и все
заняли разные места. На следующий день каждого из них спросили, какое место он занял, и каждый, естественно, назвал одно число от 1 до 10. Сумма их ответов
В автобусе ехали взрослые
и дети, причем число взрослых
относилось к числу детей как 2:3.
После того, как четыре пассажира
вышли (и никто не вошел), число
взрослых стало относиться к числу
детей как 3:4. Сколько пассажиров
7
5
1
Найдите все решения ребуса:
AБA=КА
(разные цифры заменены разными буквами, одинаковые  одинаковыми. Числа на ноль
не начинаются).
Запишем
сегодняшнюю
дату как 14022011. Теперь разрешается взять любые три цифры,
стоящие подряд, и поменять их
порядок как угодно. Всего таких
операций можно сделать 4 штуки.
Какое наибольшее число можно
получить, проделав эти четыре
операции?
конфеты всех видов, кроме, оказалась
равна
36.
Какое первоначально ехало в автобусе,
может быть, одного. Какое наименьшее число врунишек бы- если известно, что их было меньше
60 (перечислите все возможности)?
наибольшее количество конфет ло?
может быть в мешке у Деда
Мороза?
0
Пятизначное число поделили на 100, после чего сложили
неполное частное и оста6
ток. Для какого количества чисел полученный результат будет делиться на 11?
Очень умный Знайка пришел в гости к братьям-близнецам
Винтику и Шпунтику, зная, что
один из них точно никогда не говорит правду, и спросил первого:
«Ты Винтик?» «Да»,-ответил тот.
Когда Знайка спросил «Ты Винтик?» второго брата, то получил
ответ и сразу же определил, кто
есть кто. Определите, как звали
первого брата и второго брата, и
что сказал каждый из них: правду
или ложь.
8
a, b, c – цифры,
а≠c.
Квадраты чисел abc и cba –
пятизначные числа ABCDE и
EDCBA . Найти все такие трехзначные числа. (а≠c, остальные
буквы могут обозначать разные
цифры, а могут и одинаковые.
Числа с нуля не начинается.)
АБАКА-СТАРТ
Деятельная
0
0
0
0
0
У Алисы живет крокозябра.
Каждый день она съедает бананов
ровно в два раза больше своего веса,
а каждую ночь худеет в три раза.
1
Уезжая рано утром на четырёхдневные каникулы, Алиса оставила ей 40
кг бананов, и этого крокозябре в
точности хватило. Сколько весила
крокозябра, когда Алиса уезжала?
Вдоль железной дороги стоят километровые столбы на расстоянии 1 км друг от друга. Один из них
покрасили в желтый цвет и шесть –
2
в красный. Сумма расстояний от
желтого столба до всех красных
равна 14 км. Чему может быть равно
максимальное расстояние между
красными столбами?
После того, как на
борт были подняты 30 потерпевших кораблекрушение, оказалось, что за3
пасов
питьевой воды хватит только на 50 дней, а
не на 60, как раньше.
Сколько людей было на
корабле вначале?
На празднике конфет можно
было обменять любые три фантика
на карамельку в фантике и любые
пять фантиков на шоколадную конфету
4 в фантике. Миша принес на
этот праздник 50 фантиков, при
этом он хочет съесть карамелек в
три раза больше, чем шоколадных.
Какое наибольшее количество конфет ( в сумме) он сможет съесть?
В полдень черепаха начала
ползти домой, каждый час поворачивая или налево, или направо
(между поворотами она ползла прямолинейно). За первый час она про5
ползла 50 метров, а за каждый последующий час  на 50 метров
больше, чем в предыдущий. На каком наименьшем расстоянии от места старта она могла оказаться в 7
часов вечера?
Клеточная
Лена разрезала квадрат
5  5 на уголки, состоящие из
трёх клеток, и прямоугольники, состоящие из двух клеток.
Какое наименьшее число частей она могла получить?
Разрежьте фигуру на рисунке
на две равные части.
В стеклянной коробке размером
3×3×3 ячейки в некоторых ячейках лежит
по одной конфете. Дима, Серёжа и Лена
смотрят на эту коробку с трех сторон: Дима – спереди, Серёжа – сверху, а Лена –
сбоку. Сколько конфет может лежать в коробке, если все они видят по 3 конфеты
(если какие-то конфеты лежат друг за другом, то наблюдатели видят только первую
конфету)?
Сколькими способами к фигурке
(см.рис., длина стороны клетки 1 см)
можно приставить по стороне еще
одну клетку так, чтобы полученной фигуркой из пяти клеток можно было бы оклеить
5 граней куба с ребром 1см?
Конь обошел все клетки доски 6×6.На
рисунке частично отмечены клетки с указанием
номера хода, которым конь попал в отмеченную
клетку. Укажите, где будет находиться клетка с
номером 21.
17
11
2
25
23
1
30
19
15
13
8
35
9
Целые числа
Сегодня 14.02.2011. Рассмотрим эту дату как 4 двузначных или однозначных числа 14,
02, 20, 11. Например, 14 делится
на 2, но 14 и 11 не делятся одно
на другое. Найдите две ближайшие даты (одну в прошлом, другую в будущем), чтобы было
верно, что для любых двух из
получившихся четырёх чисел
одно делится на другое.
Назовём натуральное
число удивительным, если оно
равно произведению всех своих натуральных делителей,
кроме самого числа. Например, 6  первое удивительное
число. Найдите десятое удивительное число.
Трёхзначное число таково, что
если в нем стереть последнюю
цифру, то полученное двузначное
число кратно 7, если стереть
цифру десятков, то полученное
двузначное число кратно 11, если
стереть цифру сотен, то полученное двузначное число кратно 13.
Найдите все такие числа.
Два
последовательных трёхзначных
числа выписали друг за
другом. Оказалось, что
полученное шестизначное число делится на
499. Каким могло быть
это шестизначное число?
Каждую минуту
написанное на доске
натуральное число x
заменяют на число x+k,
где k – наибольший делитель x, не равный
ему. Вначале на доске
написано чётное число
N, не кратное 4. Какое
число будет написано
после 10 замен?
0
Дана доска 2020. Корабль
— это связная клеточная фигура
доски. Флот — это такой набор нескольких кораблей, что никакие два
корабля не пересекаются (т.е. не
имеют даже общих вершин). Какое
наименьшее число клеток может
6
содержать
флот, к которому нельзя
добавить ни одного корабля?
(Связная клеточная фигура
— это такое непустой набор клеток,
что из любой клетки этого набора
можно пройти в любую другую
клетку этого набора по клеткам этого набора, переходя каждый раз в
соседнюю по стороне клетку.)
Ванечка покрасил снаружи кубик размера 3см×3см×3см в синий цвет, а потом разрезал его на кубики размера 1см×1см×1см. Петечка сложил из некоторых из этих кубиков пирамидку: 1ряд - 9 кубиков, 2 ряд - 7 кубиков,..., 5
ряд - 1 кубик. Вот так:
После этого Ванечка поглядел на получившуюся пирамидку спереди и обнаружил,
что все видимые 25 граней синие. Какое максимальное количество синих граней кубиков в
общей сложности можно увидеть, если осмотреть эту пирамидку со всех шести сторон?
10
На
какое
наибольшее количество
нулей может оканчиваться
произведение
трёх натуральных чисел, сумма которых
равна 407?
0
0
АБАКА-СЕНЬОРЫ. ОТВЕТЫ
КомбиЦелые
наторика
числа
5
04.04.201
1
2 и 30.10.2010
20
a=10,
2
b=210, c=22
3
30
Логи1
3
12
Дашу и
81N/2
4
2011=
2*1006^2-
Илья –
мирный житель
2801
Геомет-
Ал-
рия
гебра
11
-200
55
(или
125 - засчитывается любой ответ)
60
143,
286,286,286
или
91,182,364,3
64
0
Глеба
0
0
Де
ятельная
16
ка
-2011
2009/2=
1004.5
1
2*1005^2+2
011=
2022061
(любая форма ответа)
236
5
0 или 1
0
34
178
3/70
0
6
1008
n+1 ≤ x
29
120
0 км
≤n+1+k
0
АБАКА-ЮНИОРЫ. ОТВЕТЫ
КомбиАлнаторика
гебра
3
-40
1
Логика
Пётр
Си-
Квадрат, из которого вырезаны 4 угловых квадратика со стороной ¼ .
Деятельная
22
Геометрия
Целые числа
30.12.2
010, 02.02.2012
дорович
0
2
5
-1
1
16
верьте
нок!
0
3
40
0
4
30
0
200
9/2=1004.5
0,1,
2,3
Дашу
и
12
Прорису11
Глеба
Илья
мирный житель
–
143,286,286
,286
или
91,182,364,
–0,75
55
(или 125 - засчитывается
любой ответ)
7
a = 10,
b = 210, c = 22
81N/2
364
5
От 9 до 27
1 - Шпун-
1
0
6
тик,
0
2 - Винтик,
оба соврали
6
236
178
1200 км
0, 1
0
0
АБАКА-СТАРТ. ОТВЕТЫ
Ко
Логика
мбинаторика
3
Мария,
1
Ольга, Ирина
Арифметика
Деятельная
5
11
Квадрат,
из которого вырезаны 4 угловых
квадратика со стороной ¼ .
Клеточная
9
0
Целые
числа
20.10.2010
и 01.01.2020
2
Есть и
другой способ
212=42,
262=52,
313=93
ВЕР-
63
МАХТ
0
3
8
Пётр Си-
42211001
150
3, 4
любое
число не меньше 2
(2+2)2+2:
16
6
88
565
дорович
0
4
30
0
2=17
другие
ты!)
5
297
6
1
818
3
1 - Шпунтик,
2 - Винтик,
оба
166167
665666
(есть и
вариан25
0
311
и
113, 301 и 103,
221 и 122, 211 и
112, 201 и 102.
49
0
0
33
со-
врали
12
Нижняя
строка, четвёртый
слева столбец
50
81N/2
6
и
Результаты математической абаки
юниорская группа
старшая группа
команды
Комби
Чис
Лог
деят
Гео
Алг
премия по строкам
10
20
30
40
50
1
1
1
1
1
Ст
вр
Ижевск-8-1
СанктПетербург239-8-1
ФракталЮМШ-8
260
260
150
150
150
260
90
140
90
200
130
260
60
150
140
260
70
100
Казань 8
СПБ-239-7-81
50
150
100
200
120
30
100
100
150
190
50
70
Киров-8-1
1514-8
Москва
120
150
100
120
70
80
0
110
150
200
70
100
Пермь9-8-1
Курган
ЦДМО-8
Екатеринбург
гимназия 9-8
110
150
100
40
100
60
1
110
120
100
60
100
60
1
30
150
150
60
30
70
1
Омск-8-1
ДваждыДва8
30
150
100
50
100
40
20
150
150
60
20
30
Л2Ш-8-1
30
80
80
70
60
100
Озерск 8
30
80
100
100
70
20
1
Ижевск-8-2
Новосибирск
-1
30
30
100
110
30
100
30
50
150
20
70
60
Пермь9-8-2
60
120
100
50
10
20
Батарея
Екатеринбург
гимназия 9 82
10
70
100
40
30
100
20
100
100
20
60
30
Киров-8-2
10
10
150
10
10
100
Казань ТТЛ
30
20
100
20
30
20
Барнаул
10
20
80
30
40
50
Омск-8-2
30
20
60
10
30
70
Л2Ш-8-2
30
10
100
30
10
30
СПБ-239-7-1
1514-7
Москва
Казань 7-1
ДваждыДва7-1
Новосибирск
-2
Нижнекамск
7
Киров-7-1
Ижевск-7
Екатеринбург
гимназия 9-7
Киров-7-2
Обычные
люди
Ульяновск-7
30
60
260
110
100
260
120
90
100
60
260
150
60
160
30
30
160
180
1
60
10
260
170
90
10
1
50
10
180
80
150
180
0
130
0
60
110
50
100
140
130
150
120
130
100
10
260
30
110
0
100
0
0
10
10
30
30
10
260
150
30
110
130
80
60
130
1
10
10
70
60
150
190
60
60
100
30
100
120
1
1
170
1550
I
0
50
960
II
1
20
50
850
1
20
20
690
III
4
1
10
660
10
660
5-6
5-6
0
630
40
600
7
8
1
30
580
9
1
30
520
0
470
10
11
20
450
10
430
20
420
1
20
420
1
20
400
0
360
0
350
20
350
10
300
20
240
0
230
0
220
10
220
18-19
20
21
22
23-24
23-24
1
1
1
1
1
1
1
13
место
150
0
1
итого
60
1
12
13
14-15
14-15
16
17
18-19
0
50
870
I
30
0
20
780
670
II
III
10
50
660
4
650
5
620
620
560
6-7
6-7
8
10
0
530
510
9
10
10
10
500
480
11
12
50
10
группа "Старт"
СПб-30-7
СПб-ЮМШ-7
Фрактал-7
Пермь9-7-2
КЛАСнО
Набережные
Челны 7
ДваждыДва7-2
Л2Ш-7-1
Омск-7
Нижний Тагил 7
Казань 7-2
Красноярск-7
Л2Ш-7-2
0
0
0
10
0
470
470
470
440
390
13-15
13-15
13-15
16
17
80
0
380
18
80
60
30
60
40
100
0
10
0
370
360
340
19
20
21-22
60
100
30
70
10
10
0
10
1
0
0
0
10
340
290
280
240
1040
21-22
23
24
25
26
I
0
30
30
30
120
70
30
0
30
10
260
260
150
150
30
30
10
60
100
150
10
60
150
110
30
100
80
80
10
50
60
30
120
60
30
10
30
10
40
10
30
160
150
140
20
60
30
0
0
150
30
60
10
10
10
150
100
80
100
60
70
10
10
1
1
Пермь9-7-1
10
30
100
10
30
10
1
10
Ижевск-6
ДваждыДва6-1
ДваждыДва6-2
260
140
180
170
180
40
1
10
110
170
150
190
180
90
0
890
II
140
120
140
140
180
140
0
860
Ижевск 6-2
140
170
130
110
120
60
0
730
Казань-6
СанктПетербург239-6
Набережные
Челны 6-1
Курган
ЦДМО-6
50
60
150
100
60
100
620
III
4
5
30
110
150
40
100
60
100
150
60
80
170
100
Пермь9-6
Татарстан 56
70
50
150
0
60
Фрактал-6
1514-6
Москва
40
1
200
60
1
50
80
1
30
540
6-7
100
40
1
30
540
6-7
30
30
60
10
480
60
30
50
0
410
8
9
100
40
150
50
0
400
110
80
10
70
50
0
360
0
120
100
10
120
0
0
350
Киров-6
Набережные
Челны 6-2
40
110
70
40
30
40
10
340
50
20
100
0
110
30
0
310
Казань 5-6
Киров-5
10
0
60
60
80
100
10
0
70
70
10
0
10
0
250
230
Озерск 6
0
140
40
0
0
0
0
180
ЧелКор
0
60
10
50
10
0
0
130
1
1
1
14
50
10
11
12
13
14
15
16
17
18
КОМАНДНАЯ МАТЕМАТИЧЕСКАЯ ОЛИМПИАДА 15.02.2011
ЗАДАНИЯ ДЛЯ ГРУППЫ «СТАРТ»
1. Мотоциклисты Вася и Петя ездят с постоянными скоростями по круговому треку
длиной 1км. Вася обнаружил, что Петя каждые 2 минуты его обгоняет. Тогда он вдвое увеличил скорость и теперь уже сам каждые 2 минуты стал обгонять Петю. С какими скоростями
ехали мотоциклисты изначально?
2. На острове рыцарей и лжецов два племени. Каждый островитянин произнес фразу:
«В моем племени лжецов больше, чем в соседнем». Может ли на острове быть ровно 2011 жителей?
3. В кружок поступили 20 школьников, среди которых Вася и Петя. Оказалось, что
каждый из поступивших знает ровно четверых других, а у Васи и Пети трое общих знакомых. Докажите, что среди поступивших в кружок есть школьник, не знакомый ни с Васей, ни
с Петей и даже не имеющий ни с кем из них общих знакомых.
4. Дано 8 трехзначных чисел. Докажите, что из них можно выбрать два и записать их
подряд таким образом, что получившееся шестизначное число будет делиться на 7.
5. Можно ли на шахматную доску 88 поставить 16 не бьющих друг друга королей
так, чтобы в каждой строке и каждом столбце стояло по 2 короля?
6. Пусть e(k) — количество четных натуральных делителей натурального числа k, а
o(k) — количество его нечетных натуральных делителей. Докажите, что e(1)+e(2)+…+e(1000)
меньше, чем o(1)+o(2)+…+o(1000).
7. На окружности отмечено 20 точек и проведен 21 соединяющий их отрезок. Докажите, что из этих отрезков можно выбрать три, составляющие несамопересекающуюся ломаную.
ЗАДАНИЯ ДЛЯ ЮНИОРОВ
1. На острове живут три племени. Каждый житель острова является либо рыцарем (то
есть всегда говорит правду), либо лжецом (то есть всегда врет). В один прекрасный день
каждый житель острова сказал: «В одном из других племен лжецов меньше, чем в нашем».
Может ли на этом острове быть ровно 2011 жителей?
2. В кружок поступили 20 школьников, среди которых Вася и Петя. Оказалось, что
каждый из поступивших знает ровно четверых других, а у Васи и Пети трое общих знакомых. Докажите, что среди поступивших в кружок есть школьник, не знакомый ни с Васей, ни
с Петей и даже не имеющий ни с кем из них общих знакомых.
3. Натуральное число n выбрано между двумя квадратами последовательных натуральных чисел, причем меньший из этих квадратов меньше n ровно на a, а больший —
больше n ровно на b. Докажите, что число n–ab — квадрат целого числа.
4. Точка D — середина стороны AC треугольника ABC. Известно, что BC = BD. На
продолжении стороны BC за точку C отмечена точка E, такая, что CE = CD. Оказалось, что
DE  AB. Во сколько раз отрезок BD больше отрезка AD?
5. Какое наибольшее количество слонов можно расставить на доске 77 так, чтобы
каждый слон бил четное число слонов?
6. На окружности отмечено n точек и проведен n+1 соединяющий их отрезок. Докажите, что из этих отрезков можно выбрать три, составляющие несамопересекающуюся ломаную.
7. Пусть e(k) — количество четных натуральных делителей натурального числа k, а
o(k) — количество его нечетных натуральных делителей. Докажите, что e(1)+e(2)+…+e(n)
отличается от o(1)+o(2)+…+o(n) меньше, чем на n.
8. Пусть x, y и z такие вещественные числа, что x  4, y  5, z  6 и x2+y2+z2  90. Докажите, что x+y+z  16.
ЗАДАНИЯ ДЛЯ СЕНЬОРОВ
15
1. На острове живут три племени. Каждый житель острова является либо рыцарем (то
есть всегда говорит правду), либо лжецом (то есть всегда врет). В один прекрасный день
каждый житель острова сказал: «В одном из других племен лжецов меньше, чем в нашем».
Может ли на этом острове быть ровно 2011 жителей?
2. 1000 пилюль весом 0,38 г и 5000 пилюль 0,038 г раскладывают по чашкам вместимостью 1 г каждая. Каким наименьшим количеством чашек удастся обойтись?
3. Какое наибольшее количество слонов можно расставить на доске 44 так, чтобы
каждый слон бил четное число слонов?
4. В четырехугольнике ABCD диагонали перпендикулярны, а стороны AB и DC параллельны. Докажите, что BCDA  ABCD.
5. В теннисном турнире с участием 10 игроков каждые два участника сыграли между
собой одну партию. При этом оказалось, что если A выиграл у B, то сумма количества проигрышей игрока A и количества побед игрока B не меньше 8. Докажите, что игроков, одержавших в этом турнире ровно 4 победы, столько же, сколько игроков, одержавших ровно 5
побед. Напомним, что ничьих в теннисе не бывает.
6. Пусть e(k) — количество четных натуральных делителей натурального числа k, а
o(k) — количество его нечетных натуральных делителей. Докажите, что e(1)+e(2)+…+e(n)
отличается от o(1)+o(2)+…+o(n) меньше, чем на n.
7. Вершины A и E квадрата ABCD и прямоугольника EBFD лежат по одну сторону от
общей диагонали BD. Точка G на прямой BE такова, что AG перпендикулярно AE. Докажите,
что BG = ED.
8. В выпуклом n-угольнике провели 2kn+1 диагональ. Докажите, что можно выбрать
2k+1 диагоналей, составляющих несамопересекающуюся ломаную.
РЕШЕНИЯ ЗАДАЧ КОМАНДНОЙ ОЛИМПИАДЫ ГРУППЫ «СТАРТ»
Задача 1. Мотоциклисты Вася и Петя ездят с постоянными скоростями по круговому треку длиной 1км. Вася обнаружил, что Петя каждые 2 минуты его обгоняет. Тогда
он вдвое увеличил скорость и теперь уже сам каждые 2 минуты стал обгонять Петю. С
какими скоростями ехали мотоциклисты изначально?
Ответ. 60 км/ч и 90 км/ч. Решение. Петя каждые две минуты проезжал на километр
больше Васи, то есть за час — на 30 км больше. После того, как Вася увеличил скорость
вдвое, уже он стал проезжать за час на 30 км больше Пети. Значит, изначально скорость Васи
составляла 30+30 = 60 км/ч, а скорость Пети — 90 км/ч.
Задача 2. На острове рыцарей и лжецов два племени. Каждый островитянин произнес фразу: «В моем племени лжецов больше, чем в соседнем». Может ли на острове быть
ровно 2011 жителей?
Ответ. Нет. Решение. Поскольку все островитяне сказали одно и то же, каждое племя
состоит либо из одних рыцарей, либо из одних лжецов. Из одних рыцарей оно состоять не
может: тогда получилось бы, что в другом племени отрицательное количество лжецов. Значит, оба племени состоят из одних лжецов. Поскольку число 2011 нечетно, в одном из племён больше жителей, чем в другом. Но тогда все лжецы из более многочисленного племени
говорят правду, что невозможно.
Задача 3. В кружок поступили 20 школьников, среди которых Вася и Петя. Оказалось, что каждый из поступивших знает ровно четверых других, а у Васи и Пети трое общих знакомых. Докажите, что среди поступивших в кружок есть школьник, не знакомый ни
с Васей, ни с Петей и даже не имеющий ни с кем из них общих знакомых.
Решение. Пусть Петя и Вася знакомы с Аней, Борей и Вовой, Петя кроме того знаком
с Гришей, а Вася — с Димой. У Гриши и Димы, кроме Пети с Васей есть ещё, самое большее, 3+3 = 6 разных знакомых, а у Ани, Бори и Вовы — тоже 2+2+2 = 6 разных знакомых,
16
всего — не больше 12. Ещё семеро — Петя, Вася, Аня, Боря, Вова, Гриша, Дима, всего — 19.
Сюда входят все знакомые Пети и Васи и знакомые их знакомых. Поэтому в кружке из 20
человек найдётся школьник, не входящий в их число, что и требовалось доказать.
Задача 4. Дано 8 трехзначных чисел. Докажите, что из них можно выбрать два и
записать их подряд таким образом, что получившееся шестизначное число будет делиться
на 7.
Решение. Если трехзначные числа a и b записаны подряд, то получившееся шестизначное число равно 1000a+b = 1001a+(b–a). Поскольку 1001 делится на 7, 1000a+b делится на 7 тогда и только тогда, когда делится на 7 разность b–a. Осталось заметить, что среди любых восьми целых чисел найдутся два, дающие одинаковые остатки при делении на 7.
Задача 5. Можно ли на шахматную доску 88 поставить 16 не бьющих друг друга королей так, чтобы в
каждой строке и каждом столбце стояло по 2 короля?
Ответ. Да. Решение. Пример — на рисунке справа.
Задача 6. Пусть e(k) — количество четных натуральных делителей натурального числа k, а o(k) — количество его нечетных натуральных делителей. Докажите,
что
e(1)+e(2)+…+e(1000)
меньше,
чем
o(1)+o(2)+…+o(1000).
К
К
К
К
К
К
К
К
К
К
К
К
К
К
К
К
Решение. Выпишем на доску все делители чисел 1, 2, …, 1000. Поскольку чисел, делящихся на 1, среди чисел от 1 до 1000 не меньше (и даже больше), чем делящихся на 2,
единиц выписано больше, чем двоек. По аналогичной причине троек выписано не меньше,
чем четверок, пятерок — не меньше, чем шестерок, …, чисел 999 — не меньше, чем чисел
1000, откуда все и следует.
Задача 7. На окружности отмечено 20 точек и проведен 21 соединяющий их отрезок. Докажите, что из этих отрезков можно выбрать три, составляющие несамопересекающуюся ломаную.
Решение. Если среди наших точек есть такая, из которой выходит не больше одного
отрезка, удалим ее вместе с отрезком и будем так делать до тех пор, пока есть такие точки.
Отрезков при этом всегда будет оставаться больше, чем точек. Поскольку трех точек с четырьмя отрезками не бывает, в какой-то момент у нас возникнет ситуация, когда из каждой
точки выходит не меньше двух отрезков, и есть точка A, из которой выходит не меньше трех
отрезков (иначе отрезков не больше, чем точек). Пусть AB, AC и AD — три выходящих из нее
отрезка, причем луч AC лежит внутри угла BAD. Рассмотрим второй отрезок CE, выходящий
из точки С. С одной из точек B и D он лежит по разные стороны от прямой AC. Если это B,
искомой будет ломаная BACE, а если D — ломаная DACE.
РЕШЕНИЯ ЗАДАЧ КОМАНДНОЙ ОЛИМПИАДЫ МЛАДШЕЙ ГРУППЫ
Задача 1. На острове живут три племени. Каждый житель острова является либо
рыцарем (то есть всегда говорит правду), либо лжецом (то есть всегда врет). В один прекрасный день каждый житель острова сказал: «В одном из других племен лжецов меньше,
чем в нашем». Может ли на этом острове быть ровно 2011 жителей?
Ответ: Нет. Решение. Поскольку в каждом племени все островитяне утверждают одно
и то же, каждое племя состоит либо из одних рыцарей, либо из одних лжецов. Из одних рыцарей оно состоять не может: тогда получилось бы, что в другом племени отрицательное количество лжецов. Значит, все три племени состоят из одних лжецов. Возьмем самое многочисленное племя М. Поскольку 2011 не делится на 3, найдется племя, в котором людей
меньше. Получается, что все люди их племени М говорят правду, что невозможно.
17
Задача 2. В кружок поступили 20 школьников, среди которых Вася и Петя. Оказалось, что каждый из поступивших знает ровно четверых других, а у Васи и Пети трое общих знакомых. Докажите, что среди поступивших в кружок есть школьник, не знакомый ни
с Васей, ни с Петей и даже не имеющий ни с кем из них общих знакомых.
Решение. Пусть Петя и Вася знакомы с Аней, Борей и Вовой, Петя кроме того знаком
с Гришей, а Вася — с Димой. У Гриши и Димы, кроме Пети с Васей есть ещё, самое большее, 3+3 = 6 разных знакомых, а у Ани, Бори и Вовы — тоже 2+2+2 = 6 разных знакомых,
всего — не больше 12. Ещё семеро — Петя, Вася, Аня, Боря, Вова, Гриша, Дима, всего — 19.
Сюда входят все знакомые Пети и Васи и знакомые их знакомых. Поэтому в кружке из 20
человек найдётся школьник, не входящий в их число, что и требовалось доказать.
Задача 3. Натуральное число n выбрано между двумя квадратами последовательных
натуральных чисел, причем меньший из этих квадратов меньше n ровно на a, а больший —
больше n ровно на b. Докажите, что число n–ab — квадрат целого числа.
Решение. Пусть k2 < n < (k+1)2. Тогда a+b = (k+1)2–k2 = 2k+1, и n–ab = k2+a–a(2k+1–
a) = k –2ka+a2 = (k–a)2.
2
Задача 4. Точка D — середина стороны AC треугольника ABC. Известно, что
BC = BD. На продолжении стороны BC за точку C отмечена точка E, такая, что CE = CD.
Оказалось, что DE  AB. Во сколько раз отрезок BD больше отрезка AD?
Ответ: В 2 раза. Решение. Пусть углы при основании DE равнобедренного треугольника DCE равны . По теореме о внешнем угле треугольника угол BCA равен 2. Но тогда
его биссектриса параллельна прямой DE, и, стало быть, перпендикулярна AB, то есть является в треугольнике ABC биссектрисой и высотой. Следовательно, AC = BC = BD, откуда и получаем ответ.
Задача 5. Какое наибольшее количество слонов можно расставить на доске 77 так,
чтобы каждый слон бил четное число слонов?
Ответ. 41. Решение. Будем считать, что угловые клетки доски — черные. Пример:
слоны стоят на всех белых клетках доски, а также на всех черных, кроме тех, что находятся в
двух крайних вертикалях. Оценка. Заметим, что если в углу стоит слон, то больше слонов на
исходящей из этого угла диагонали нет. Стало быть, если в углах стоят два слона, то на доске
не менее 11 пустых клеток (обе диагонали минус две угловые клетки), а если один — то не
менее 8 пустых клеток (6 клеток на одной диагонали и два пустых угла на другой). В обоих
случаях на доске не более 49–8 = 41 слона. Пусть все углы пусты. Рассмотрим клетку, соседнюю с угловой по диагонали, и три соседних с ней по диагонали не угловых клетки. На всех
четырех этих клетках слоны стоять не могут, так как тогда одного из них будут бить три. Поэтому в каждой такой четверке хотя бы одна из клеток пуста, и, поскольку таких четверок
четыре, и они не имеют общих клеток, мы заработали еще 4 пустых клетки, кроме угловых,
что и завершает доказательство.
Задача 6. На окружности отмечено n точек и проведен n+1 соединяющий их отрезок. Докажите, что из этих отрезков можно выбрать три, составляющие несамопересекающуюся ломаную.
Решение. Если среди наших точек есть такая, из которой выходит не больше одного
отрезка, удалим ее вместе с отрезком и будем так делать до тех пор, пока есть такие точки.
Отрезков при этом всегда будет оставаться больше, чем точек. Поскольку трех точек с четырьмя отрезками не бывает, в какой-то момент у нас возникнет ситуация, когда из каждой
точки выходит не меньше двух отрезков, и есть точка A, из которой выходит не меньше трех
отрезков. Пусть AB, AC и AD — три выходящих из нее отрезка, причем луч AC лежит внутри
угла BAD. Рассмотрим второй отрезок CE, выходящий из точки С. С одной из точек B и D он
18
лежит по разные стороны от прямой AC. Если это B, искомой будет ломаная BACE, а если D
— ломаная DACE.
Задача 7. Пусть e(k) — количество четных натуральных делителей натурального
числа k, а o(k) — количество его нечетных натуральных делителей. Докажите, что
e(1)+e(2)+…+e(n) отличается от o(1)+o(2)+…+o(n) меньше, чем на n.
Решение. Выпишем на доску все делители чисел 1, 2, …, n. Пусть выписано s1 единиц,
s2 двоек, …, sn чисел n. Очевидно, что n = s1  s2  …  sn. Заметим, что |(o(1)+o(2)+…+o(n))–
(e(1)+e(2)+…+e(n))| равняется модулю знакопеременной суммы s1–s2+s3–…+(–1)n+1sn. Эта
сумма не больше s1 = n, поскольку группируя s3 и –s2, s5 и –s4 и т.д., получаем неположительные слагаемые (а если n четно, то непарное слагаемое –sn отрицательно). С другой стороны,
группируя s1 и –s2, s3 и –s4 и т.д., получаем неотрицательные слагаемые (а если n нечетно, то
непарное слагаемое sn положительно), так что указанная сумма не меньше 0, что и завершает
доказательство.
Задача 8. Пусть x, y и z такие вещественные числа, что x  4, y  5, z  6 и
x +y +z2  90. Докажите, что x+y+z  16.
2
2
Решение. Положим a = x–4, b = y–5, c = z–6. По условию числа a, b и c неотрицательны. Если какое-то из них не меньше 1, то x+y+z  4+5+6+1 = 16, и все доказано. Дальше будем считать, что a, b и c меньше 1. Тогда
90 ≤ x2+y2+z2 = (a+4)2+(b+5)2+(c+6)2 = a2+b2+c2+8a+10b+12c+16+25+36 ≤ a+b+c+8a+1
0b+12c+77.
Приведя подобные члены и вычтя 77, получаем, что 13(a+b+c)  9a+11b+13c  13, откуда a+b+c  1, и x+y+z = 4+5+6+a+b+c  16.
РЕШЕНИЯ ЗАДАЧ КОМАНДНОЙ ОЛИМПИАДЫ СТАРШЕЙ ГРУППЫ
Задача 1. На острове живут три племени. Каждый житель острова является либо
рыцарем (то есть всегда говорит правду), либо лжецом (то есть всегда врет). В один прекрасный день каждый житель острова сказал: «В одном из других племен лжецов меньше,
чем в нашем». Может ли на этом острове быть ровно 2011 жителей?
Ответ: Нет. Решение. Поскольку в каждом племени все островитяне утверждают одно
и то же, каждое племя состоит либо из одних рыцарей, либо из одних лжецов. Из одних рыцарей оно состоять не может: тогда получилось бы, что в другом племени отрицательное количество лжецов. Значит, все три племени состоят из одних лжецов. Возьмем самое многочисленное племя М. Поскольку 2011 не делится на 3, найдется племя, в котором людей
меньше. Получается, что все люди их племени М говорят правду, что невозможно.
Задача 2. 1000 пилюль весом 0,38 г и 5000 пилюль 0,038 г раскладывают по чашкам
вместимостью 1 г каждая. Каким наименьшим количеством чашек удастся обойтись?
Ответ. 577. Решение. Заменим каждую тяжелую пилюлю десятью легкими. От этого
наши возможности раскладывать пилюли по чашкам не ухудшатся. Так как
260,038 < 1 < 270,038, в одну чашку мы сможем положить максимум 26 пилюль. Всего пилюль в пересчете на легкие у нас 15000, а 576 < 15000:26 < 577. Поэтому чашек потребуется
не меньше, чем 577. С другой стороны, такого количества чашек
хватит, если положить в 500 чашек по две большие и 6 маленьких,
С С С
в 76 чашек по 26 маленьких пилюль, а в одну — 24 маленькие пи- С
С
люли.
С
С
Задача 3. Какое наибольшее количество слонов можно
расставить на доске 44 так, чтобы каждый слон бил четное
число слонов?
19
С С С
Ответ. 10. Решение. Пример с 10 слонами — на рисунке справа. Чтобы доказать, что
больше 10 слонов быть не может, покажем, что на полях одного цвета может стоять не
больше 5 слонов. Всего есть 8 полей данного цвета. Если слон стоит на угловой клетке, то
больше слонов на исходящей из этого угла диагонали нет, то есть всего слонов на клетках
этого цвета не больше 5. Если же в угловых клетках слоны не стоят, а во всех 6 оставшихся
клетках — стоят, то есть два слона, которых бьют три других, что противоречит условию.
Задача 4. В четырехугольнике ABCD диагонали перпендикулярны, а стороны AB и DC
параллельны. Докажите, что BCDA  ABCD.
Решение. Пусть O — точка пересечения диагоналей четырехугольника. Положим
OA = a, OB = b. Треугольник OAB подобен треугольнику OCD по двум углам с некоторым
коэффициентом k, поэтому OC = ka, OD = kb. По теореме Пифагора AB2 = a2+b2,
CD2 = k2a2+k2b2, BC2 = k2a2+b2, AD2 = a2+k2b2. Задача свелась к доказательству неравенства
(k2a2+b2)(a2+k2b2)  (a2+b2)(k2a2+k2b2), которое после раскрытия скобок и приведения подобных членов сводится к очевидному неравенству (k2–1)2a2b2  0.
Задача 5. В теннисном турнире с участием 10 игроков каждые два участника сыграли между собой одну партию. При этом оказалось, что если A выиграл у B, то сумма количества проигрышей игрока A и количества побед игрока B не меньше 8. Докажите, что
игроков, одержавших в этом турнире ровно 4 победы, столько же, сколько игроков, одержавших ровно 5 побед. Напомним, что ничьих в теннисе не бывает.
Решение. Сопоставим каждому игроку количество очков, равное числу его побед. Как легко
видеть, суммарное количество очков равно количеству игр, то есть 45. Покажем, что каждый
игрок выиграл хотя не более 5 партий.
Пусть игрок А выиграл у игрока B, и количество набранных ими очков равно a и b соответственно. По условию 9–a+b  8  a –b ≤ 1 (*).
Если какой-то игрок выиграл хотя бы 7 партий, то проигравшие ему игроки в силу (*) выиграли хотя бы по 6. Но тогда суммарное количество набранных ими очков не меньше
67+7 = 49, что больше 45. Если какой-то из игроков A набрал 6 очков, то проигравшие ему
набрали хотя бы по 5. Рассмотрим игрока, который выиграл у A. Если он выиграл у всех игроков, у которых выиграл А, то он набрал хотя бы 7 очков, что невозможно. Если он проиграл кому-то из них, то в силу (*) количество набранных им очков не может быть меньше 4.
Отсюда получается, что общее количество набранных очков не меньше, чем 6+56+34 = 48, а
это больше 45.
Заметим, что игроки, набравшие 5 очков, найтись должны, кроме того их должно быть
не меньше 5: иначе всего игроки набрали меньше 45 очков. Покажем, что не найдется игрока, набравшего меньше 4 очков. Пусть такой игрок нашелся. Тогда в силу (*) он не мог проиграть игроку, набравшему 5 очков. Но выиграть у всех игроков, набравших 5 очков, он тоже
не мог, так как их не меньше пяти — противоречие. Получаем, что все игроки одержали либо
4, либо 5 побед. Простой подсчет суммы очков, показывает что таких игроков поровну.
Задача 6. Пусть e(k) — количество четных натуральных делителей натурального
числа k, а o(k) — количество его нечетных натуральных делителей. Докажите, что
e(1)+e(2)+…+e(n) отличается от o(1)+o(2)+…+o(n) меньше, чем на n.
Решение. Выпишем на доску все делители чисел 1, 2, …, n. Пусть выписано s1 единиц,
s2 двоек, …, sn чисел n. Очевидно, что n = s1  s2  …  sn. Заметим, что |(o(1)+o(2)+…+o(n))–
(e(1)+e(2)+…+e(n))| равняется модулю знакопеременной суммы s1–s2+s3–…+(–1)n+1sn. Эта
сумма не больше s1 = n, поскольку группируя s3 и –s2, s5 и –s4 и т.д., получаем неположительные слагаемые (а если n четно, то непарное слагаемое –sn отрицательно). С другой стороны,
группируя s1 и –s2, s3 и –s4 и т.д., получаем неотрицательные слагаемые (а если n нечетно, то
непарное слагаемое sn положительно), так что указанная сумма не меньше 0, что и завершает
доказательство.
20
Задача 7. Вершины A и E квадрата ABCD и прямоугольника EBFD лежат по одну
сторону от общей диагонали BD. Точка G на прямой BE такова, что AG перпендикулярно
AE. Докажите, что BG = ED.
Решение. Все вершины квадрата ABCD и прямоугольника EBFD лежат на окружности
радиуса BD/2 с центром O в середине отрезка BD. Поскольку вписанный в нее угол FAE опирается на диаметр, он прямой, то есть точка G лежит на пересечении прямых BE и AF: внутри окружности, если точка E лежит на дуге AD, и вне ее в противном случае.
Рассмотрим первый случай. Вписанный угол EBF — прямой, поскольку опирается на
диаметр EF. Вписанный угол BFA равен 45 градусам, поскольку опирается на дугу AB, равную 90 градусам. Поэтому прямоугольный треугольник GBF — равнобедренный, и нам достаточно доказать, что BF = ED. Но это очевидно, поскольку треугольники OBF и ODE равны по первому признаку.
Во втором случае вписанный угол BEA равен 135 градусам. Поэтому треугольник
GAE — прямоугольный равнобедренный, откуда AE = AG. Кроме того, AB = AD и каждый из
углов GAB и DAE равен 90 градусам плюс половина дуги EB. Поэтому треугольники GAB и
EAD равны по первому признаку, откуда BG = ED.
Задача 8. В выпуклом n-угольнике провели 2kn+1 диагональ. Докажите, что можно
выбрать 2k+1 диагоналей, составляющих несамопересекающуюся ломаную.
Решение. Индукция по k. При k = 0 утверждение задачи очевидно. Пусть верно при
k = m. Докажем для k = m+1. Для каждой вершины удалим две выходящие из нее диагонали,
отсекающие наименьшее количество вершин по и против часовой стрелки соответственно
(если из вершины выходит меньше двух диагоналей — удалим все). После этого у нас останется не меньше, чем 2(k–1)n+1 диагоналей, и по предположению индукции найдется (k–1)звенная несамопересекающаяся ломаная из этих диагоналей. Пусть AB — ее первое звено.
Возьмем стертую диагональ AC, которая идет в том же направлении, что и AB. Добавляя к
найденной ломаной начальное звено CA, получаем искомую k-звенную несамопересекающуюся ломаную.
21
Итоги командной олимпиады
Команда
Старшая группа
1 2 3 4
5
6
7
8
7
7
7
5
5
0
7
7
3
7
7
7
7
0
7
7
7
7
6
0
6
0
7
7
7
7
7
0
5
0
7
7
7
7
6
0
5
0
7
7
5
7
6
0
5
0
7
7
7
7
2
0
5
0
7
7
7
7
0
0
7
0
7
7
7
7
0
0
6
0
7
7
7
7
0
0
5
0
7
7
7
5
0
0
7
0
7
3
7
7
2
0
7
0
7
3
7
7
0
0
5
0
6
7
7
3
0
0
5
0
7
6
7
0
0
0
5
0
7
7
0
0
2
0
5
0
7
3
3
0
0
0
7
0
Ижевск-8-2
0
7
3
7
0
0
0
0
Л2Ш-8-1
7
2
4
0
0
0
0
0
Киров-8-2
4
0
2
0
0
0
7
0
Л2Ш-8-2
5
2
6
0
0
0
0
0
Екатеринбург гимназия 9-8-2
7
5
0
0
0
0
0
0
Озерск 8
Барнаул
0
3
2
0
0
0
5
0
0
2
2
0
0
0
0
0
При равенстве очков рейтинговые места команд определялись жребием.
22
вторая
7
первая
7
Место
1-2
1-2
3-4
3-4
5
6
7-8
7-8
9
10-12
10-12
10-12
13
14
15
16
17
18
19-21
19-21
19-21
22
23
24
высшая
Ижевск-8-1
Санкт-Петербург-239-8-1
Киров-8-1
Пермь9-8-1
Фрактал-ЮМШ-8
СПБ-239-7-8-1
Казань 8
Курган ЦДМО-8
Пермь9-8-2
1514-8 Москва
ДваждыДва-8
Новосибирск - 1
Омск-8-1
Омск-8-2
Батарея
Казань ТТЛ
Екатеринбург гимназия 9 8

45
45
40
40
39
37
35
35
34
33
33
33
29
28
25
21
20
17
13
13
13
12
10
4
6
7
7
0
7
0
7
ДваждыДва-7-1
7
5
7
7
6
0
0
6
Ульяновск-7
7
7
7
7
4
2
0
4
Обычные люди
7
5
7
7
7
0
0
3
Санкт-Петербург239 7-1
7
2
7
7
6
6
0
0
Фрактал-7
Москва 1514-7
7
7
7
4
0
7
7
7
7
6
1
0
0
0
4
0
Санкт-Петербург ЮМШ 7
7
5
7
0
6
3
0
0
Казань 7-1
6
7
0
7
5
0
0
0
Л2Ш 7-1
7
5
7
0
4
0
0
2
Новосибирск-2
7
7
0
7
3
0
0
0
Ижевск-7
7
7
0
7
0
0
0
0
КЛАСнО
0
5
7
0
5
1
0
0
Киров 7-2
6
4
6
0
0
0
0
0
Набережные Челны-7
3
4
7
0
0
2
0
0
Пермь 9 7-2
0
7
0
0
5
0
0
4
Л2Ш 7-2
7
5
0
0
2
0
0
0
Екатеринбург 9-7
7
5
0
0
0
0
0
0
Нижнекамск-7
7
5
0
0
0
0
0
0
ДваждыДва 7-2
7
2
0
0
2
0
0
0
Нижний Тагил-7
1
7
0
0
3
0
0
0
Санкт-Петербург30-7
7
4
0
0
0
0
0
0
Пермь 9 7-1
3
0
0
2
0
3
0
0
Омск-7
5
1
1
0
0
0
0
0
Казань 7-2
3
3
0
0
0
0
0
0
Красноярск-7
0
0
0
0
0
0
0
0
23
1
2-3
2-3
4
5
6
7
8
25
25
24
21
18
16
16
16
9-10
9-10
11
12
13
14-16
14-16
14-16
14
12
12
11
11
11
8
7
6
0
17
18-19
18-19
20-22
20-22
20-22
23
24
25
26
вторая
7
первая
Киров 7-1

41
38
38
36
35
33
31
28
высшая
Младшая группа
1 2 3 4 5 6 7 8
3
6
5
4
4
5
5
5
3
2
0
0
5
0
0
0
0
0
0
4
7
7
7
7
7
2
0
7
5
3
7
0
0
0
0
0
0
0
5
7
7
7
7
7
7
7
0
0
0
0
0
7
7
0
0
0
0
6
3
3
3
2
0
4
0
0
0
0
0
0
0
0
0
0
0
0
7
0
0
0
0
0
0
0
0
0
0
0
0
0
0
0
0
0
0
 Место
37
1
36
2
35
3
34
4
32
5
27
6
26
7
22
8
19
9
17
10
15
11
14 12-13
14 12-13
12
14
7 15-16
7 15-16
4 17-18
4 17-18
При равенстве очков рейтинговые места команд определялись жребием.
24
первая
2
7
7
7
7
6
2
7
5
5
7
5
2
7
5
7
7
4
4
высшая
Группа "Старт"
1
7
Казань-6
7
Пермь9-6
7
ДваждыДва-6-1
7
Ижевск-6-1
7
Санкт-Петербург-239-6
7
Ижевск 6-2
7
Курган ЦДМО-6
7
1514-6 Москва
7
ДваждыДва-6-2
7
Киров-5
3
Набережные Челны 26 6-1
7
Фрактал-6
0
Киров-6
0
Казань 5
0
Татарстан 6
0
Набережные Челны 26 6-2
0
Озерск 6
0
ЧелКор
МАТЕМАТИЧЕСКАЯ ОЛИМПИАДА 15.02.2011
ЗАДАНИЯ ДЛЯ 6 КЛАССА
1. У четырехзначного числа X взяли две последние цифры, поменяли их местами и
полученное двузначное число прибавили к X. Получилось число 2011. Чему могло быть равно число X? Двузначное число не может начинаться с нуля.
2. На доску выписали 20 различных натуральных чисел. Оказалось, что среди них 11
чисел делятся на 13, а 13 чисел делятся на 11. Докажите, что среди них есть число, большее
500.
3. Семь футбольных команд провели турнир в один круг, причем каждая команда
ровно два матча выиграла и ровно два проиграла. Могло ли такое случиться, что никакие 3
команды не выиграли друг у друга по циклу (то есть нет трёх таких команд А, Б и В, что А
выиграла у Б, Б — у В, а В — у А)?
4. На окружности расположено 100 точек. Каждая точка окрашена в один из двух цветов. Оказалось, что количество прямых, соединяющих разноцветные точки, равно количеству прямых, соединяющих одноцветные точки. Сколько всего точек каждого из цветов может быть?
5. Каким наименьшим количеством коней можно побить все черные поля доски 99,
угловые поля которой — чёрные?
МАТЕМАТИЧЕСКАЯ ОЛИМПИАДА 15.02.2011
ЗАДАНИЯ ДЛЯ 7 КЛАССА
1. Маша первый вторник месяца провела в Кирове, а первый вторник после первого понедельника этого же месяца она была в Казани. В первую среду следующего месяца Маша была в Вологде, а первую среду после первого вторника того же месяца она провела в Костроме.
Где Маша в том году была 8 марта?
2. Дан клетчатый квадрат 1111. Его клетки покрашены в шахматном порядке в черный и белый цвета, причем углы квадрата черные. Можно ли в этом квадрате разместить несколько квадратиков 33 с вырезанной центральной клеткой так, чтобы каждая белая клетка
исходного квадрата была покрыта ровно один раз?
3. Даны 20 различных натуральных чисел. Девять из них делятся на 19, восемь из них
делятся на 18, семь из них делятся на 17. Докажите, что одно из них больше 600.
4. Квадраты ABCD и DEFG таковы, что точка E лежит на отрезке CD, а точка D — на
отрезке AG. Прямые AC и EG пересекаются в точке H. Докажите, что точки B, F, H лежат на
одной прямой.
5. Найдите все пары простых чисел p и q такие, что p6–q2 = (p–q)2/2.
МАТЕМАТИЧЕСКАЯ ОЛИМПИАДА 15.02.2011
ЗАДАНИЯ ДЛЯ 8 КЛАССА
25
1. Сложили числа 9, 99, 999, …, 99…99 (2011 девяток). Сколько единиц в записи получившейся суммы?
2. Дан клетчатый прямоугольник 20092013. Его клетки покрашены в шахматном порядке в черный и белый цвета, причем углы квадрата черные. Можно ли в этом квадрате
разместить несколько квадратиков 33 с вырезанной центральной клеткой так, чтобы каждая
белая клетка исходного квадрата была покрыта ровно один раз?
3. Точки D и E — основания перпендикуляров, опущенных из вершины A треугольника ABC на биссектрисы углов B и C. Докажите, что прямые DE и BC параллельны.
4. Найдите все пары простых чисел p и q такие, что p6–q2 = (p–q)2/2.
5. Докажите, что при любых положительных a, b и c выполнено неравенство
2
a b c
 1 1 1



(
a

b

c
)


   .
b
c
a


a b c
Решения задач личной олимпиады 6 класса
Задача 1. У четырехзначного числа X взяли две последние цифры, поменяли их местами
и полученное двузначное число прибавили к X. Получилось число 2011. Чему могло быть равно
число X? Двузначное число не может начинаться с нуля.
Ответ: 2010. Решение. По условию 2011 = abcd  dc  ab00  cd  dc  ab00  11(c  d ) ,
откуда ab00  2000  11(c  d  1) . Поскольку c+d < 19, 11(c+d–1) делится на 100 только при
c+d = 1, то есть при с = 0, d = 1 или с = 1, d = 0. Второй случай невозможен, так как двузначное
число не может начинаться с нуля, а первый дает ответ.
Задача 2. На доску выписали 20 различных натуральных чисел. Оказалось, что среди
них 11 чисел делятся на 13, а 13 чисел делятся на 11. Докажите, что среди них есть число,
большее 500.
Решение. Так как 11 и 13 — простые числа, числа, делящиеся на 11 и 13 одновременно,
делятся на 1113. Легко видеть, что среди выписанных есть по крайней мере четыре таких числа. Наибольшее из них не меньше, чем 41113 > 500.
Задача 3. Семь футбольных команд провели турнир в один круг, причем каждая команда ровно два матча выиграла и ровно два проиграла. Могло ли такое случиться, что никакие 3 команды не выиграли друг у друга по циклу (то есть нет трёх таких команд А, Б и
В, что А выиграла у Б, Б — у В, а В — у А)?
Ответ. Могло. Решение. Изобразим команды семью точками на окружности и будем
считать, что каждая команда выиграла у двух следующих за ней по часовой стрелке. Тогда если команда А выиграла у Б, а Б — у В, то между В и А по часовой стрелке не меньше двух команд, и В у А выиграть не могла.
Задача 4. На окружности расположено 100 точек. Каждая точка окрашена в один
из двух цветов. Оказалось, что количество прямых, соединяющих разноцветные точки, равно количеству прямых, соединяющих одноцветные точки. Сколько всего точек каждого из
цветов может быть?
Ответ. 45 одного цвета и 55 другого. Решение. Пусть у нас есть a точек одного цвета и
100–a другого. Всего прямых, соединяющих различные точки, у нас 10099/2 = 5099. Поэтому
прямых, соединяющих разноцветные точки, должно быть a(100–a) = 2599 (и это равносильно
утверждению задачи). Так как 25 делится на простое число 5, одно из чисел a и 100–a должно
делиться на 5. Но тогда, поскольку 100 делится на 5, на 5 должно делится и другое из этих чисел. Так как 99 делится на простое число 3, одно из чисел a и 100–a должно делиться на 3. Поскольку 100 не делится на 3, другое из этих чисел на 3 не делится, поэтому одно из чисел a и
26
100–a должно делиться на 9, а, значит, и на 45. Поскольку оба числа a и 100–a нечетны и
меньше 100, это число может равняться только 45. Тогда другое равно 55.
Задача 5. Каким наименьшим количеством коней можно побить все черные поля доски 99, угловые поля которой — чёрные?
Ответ: 10. Решение. Заметим, что пять полей, помеченных
единицами, можно побить, самое меньшее, тремя конями. Еще
минимум три коня понадобятся, чтобы побить поля, отмеченные
двойками, и по два коня — на поля, отмеченные тройками и четверками (легко видеть, что один конь бить поля, отмеченные разными цифрами, не может). Поэтому коней потребуется не меньше 10. Десятью конями побить все черные клетки можно, если,
например, поставить их на поля, отмеченные звездочками на рисунке справа. (и этот пример — далеко не единственный).
1
1
1 *
1
1 *
3
3
*
*
*
*
*
2
2
* 2
*
4 *
Решения задач личной олимпиады 7 класса
Задача 1. Маша первый вторник месяца провела в Кирове, а первый вторник после
первого понедельника этого же месяца она была в Казани. В первую среду следующего месяца Маша была в Вологде, а первую среду после первого вторника того же месяца она провела в Костроме. Где Маша в том году была 8 марта?
Ответ. В Костроме. Решение. Поскольку вторник, который Маша провела в Казани, не
был первым вторником этого месяца, а понедельник перед ним был первым понедельником
этого месяца, этот месяц начинался со вторника. По аналогичной причине следующий месяц
начинался со среды. Но такое возможно только если в предыдущем месяце было 29 дней. Значит, предыдущий месяц — февраль, 1 марта Маша была в Вологде, а 8 марта — в Костроме.
Задача 2. Дан клетчатый квадрат 1111. Его клетки покрашены в шахматном порядке в черный и белый цвета, причем углы квадрата черные. Можно ли в этом квадрате разместить несколько квадратиков 33 с вырезанной центральной клеткой так, чтобы каждая
белая клетка исходного квадрата была покрыта ровно один раз?
Ответ. Можно. Решение. Назовем квадратик 33 с вырезанной центральной клеткой
рамкой. Квадрат 44 покрывается с соблюдением условия задачи двумя рамками с центрами
на черной диагонали. Покроем два противоположных угла и центр квадрата 1111 тремя рамками, а оставшуюся часть квадрата — восемью квадратами 44, прилегающими по два к сторонам квадрата, каждый из которых покрыт двумя рамками. Легко проверить, что любые два
квадрата 44 либо не пересекаются, либо пересекают по одной черной клетке.
Задача 3. Даны 20 различных натуральных чисел. Девять из них делятся на 19, восемь
из них делятся на 18, семь из них делятся на 17. Докажите, что одно из них больше 600.
Решение. Заметим, что любые два из чисел 17, 18 и 19 взаимно просты. Поэтому если
есть число, которое делится на два или три из них, то оно делится и на произведение этих двух
или трёх. Поставим около каждого данного числа, делящегося на 17, красный крестик, делящегося на 18 — синий крестик, делящегося на 19 — зеленый крестик. Всего будет выставлено
24 крестика. Если есть число с тремя крестиками, то оно делится и на 171819 > 600, и все доказано. Если же такого числа нет, то есть четыре числа с двумя крестиками. Поскольку из трех
цветов можно составить только три различные пары, среди этих четырех чисел найдутся два, у
которых цвета крестиков одинаковы. Тогда наибольшее из этих двух чисел не меньше, чем
21718, что больше 600.
Задача 4. Квадраты ABCD и DEFG таковы, что точка E лежит на отрезке CD, а
точка D — на отрезке AG. Прямые AC и EG пересекаются в точке H. Докажите, что точки
B, F, H лежат на одной прямой.
27
Решение. Пусть стороны квадратов ABCD и DEFG равны a и b соответственно. Как
легко видеть, CHE — равнобедренный прямоугольный треугольник с основанием CE. Опустим из точки H перпендикуляры HK и HL на BC и CD соответственно. Очевидно,
KC = CL = (a–b)/2. Пусть M — точка пересечения прямых BC и GF. Тогда KM = KC+CM = (a–
b)/2+b = (a+b)/2 = a–(a–b)/2 = BK. Пусть KN — средняя линия треугольника BMF. Она лежит
на луче KH и равна MF/2 = (a–b)/2 = KH. Значит, точки N и H совпадают, что и требовалось
доказать.
Задача 5. Найдите все пары простых чисел p и q такие, что p6–q2 = (p–q)2/2.
Ответ. p = 3, q = 23. Решение. Раскрыв скобки в правой части и умножив обе части на 2,
получим
2p6–2q2 = p2–2pq+q2, откуда 3q2 = 2p6–p2+2pq (*). Получается, что 3q2 делится на p. Поскольку
p и q просты и, очевидно, различны, p = 3. Подставляя p = 3 в (*) и деля обе части на 3, после
преобразований получаем q2–2q = 483, то есть q(q–2) = 483. Перебором нетрудно убедиться,
что число 483 можно представить в виде произведения двух сомножителей, отличающихся на
2, единственным способом 483 = 2321, откуда и получается ответ.
Решения задач личной олимпиады 8 класса
Задача 1. Сложили числа 9, 99, 999, …, 99…99 (2011 девяток). Сколько единиц в записи
получившейся суммы?
Ответ: 2007. Решение. Сумма равна (10–1)+(100–1)+…+(102011–1) = 11…110–
2011 = 11…1109099, где в числе 11…110 — 2011 единиц, и, следовательно, в числе
11…1109099 — 2007 единиц.
Задача 2. Дан клетчатый квадрат 20092013. Его клетки покрашены в шахматном
порядке в черный и белый цвета, причем углы квадрата черные. Можно ли в этом квадрате
разместить несколько квадратиков 33 с вырезанной центральной клеткой так, чтобы каждая белая клетка исходного квадрата была покрыта ровно один раз?
Ответ: Нельзя. Решение. Каждый квадратик с вырезанной центральной клеткой покрывает 4 белых клетки. Но в прямоугольнике 20092013 с черными углами (20092013–1)/2
белых клеток, а это число, как легко проверить, не делится на 4.
Задача 3. Точки D и E — основания перпендикуляров, опущенных из вершины A треугольника ABC на биссектрисы углов B и C. Докажите, что прямые DE и BC параллельны.
Решение. Пусть A1 — точка, симметричная A относительно биссектрисы угла B. Заметим, что точка A1 лежит на прямой BC, так как прямые BA и BC симметричны относительно
биссектрисы угла между ними. Аналогично точка A2, симметричная A относительно биссектрисы угла C, лежит на прямой BC. В треугольнике AA1A2 точки D и E — середины сторон AA1
и AA2 соответственно. Поэтому его средняя линия DE параллельна прямой A1A2, которая совпадает с прямой BC.
Задача 4. Найдите все пары простых чисел p и q такие, что p6–q2 = (p–q)2/2.
Ответ. p = 3, q = 23. Решение. Раскрыв скобки в правой части и умножив обе части на 2,
получим
2p6–2q2 = p2–2pq+q2, откуда 3q2 = 2p6–p2+2pq (*). Получается, что 3q2 делится на p. Поскольку
p и q просты и, очевидно, различны, p = 3. Подставляя p = 3 в (*) и деля обе части на 3, после
преобразований получаем q2–2q–483 = 0. Полученное квадратное уравнение имеет корни –21 и
23, из которых простым числом является второй.
Задача 5. Докажите, что при любых положительных a, b и c выполнено неравенство
2
a b c
1 1 1
     (a  b  c)     .
b c a
a b c
28
Решение. Раскрыв скобки и собрав все члены, содержащие переменные, в левой части
неравенства, его нетрудно затем привести к виду
2
2
2
a  b   c  a b c a c b
  1    1    1            6 .
b  c  a  b c a c b a
Осталось заметить, что
a b c
abc
a c b
   33
 3 и, аналогично,    3 .
b c a
bca
c b a
29
ПОБЕДИТЕЛИ И ПРИЗЕРЫ ЛИЧНОЙ ОЛИМПИАДЫ
Фамилия Имя
Зайцев Егор
Ярцев Дмитрий
Шувалова Мира
Фролов Иван
Филиппова Мария
Овечкин Григорий
Кучеров Илья
Лупуляк Ольга
Соколов Игнат
Камзолов Владимир
Доронин Георгий
Петров Тимур
Лычагина Елена
Жуков Матвей
Беликов Дмитрий
Салимов Руслан
Кутиков Иван
Орешников Даниил
Лялина Альбина
Халиков Тимур
Фролов Дмитрий
Сапожников Артём
Макарочкин Илья
Киракосян Александр
Зосимова Жанна
Зайцев Тимофей
Волковыский Станислав
Валиева Руфина
Горячев Владимир
Цаплин Александр
Блохина Полина
Кузнецов Александр
Ходунов Павел
Максимов Антон
Будин Николай
Мрыхин Михаил
Евтушевский Всеволод
Захаров Павел
Кубарев Алексей
Попов Даниил
Спиченко Александр
Кокин Владислав
Лосев Илья
Назаров Андрей
Нестеров Никита
Ткаченко Дмитрий
Шатский Владимир
Воронин Денис
Зимин Александр
класс
6
6
6
6
6
6
6
6
6
6
6
6
6
6
6
6
6
6
6
6
6
6
6
6
6
6
6
6
6
6
7
7
7
7
7
7
7
7
7
7
7
7
7
7
7
7
7
7
7
город
1
7
7
6
7
7
7
6
6
7
6
7
3
7
7
3
6
7
7
1
7
2
6
2
2
7
2
1
7
7
7
7
7
7
7
7
7
7
5
7
7
7
7
7
7
3
7
7
7
7
Ижевск
Челябинск
Санкт-Петербург
Москва
Ижевск
Ижевск
Ижевск
Санкт-Петербург
Курган
Казань
Санкт-Петербург
Москва
Курган
Санкт-Петербург
Новосибирск
Москва
Челябинск
Санкт-Петербург
Ижевск
Казань
Москва
Ижевск
Екатеринбург
Санкт-Петербург
Ижевск
Москва
Санкт-Петербург
Набережные Челны
Москва
Пермь
Санкт-Петербург
Санкт-Петербург
Санкт-Петербург
Челябинск
Киров
Ижевск
Санкт-Петербург
Ульяновск
Москва
Казань
Казань
Москва
Санкт-Петербург
Пермь
Ульяновск
Челябинск
Москва
Новосибирск
Ульяновск
30
2
7
6
6
7
7
6
7
7
7
6
7
7
6
7
6
6
3
5
7
7
6
6
6
6
5
7
7
6
7
6
7
7
7
7
7
7
7
0
7
7
7
7
7
7
7
7
7
7
0
3
7
7
7
7
7
7
7
7
7
7
7
7
0
0
0
0
7
0
7
0
7
0
7
7
0
6
7
0
0
0
7
7
7
7
7
7
7
7
4
5
4
7
4
2
7
4
3
3
2
4
7
7
7
3
0
3
0
2
0
0
0
0
6
2
7
3
0
4
1
1
0
0
0
0
0
0
0
0
1
0
7
7
7
7
7
7
7
7
7
6
7
3
0
7
7
6
7
6
7
5
7
2
0
2
2
0
3
0
0
2
0
2
0
2
2
2
0
0
0
0
0
3
0
0
3
0
0
2
0
2
7
7
7
6
3
1
0
6
0
0
0
0
6
1
0
0
0
0
7
∑
35
29
26
26
23
23
23
22
21
21
21
19
19
18
18
17
17
16
16
15
15
15
15
15
15
15
15
15
15
15
35
35
35
34
31
29
28
25
25
25
25
24
24
24
24
24
24
23
23
награда
I
II
II
II
III
III
III
III
III
III
III
III
III
по
по
по
по
по
по
по
по
по
по
по
по
по
по
по
по
по
I
I
I
I
II
II
II
II
II
II
II
III
III
III
III
III
III
III
III
Фамилия Имя
Рубанов Илья
Морозов Никита
Смирнов Виталий
Бочков Иван
Бучкин Александр
Монаков Григорий
Рогозина Анна
Савон Юлия
Хальзов Ян
Шишкин Михаил
Мишняков Виктор
Нуждов Глеб
Севастюк Дмитрий
Богданов Илья
Васильчишин Сергей
Кириллов Андрей
Огарок Петр
Петров Степан
Юшко Илья
Артамонов Николай
Горский Дмитрий
Корчагин Максим
Лазарев Алексей
Лужецкий Илья
Мартинсон Михаил
Овчинников Максим
Рогозина Татьяна
Ягламунов Владислав
Золотов Борис
Воробьева Екатерина
Гашигуллин Руслан
Зайцев Дмитрий
Кузнецова Елизавета
Мельников Артем
Никитина Ксения
Прозоров Лев
Сергунин Андрей
Соколов Андрей
Шестакова Анна
Багиров Фарид
Кайсин Илья
Клюев Даниил
Боровков Данила
Волостнов Алексей
Калитова Ольга
Нигметзянова Диана
Сотников Сергей
Холмогоров Ефим
Шарафутдинов Азат
Смердов Антон
класс
7
7
7
7
7
7
7
7
7
7
7
7
7
7
7
7
7
7
7
7
7
7
7
7
7
7
7
7
7
7
7
7
7
7
7
7
7
7
7
7
7
8
8
8
8
8
8
8
8
8
город
Киров
Екатеринбург
Москва
Ижевск
Ижевск
Санкт-Петербург
Киров
Санкт-Петербург
Москва
Киров
Москва
Киров
Санкт-Петербург
Москва
Киров
Москва
Москва
Санкт-Петербург
Красноярск
Санкт-Петербург
Москва
Казань
Москва
Москва
Москва
Набережные Челны
Киров
Челябинск
Санкт-Петербург
Москва
Казань
Москва
Москва
Нижний Тагил
Курган
Киров
Новосибирск
Москва
Ижевск
Санкт-Петербург
Киров
Санкт-Петербург
Санкт-Петербург
Казань
Ижевск
Казань
Санкт-Петербург
Ижевск
Киров
Киров
1
2
3
7
7
7
7
7
7
7
7
7
6
0
5
7
0
7
7
7
7
7
7
7
1
7
7
7
7
3
7
7
2
7
3
7
5
7
7
7
7
0
4
7
7
7
7
7
7
7
7
31
2
7
7
7
0
7
7
7
0
7
0
7
7
7
7
7
7
7
7
7
0
7
7
7
7
7
7
0
0
7
7
0
7
7
7
0
7
0
7
7
0
7
7
7
0
7
7
7
7
7
7
3
7
5
3
7
4
7
7
7
7
7
0
7
7
7
5
5
4
5
5
4
4
3
4
4
4
4
7
5
3
3
3
7
3
7
5
3
3
3
3
7
3
7
7
7
7
7
7
7
7
7
4
7
7
5
0
3
0
0
7
0
7
6
0
6
0
0
7
0
0
0
7
0
0
0
6
0
0
4
0
5
0
7
0
0
0
5
2
7
0
0
0
7
5
7
7
0
0
6
7
0
6
5
0
0
0
7
0
0
0
0
0
0
0
0
0
0
0
0
1
0
0
0
0
1
0
0
0
0
0
6
0
0
0
1
0
0
0
0
0
0
0
3
0
7
0
7
7
7
1
0
7
0
∑
23
22
22
21
21
21
21
21
21
21
20
20
20
19
19
19
19
19
19
18
18
18
18
18
18
18
18
18
18
17
17
17
17
17
17
17
17
17
17
17
17
30
28
28
28
28
28
28
28
27
награда
III
III
III
по
по
по
по
по
по
по
по
по
по
по
по
по
по
по
по
по
по
по
по
по
по
по
по
по
по
по
по
по
по
по
по
по
по
по
по
по
по
I
II
II
II
II
II
II
II
II
Фамилия Имя
Чернега Никита
Яровиков Юрий
Симарова Екатерина
Семенов Константин
Лебедева Дарья
Амирханов Николай
Глазырин Евгений
Журбенко Дмитрий
Кочкин Иван
Прокошев Никита
Сергеев Александр
Цаплин Сергей
Буренёв Иван
Подгузов Никита
Чистов Иван
Шишкин Евгений
Якутов Дмитрий
Новиков Александр
Фирсов Филипп
Бугакова Надежда
Агеева Мария
Брауде-Золотарев Илья
Деревянко Никита
Каранович Анри
Киселев Сергей
Нестеров Роман
Пластинин Виталий
Рембовская Александрина
Толмачев Дмитрий
Цейтина Елена
Плетнев Никита
класс
8
8
8
8
8
8
8
8
8
8
8
8
8
8
8
8
8
8
8
8
8
8
8
8
8
8
8
8
8
8
8
город
Казань
Казань
Санкт-Петербург
Ижевск
Санкт-Петербург
Санкт-Петербург
Курган
Москва
Киров
Пермь
Москва
Пермь
Санкт-Петербург
Санкт-Петербург
Казань
Пермь
Пермь
Новосибирск
Москва
Санкт-Петербург
Ижевск
Москва
Омск
Москва
Омск
Екатеринбург
Санкт-Петербург
Москва
Екатеринбург
Санкт-Петербург
Омск
1
7
7
4
4
7
0
7
7
7
7
7
7
7
7
7
7
7
4
7
4
0
7
7
7
7
7
7
7
7
7
7
32
2
7
0
7
7
7
7
1
7
7
7
7
7
7
1
7
7
0
7
0
1
7
7
7
0
7
7
7
7
7
7
0
3
7
7
7
7
3
7
7
0
7
7
7
0
0
5
5
0
7
7
0
7
7
0
0
7
0
0
0
0
0
0
0
4
5
6
7
0
7
7
6
7
0
0
0
7
6
6
0
5
5
0
4
5
0
0
0
0
0
0
0
0
0
0
6
5
1
7
2
7
0
0
0
0
0
0
0
0
0
0
0
0
0
0
7
0
0
0
0
0
0
0
0
0
0
0
0
∑
27
27
27
25
24
21
21
21
21
21
21
21
20
19
19
19
19
18
18
17
14
14
14
14
14
14
14
14
14
14
13
награда
II
II
II
II
II
III
III
III
III
III
III
III
III
III
III
III
III
III
III
III
по
по
по
по
по
по
по
по
по
по
по
XXXVII УРАЛЬСКИЙ ТУРНИР ЮНЫХ МАТЕМАТИКОВ. КИРОВ, 14-20.02.2011
ПРАВИЛА МАТЕМАТИЧЕСКОГО БОЯ
Общие положения. Математический бой – это соревнование двух команд в решении
математических задач. Он состоит из двух частей. Сначала команды получают условия задач
и определенное время на их решение. При решении задач команда может использовать любую литературу, но не имеет права общаться по поводу решения задач ни с кем, кроме жюри.
По истечении этого времени начинается собственно бой, когда команды в соответствии с
правилами рассказывают друг другу решения задач. Если одна команда рассказывает решение, то другая оппонирует его, т.е. ищет в нем ошибки (недостатки), и, если решения нет, то,
возможно, приводит свое. При этом выступления оппонента и докладчика оцениваются жюри в баллах (за решение и за оппонирование). Если команды, обсудив предложенное решение, все-таки до конца задачу не решили или не обнаружили допущенные ошибки, то часть
баллов (или даже все баллы) может забрать себе жюри боя. Если по окончании боя результаты команд отличаются не более чем на 3 балла, то считается, что бой закончился вничью. В
противном случае побеждает команда, которая по окончании боя набирает больше баллов.
Если же по условиям боя он не может закончиться вничью, то жюри до боя объявляет это
командам и оглашает процедуру определения победителя.
Капитаны команд имеют право попросить жюри о предоставлении перерыва в ходе
боя на 5–10 минут (примерно через каждые полтора часа). Перерыв может предоставляться
только между обсуждением двух различных задач (между раундами). При этом команда, которая должна сделать вызов, делает его в письменной форме (без оглашения) непосредственно перед началом перерыва и сдает жюри, которое оглашает этот вызов сразу после окончания перерыва.
Вызовы. Бой состоит из нескольких раундов. В начале каждого раунда (если не происходит отказ от вызова – см. ниже пункт “Окончание боя”) одна из команд вызывает другую на одну из задач, решения которых еще не рассказывались (например: “Мы вызываем
команду соперников на задачу номер 8”). После этого вызванная команда сообщает, принимает ли она вызов, т.е. согласна ли рассказывать решение задачи, на которую была вызвана
(ответ можно обдумывать, но не более 1 минуты). Если да, то она выставляет докладчика,
который должен рассказать решение, а вызвавшая команда выставляет оппонента, обязанность которого – искать в решении ошибки. Если нет, то докладчика обязана выставить команда, которая вызывала, а отказавшаяся отвечать команда выставляет оппонента. Команда,
желающая сохранить выходы к доске, может отказаться выставлять оппонента. Тогда она в
этом раунде не участвует (и изменить своего решения уже не может).
Ход раунда. Доклад. В начале раунда докладчик рассказывает решение. Доклад должен содержать ответы на все поставленные в задаче вопросы и доказательство правильности
и полноты полученных ответов. В частности, докладчик обязан доказать каждое сформулированное им промежуточное утверждение либо сослаться на него, как на общеизвестное. Докладчик должен стремиться к ясности изложения, в частности, он обязан повторить по
просьбе оппонента или жюри любую часть своего доклада. Время на доклад ограничивается
15 минутами, после чего жюри решает, разрешать ли докладчику рассказывать дальше.
Докладчик может иметь при себе записи и заглядывать в них, но жюри имеет право
запретить ему ими пользоваться, если сочтет, что докладчик читает решение по бумажке. В
докладе нельзя ссылаться на вычисления, проведенные с помощью калькулятора или иной
вычислительной техники и не подтвержденные иным способом.
Докладчик имеет право :
– до начала выступления вынести на доску всю необходимую информацию
(чертежи, вычисления и т.п.);
– не отвечать на вопросы оппонента, заданные до начала обсуждения;
33
– просить оппонента уточнить свой вопрос (в частности, докладчик может
предложить свою версию вопроса: “Правильно ли я понимаю, что вы спросили о
том-то и том-то?”);
– отказаться отвечать на вопрос, сказав, что: (а) он не имеет ответа на этот
вопрос; (б) он уже ответил на этот вопрос (объяснив, когда и как); (в) вопрос некорректен или выходит за рамки научной дискуссии по поставленной задаче. В случае
несогласия оппонента с основаниями (б) и (в) арбитром выступает жюри.
Докладчик не обязан:
– излагать способ получения ответа, если он может доказать его правильность и полноту ответа другим путем;
– сравнивать свой метод решения с другими возможными методами, в том
числе с точки зрения краткости, красоты и пригодности для решения других задач.
Докладчик обязан рассказывать решение в вежливой, корректной форме, критикуя
действия оппонента, не допускать критики его личности, обращаться к оппоненту только на
“Вы”.
Окончание доклада докладчик обязан чётко зафиксировать фразой «Доклад окончен».
В течение 10 секунд после этого докладчик или его команда имеют право отозвать эти слова
(оппонент в это время молчит). Если такого не случилось, слово передаётся оппоненту.
Оппонирование. Пока доклад не окончен, оппонент может задавать вопросы только с
согласия докладчика, но имеет право просить повторения части решения и разрешать докладчику не доказывать какие-либо очевидные с точки зрения оппонента факты. После
окончания доклада оппонент имеет право задавать вопросы докладчику. Если в течение минуты оппонент не задал ни одного вопроса, то считается, что у него нет вопросов. Если докладчик в течение минуты не начинает отвечать на вопрос, то считается, что у него нет ответа.
В качестве вопроса оппонент может :
– потребовать у докладчика повторить любую часть доклада;
– попросить уточнения любого из высказываний докладчика, в том числе:
(а) попросить дать определение любого термина (“Что Вы понимаете под ...”); (б)
переформулировать утверждение докладчика своими словами и попросить подтверждения (“Правильно ли я понимаю, что Вы утверждаете следующее: ...”);
– попросить докладчика доказать сформулированное тем неочевидное необщеизвестное утверждение (в спорных случаях вопрос об известности или очевидности решает жюри; во всяком случае, известными считаются факты, изучающиеся в общеобразовательной школе);
– после ответа на вопрос выразить удовлетворенность или мотивированную
неудовлетворенность ответом.
Если оппонент считает, что докладчик тянет время, придумывая решение у доски, или
что существенная часть доклада не является изложением решения обсуждаемой задачи, он
имеет право (но не ранее, чем через 10 минут после начала доклада) попросить докладчика
предъявить ответ (если таковой в задаче подразумевается) или план дальнейших рассуждений.
Оппонент обязан:
– формулировать свои вопросы в вежливой, корректной форме, обращаться
к докладчику только на “Вы”;
– критикуя доклад, не допускать критики докладчика;
– повторять и уточнять свои вопросы по просьбе докладчика или жюри.
34
По итогам доклада и ответов на вопросы оппонент имеет право дать свою оценку докладу и обсуждению в одной из следующих форм: (а) признать решение правильным; (б)
признать решение (ответ) в основном правильным, но имеющим недостатки и/или пробелы с
обязательным их указанием; (в) признать решение (ответ) неправильным с указанием ошибок в обоснованиях ключевых утверждений доклада или контрпримеров к ним (или ответу),
или указанием существенных пробелов в обоснованиях или плане решения. В ситуациях (а)
и (б) оппонент должен чётко зафиксировать своё мнение словами "С решением согласен"
или "Других возражений нет" соответственно. В течение 10 секунд после этого оппонент или
его команда имеют право отозвать эти слова (докладчик в это время молчит). Если такого не
случилось, оппонент и его команда в этом раунде больше не участвуют.
Если оппонент имеет контрпример, опровергающий решение докладчика в целом, и
этот контрпример сам является решением задачи (такое бывает, например, в случаях, когда
вопрос задачи звучит как “Можно ли …?”, “Верно ли, что …?” и т.п.), то оппонент имеет
право заявить: “Я с решением не согласен, у меня есть контрпример”, но сам контрпример
пока докладчику не предъявлять (хотя жюри имеет право потребовать от оппонента предъявления контрпримера в письменном виде, чтобы убедиться в корректности заявления оппонента). В этом случае, если докладчик не изменит своего решения в течение минуты или после взятого командой перерыва, оппонент получает право предъявить докладчику упомянутый контрпример, причем докладчик и его команда уже не имеют права менять решение или
ответ.
Аналогично, если решение требует перебора случаев, оппонент имеет право заявить
“Я с решением не согласен, рассмотрены не все случаи”, не указывая пока докладчику явно,
какой именно случай не рассмотрен. Дальнейшие действия докладчика, жюри и оппонента
такие же, как в ситуации с контрпримером.
Участие жюри в обсуждении. После окончания диалога докладчика и оппонента
жюри задает свои вопросы. При необходимости оно может вмешиваться и раньше. Жюри
вправе прекратить доклад или диалог, если он зашел в тупик, утратил связь с существом обсуждаемой задачи или не может быть закончен за разумное время.
Выступающие и команда. Докладчик и оппонент могут обращаться к своим капитанам с просьбой о замене или перерыве для консультации. Другое общение между командой и
докладчиком (оппонентом) допускается только во время полуминутного перерыва, который
любая команда может взять в любой момент (при этом соперники тоже могут пользоваться
этим временем). Каждая команда может взять в течение одного боя не более 6 полуминутных
перерывов (см. также ниже пункт “Число выходов к доске”). Команда имеет право полностью использовать полуминутный перерыв, взятый командой соперников, даже если та закончила его досрочно.
Перемена ролей. Некорректный вызов. Порядок вызовов. Если по ходу дискуссии
жюри установило, что оппонент доказал отсутствие у докладчика решения и ранее не произошел отказ от вызова, то возможны два варианта. Если вызов на этот раунд был принят, то
оппонент получает право (но не обязан) рассказать свое решение. Если оппонент взялся рассказывать свое решение, то происходит полная перемена ролей: бывший докладчик становится оппонентом и может зарабатывать баллы за оппонирование. Если же вызов на этот раунд не был принят, то говорят, что вызов был некорректным. В этом случае перемены ролей
не происходит, а команда, вызывавшая некорректно, должна снова вызывать соперника в
следующем раунде. Во всех остальных случаях в следующем раунде вызывает та команда,
которая была вызвана в текущем раунде.
Принятый вызов всегда считается корректным!
Если же оппонент не доказал, что у докладчика нет решения, но выявил в предложенном решении некоторые конкретные недостатки, то, если ранее не произошел отказ от вызо-
35
ва и вызов на этот раунд был принят, оппонент получает право (но не обязан) устранить все
(или некоторые) из этих недостатков (“залатать дыры”). Такое же право оппонент получает, если он доказал, что у докладчика нет решения, но отказался рассказывать собственное
решение. Если оппонент взялся “залатывать дыры”, то происходит частичная перемена ролей: оппонент обязан сформулировать предварительно, что именно он будет делать (например, разбирать такой-то не разобранный докладчиком случай, доказывать такое-то недоказанное докладчиком утверждение или что-либо еще), а бывший докладчик становится оппонентом и может зарабатывать баллы за оппонирование сформулированных утверждений.
При проверке корректности вызова частичная перемена ролей невозможна.
Обратной перемены ролей ни в каком случае не происходит!
Число выходов к доске. Каждый член команды имеет право выйти к доске в качестве
докладчика или оппонента не более двух раз за бой. Команда имеет право не более трех раз
за бой заменять докладчика или оппонента, причем каждый раз выход засчитывается как тому, кого заменили, так и тому, кто вышел на замену. Кроме того, при замене время, отведенное команде на перерывы, уменьшается на 1 минуту. Эту минуту можно использовать непосредственно перед заменой, а можно и не использовать — в последнем случае команда соперников тоже не имеет права пользоваться ею.
Отказ от вызова. Окончание боя. В любой момент боя команда, которая должна вызывать, может отказаться делать это (обычно это происходит, когда у команды больше нет
решенных задач, если она не хочет делать вызов, который может оказаться некорректным).
Тогда другая команда получает право (но не обязана) рассказывать решения оставшихся задач. При этом команда, отказавшаяся делать вызов, может выставлять оппонентов и получать баллы только за оппонирование, но рассказывать решения она уже не имеет права, даже
если они у нее и появятся (то есть после отказа от вызова не происходит ни полной, ни частичной перемены ролей).
Бой заканчивается, когда не остается необсужденных задач либо когда одна из команд
отказалась от вызова, а другая команда отказалась рассказывать решения оставшихся задач.
Первый вызов. Конкурс капитанов. Кто будет делать первый вызов, определяет
команда, победившая в конкурсе капитанов. Он проводится в начале боя. Капитанам предлагается задача. Капитан, первым сообщивший жюри о своем желании отвечать, получает такое право. Если он рассказывает правильное решение, то он победил, а если неправильное –
победил его соперник. При этом что понимается под “правильным решением”: просто верный ответ, ответ с объяснением или что-либо еще – жюри при необходимости уточняет перед началом конкурса капитанов.
На решение задачи конкурса капитанов жюри отводит определенное время. Если за
это время ни один из капитанов не высказал желания отвечать, жюри может заменить задачу
или выявить победителя жребием. Вместо задачи жюри может предложить капитанам сыграть в игру. В этом случае победителем считается тот, кто выигрывает игру. Возможны и
другие схемы проведения конкурса капитанов. Жюри боя заранее определяет способ проведения конкурса капитанов и сообщает о нем командам перед началом боя.
При желании на конкурс вместо капитана можно выставить любого другого члена команды.
Начисление баллов. Каждая задача оценивается в 12 баллов, которые по итогам раунда распределяются между докладчиком, оппонентом и жюри. Если докладчик, не опираясь
существенно на наводящие вопросы и иные соображения жюри и/или оппонента, рассказал
правильное и полное решение, все 12 баллов достаются ему. Если же в решении были выявлены "дыры" (пробелы), то жюри по окончании дискуссии определяет их стоимость. После
этого оппонент, как правило, сразу получает половину стоимости обнаруженных им дыр.
Если некоторые из этих "дыр" были в ходе дискуссии полностью или частично закрыты, со-
36
ответствующая часть остатка их общей стоимости распределяется между докладчиком и оппонентом пропорционально их вкладу в закрытие "дыр". При этом вкладом оппонента может
признаваться не только закрытие им дыры (в случае полной или частичной перемены ролей),
но и помощь докладчику в закрытии дыр путем высказанных по окончании доклада наводящих соображений. Все оставшиеся баллы жюри забирает себе.
Если ошибки или пробелы в докладе указаны самим докладчиком и не устранены его
командой, то оппонент получает за них баллы так, как если бы он сам нашел эти недостатки.
В частности, если, получив отказ от вызова, капитан вызывающей команды сразу признается,
что у его команды нет решения, то команда соперников получает 6 баллов за оппонирование
(которое в этом случае могло бы состоять из одной фразы: “У Вас нет решения”), а вызов
признается некорректным. Докладчик и оппонент в этом случае не назначаются и выходы к
доске не засчитываются.
Если не было полной перемены ролей, то оппонент не может получить больше 6 баллов. Оппонент, доказавший некорректность вызова, получает 6 баллов независимо от суммарной стоимости найденных им дыр.
Капитан. Во время боя только капитан может от имени команды обращаться к жюри
и соперникам: сообщать о вызове или отказе, просить перерыв и т.д. Он имеет право в любой
момент прекратить доклад или оппонирование представителя своей команды. Если капитан у
доски, он оставляет за себя заместителя, исполняющего в это время обязанности капитана.
Имена капитана и заместителя сообщаются жюри до начала решения задач.
Во время решения задач главная обязанность капитана — координировать действия
членов команды так, чтобы имеющимися силами решить как можно больше задач. Для этого
капитан распределяет между членами команды задачи для решения (с учетом их пожеланий),
следит, чтобы каждая задача кем-то решалась, организует проверку найденных решений. Капитан заранее выясняет, кто будет докладчиком или оппонентом по той или иной задаче и
определяет всю тактику команды на предстоящем бое.
Жюри. Жюри является верховным толкователем правил боя. В случаях, не предусмотренных правилами, оно принимает решение по своему усмотрению. Решения жюри являются обязательными для команд.
Во время решения командами задач всякое существенное разъяснение условий задач,
данное одной из команд, должно быть в кратчайшее время сообщено жюри всем остальным
командам.
Жюри может снять вопрос оппонента (например, если он не по существу), прекратить
доклад или оппонирование, если они затягиваются. Если жюри не может быстро разобраться
в решении, оно может с согласия обоих капитанов выделить своего представителя, который
продолжит обсуждение задачи совместно с докладчиком и оппонентом в другом помещении.
При этом бой продолжается по другим задачам, а очки по этой задаче начисляются позже.
Жюри ведет протокол боя. Если одна из команд не согласна с принятым жюри решением по задаче, она имеет право немедленно потребовать перерыв на несколько минут для
разбора ситуации с участием Старшего по лиге. После начала следующего раунда счет
предыдущего раунда, как правило, изменен быть не может. Единственное исключение составляет случай, когда Старший по лиге по результатам разбора ситуации принимает решение поставить вопрос о ней на заседании Методической комиссии (МК) турнира. В этом случае МК должна принять решение по этому вопросу в тот же день. Вопрос о корректности вызова на заседание МК вынесен быть не может и должен разрешаться Старшим лиги на месте.
Жюри следит за порядком. Оно может оштрафовать команду за шум, некорректное
поведение, общение со своим представителем, находящимся у доски.
37
Жюри обязано мотивировать свои решения, не вытекающие непосредственно из правил боя.
Зрители. За ходом боя могут с разрешения жюри наблюдать зрители: к ним относятся
все, кроме членов играющих команд, жюри боя и Методической комиссии турнира. Зрители
не имеют права общаться с командами, вслух комментировать ход боя или иным образом
мешать его проведению. Зрители, нарушающие эти правила, обязаны по решению жюри боя
покинуть помещение, где он проходит.
РЕГЛАМЕНТ ПОДГОТОВКИ К МАТЕМАТИЧЕСКОМУ БОЮ
1. При подготовке к бою членам играющих команд запрещается:
 пользоваться при решении задач помощью людей, не входящих в команду;
 без крайней необходимости общаться с людьми, не входящими в команду (за исключением членов Методической комиссии Кубка и уполномоченных ею лиц);
 пользоваться компьютерами, программируемыми калькуляторами, смартфонами и
иными подобными устройствами; выходить в Интернет, использовать средства программирования и литературу на электронных носителях.
2. При подготовке к бою членам играющих команд разрешается:
 пользоваться непрограммируемыми калькуляторами;
 пользоваться литературой на бумажных носителях.
При установленном нарушении п. 1 команда-нарушитель снимается с боя и ей засчитывается
поражение со счетом 0:60.
ПРАВИЛА БЛИЦ-БОЯ.
1. Блиц-бой проводится для определения мест команд в случае равенства баллов в групповом
турнире у двух или более команд, если (в случае двух команд) личная встреча между командами закончилась вничью, а также для определения победителя при ничейном исходе тех
финальных боев, которые по регламенту не могут закончиться вничью.
2. Командам выдается 8 задач на 25 минут.
3. Ответы сдаются в письменном виде.
4. За каждый верный ответ начисляется 3 очка, за каждый неверный снимается 1 очко.
5. Места команд определяются по сумме баллов. При равенстве сумм баллов места определяются по результатам в командной олимпиаде. Если и эти результаты равны, места определяется жребием.
38
МАТЕМАТИЧЕСКИЙ БОЙ №1. 16.02.2011
СТАРШАЯ ГРУППА, ВЫСШАЯ ЛИГА
1. Куб 333 состоит из 27 единичных кубиков. В центре одного из кубиков сидит кубогрызка, которая может перебираться по прямой в центр соседнего кубика (то есть имеющего с ее нынешним кубиком общую грань). Добравшись до центра нового кубика, кубогрызка поворачивает на 90. Какую наибольшую длину может иметь замкнутый маршрут кубогрызки, не проходящий ни через какой кубик, кроме начального, больше одного раза, и
возвращающийся в начальный только один раз под прямым углом к начальному ходу?
2. В равнобочной трапеции ABCD из точки B опущена высота на большее основание
AD и на ее продолжении взята точка E. Отрезки EC и BD пересекаются в точке P. Докажите,
что SPED = SEBA+SBPC.
3. Решите в целых неотрицательных числах уравнение (xy–7)2 = x2+y2.
4. Докажите, что
a2
b2
3a  2b  c


ab bc
4
при всех положительных a, b и c.
5. На столе лежат n камней. Два игрока по очереди берут камни из кучи; каждый игрок при своем ходе может взять любое количество камней, являющееся квадратом натурального числа. Проигрывает не имеющий хода. Докажите, что для бесконечно многих n второй
игрок может обеспечить себе победу.
6. Дано натуральное число k > 1. Какое наибольшее количество прямых можно расположить на плоскости таким образом, чтобы среди любых k из них нашлись две прямые, образующие угол 60?
7. В треугольнике ABC угол B равен 120 и BC = 2AB. Найдите угол между медианами
AM и BK.
8. Существует ли такое нецелое положительное число x, что [x]3+x2 = x3+[x]2? (Через
[x] обозначается целая часть числа x, то есть наибольшее целое число, не превосходящее x).
СТАРШАЯ ГРУППА, ПЕРВАЯ ЛИГА
1. Дан клетчатый квадрат 99. В центре одного из квадратиков сидит квадрогрызка,
которая может перебираться по прямой в центр соседнего (по стороне) квадратика. Добравшись до центра нового квадратика, квадрогрызка поворачивает на 90. Какую наибольшую
длину может иметь замкнутый маршрут квадрогрызки, не проходящий ни через какой квадратик больше одного раза?
2. Точка D лежит внутри треугольника ABC. На сторонах этого треугольника построены внешним образом прямоугольники BCEF, CAGH и ABKL, площадь каждого из которых
вдвое больше площади треугольника ABC. Докажите, что сумма площадей треугольников
DEF, DGH и DKL в четыре раза больше площади треугольника ABC.
3. Решите в целых неотрицательных числах уравнение (xy–7)2 = x2+y2.
4. Докажите, что
a2
b2
3a  2b  c


ab bc
4
при всех положительных a, b и c.
5. На столе лежат n камней. Два игрока по очереди берут камни из кучи; каждый игрок при своем ходе может взять любое количество камней, являющееся квадратом натураль-
39
ного числа. Проигрывает не имеющий хода. Докажите, что для некоторого n > 1000000 второй игрок может обеспечить себе победу.
6. Дано натуральное число k > 1. Какое наибольшее количество прямых можно расположить на плоскости таким образом, что среди любых k из них нашлись две прямые, образующие угол 90?
7. В треугольнике ABC угол B равен 120 и BC = 2AB. Найдите угол между медианами
AM и BK.
8. Существует ли такое нецелое положительное число x. что [x]+x2 = x+[x]2? (Через [x]
обозначается целая часть числа x, то есть наибольшее целое число, не превосходящее x).
СТАРШАЯ ГРУППА, ВТОРАЯ ЛИГА
1. Квадрат 99 состоит из 81 единичного квадратика. В центре одного из квадратиков
сидит квадрогрызка, которая может перебираться по прямой в центр соседнего по стороне
квадратика. Добравшись до центра нового квадратика, квадрогрызка поворачивает на 90.
Какую наибольшую длину может иметь замкнутый маршрут квадрогрызки, не проходящий
ни через какой квадратик, кроме начального, больше одного раза, и возвращающийся в
начальный только один раз?
2. Точка D лежит внутри треугольника ABC. На сторонах этого треугольника построены внешним образом прямоугольники BCEF, CAGH и ABKL, площадь каждого из которых
вдвое больше площади треугольника ABC. Докажите, что сумма площадей треугольников
DEF, DGH и DKL в четыре раза больше площади треугольника ABC.
3. Над пятизначным числом проводят следующую операцию: заменяют одну из его
цифр на последнюю цифру суммы цифр данного числа. Какое наименьшее число можно получить из числа 13579 несколькими такими операциями?
4. Докажите, что
a2
1
3a  b  2


a 1 1 b
4
при всех положительных a и b.
5. Участники олимпиады устроили вечеринку, договорившись разделить расходы поровну. Получив счет на $1680, они посчитали было, сколько нужно заплатить каждому
участнику, но вдруг заметили, что четыре человека уже ушли, из-за чего взнос каждого из
оставшихся возрос на $1. Сколько человек участвовало в вечеринке?
6. Какое наибольшее число прямых можно провести на плоскости таким образом,
чтобы среди любых пяти из них нашлись две, образующие угол в 90?
7. В треугольнике ABC угол B равен 120 и BC = 2AB. Найдите угол между медианами
AM и BK.
8. Существует ли такое нецелое положительное число x, что [x]2+x = x2+[x]? (Через [x]
обозначается целая часть числа x, то есть наибольшее целое число, не превосходящее x).
МЛАДШАЯ ГРУППА, ВЫСШАЯ ЛИГА
1. На столе лежат a красных, b синих и c белых бусин (причем есть хотя бы по одной
бусине каждого цвета). Петя и Вася ходят по очереди. За каждый ход игрок берет со стола
две или три бусины. Первым ходит Петя. Побеждает тот игрок, после хода которого со стола
исчезнут бусины хотя бы одного из цветов. При каких значениях a, b и c Петя имеет выигрышную стратегию?
2. Даны различные простые числа
abc+bcd+cda+abd+173 ≤ 2abcd.
a,
40
b, c и d. Докажите неравенство
3. Какое наибольшее число прямых можно провести на плоскости таким образом,
чтобы среди любых ста из них нашлись две перпендикулярные?
4. Существует ли такое нецелое положительное число x, что [x]3+x2 = x3+[x]2? (Через
[x] обозначается целая часть числа x, то есть наибольшее целое число, не превосходящее x).
5. Найдите все пары натуральных чисел a и b, для которых 4a+4a2+4 = b2.
6. Про натуральное число n известно три факта:
1) Если оно делится на 3, то оно лежит между 50 и 59 включительно.
2) Если оно не делится на 4, то оно лежит между 60 и 69 включительно.
3) Если оно не делится на 6, то оно лежит между 70 и 79 включительно.
Чему может быть равно число n?
7. На стороне CD выпуклого четырехугольника ABCD отмечена такая точка E, что
AD = DE. На отрезке AE отмечена такая точка F, что AF = EC. Известно, что
ADB = BDC = 90–ABE. Докажите, что BF = BC.
8. Дан клетчатый квадрат 99. В центре одного из квадратиков сидит квадрогрызка,
которая может перебираться по прямой в центр соседнего по стороне квадратика. Добравшись до центра нового квадратика, квадрогрызка поворачивает на 90. Какую наибольшую
длину может иметь замкнутый маршрут квадрогрызки, не проходящий ни через какой квадратик, кроме начального, больше одного раза, и возвращающийся в начальный только один
раз?
МЛАДШАЯ ГРУППА, ПЕРВАЯ ЛИГА
1. На столе лежат a красных, b синих и 4 белых бусины (причем есть хотя бы по одной красной и синей бусине). Петя и Вася ходят по очереди. За каждый ход игрок берет со
стола две или три бусины. Первым ходит Петя. Побеждает тот игрок, после хода которого со
стола исчезнут бусины хотя бы одного из цветов. При каких значениях a и b Петя имеет выигрышную стратегию?
2. Даны различные простые числа a, b и c. Докажите, что ab+bc+ca+29 ≤ 2abc.
3. Какое наибольшее число прямых можно провести на плоскости таким образом,
чтобы среди любых трех из них нашлись две перпендикулярные?
4. Существует ли такое нецелое положительное число x, что [x]3+x2 = x3+[x]2? (Через
[x] обозначается целая часть числа x, то есть наибольшее целое число, не превосходящее x).
5. Найдите все пары (m,n) натуральных чисел, для которых число 4(mn+1) делится на
(m+n) .
2
6. Про натуральное число n известно три факта:
1) Если оно делится на 3, то оно лежит между 50 и 59 включительно.
2) Если оно не делится на 4, то оно лежит между 60 и 69 включительно.
3) Если оно не делится на 6, то оно лежит между 70 и 79 включительно.
Чему может быть равно число n?
7. На стороне CD выпуклого четырехугольника ABCD отмечена такая точка E, что
AD = DE. На отрезке AE отмечена такая точка F, что AF = EC. Известно, что
ADB = BDC = 90–ABE. Докажите, что BF = BC.
8. За круглым столом сидят 180 человек, каждый из которых — рыцарь или лжец.
Каждый из них произнес фразу: «Среди 17 человек, сидящих следом за мной по часовой
стрелке, не менее 9 лжецов». Сколько рыцарей может сидеть за этим столом?
41
МЛАДШАЯ ГРУППА, ВТОРАЯ ЛИГА
1. На столе лежат 5 красных, 6 синих и 6 белых бусин. Петя и Вася ходят по очереди.
За каждый ход игрок берет со стола две или три бусины. Первым ходит Петя. Побеждает тот
игрок, после хода которого со стола исчезнут бусины хотя бы одного из цветов. Кто выигрывает при правильной игре?
2. Даны различные простые числа p и q. Докажите, что p+q+7 ≤ 2pq.
3. Какое наибольшее число прямых можно провести на плоскости таким образом,
чтобы среди любых трех из них нашлись две перпендикулярные?
4. Существует ли такое нецелое положительное число x, что [x]2+x = x2+[x]? (Через [x]
обозначается целая часть числа x, то есть наибольшее целое число, не превосходящее x).
5. Найдите все пары (m,n) натуральных чисел, для которых число 4(mn+1) делится на
(m+n) .
2
6. Про натуральное число n известно три факта:
1) Если оно делится на 3, то оно лежит между 50 и 59 включительно.
2) Если оно не делится на 4, то оно лежит между 60 и 69 включительно.
3) Если оно не делится на 6, то оно лежит между 70 и 79 включительно.
Чему может быть равно число n?
7. Чтобы пронумеровать страницы объемистого тома по порядку, начиная с первой,
печатник использовал 2010 цифр. Сколько страниц содержит том?
8. За круглым столом сидят 180 человек, каждый из которых — рыцарь или лжец.
Каждый из них произнес фразу: «Среди 17 человек, сидящих следом за мной по часовой
стрелке, не менее 9 лжецов». Сколько рыцарей может сидеть за этим столом?
ГРУППА «СТАРТ», ВЫСШАЯ ЛИГА
1. На столе лежат 17 красных и 37 синих бусин. Петя и Вася ходят по очереди. За
каждый ход игрок берет со стола две или три бусины. Первым ходит Петя. Побеждает тот
игрок, после хода которого со стола исчезнут бусины хотя бы одного из цветов. Кто из игроков может обеспечить себе выигрыш, независимо от игры соперника?
2. Для каких натуральных n найдутся натуральные числа a и b такие, что сумма цифр
каждого из чисел a, b, a+b равна n?
3. Какое наибольшее число прямых можно провести на плоскости таким образом,
чтобы среди любых десяти из них нашлись две перпендикулярные?
4. В стране 100 городов. Некоторые пары городов соединены дорогами, причем из
каждого города выходит ровно 5 дорог. Страна разделилась на 2 республики по 50 городов в
каждой. Докажите, что в первой республике дорог, соединяющих её города, столько же,
сколько во второй.
5. За круглым столом сидят 180 человек, каждый из которых — рыцарь или лжец (рыцари всегда говорят правду, лжецы всегда лгут). Каждый из них произнес фразу: «Среди 17
человек, сидящих следом за мной по часовой стрелке, не менее 9 лжецов». Сколько рыцарей
может сидеть за этим столом?
6. Про натуральное число n известно три факта:
1) Если оно делится на 3, то оно лежит между 50 и 59 включительно.
2) Если оно не делится на 4, то оно лежит между 60 и 69 включительно.
3) Если оно не делится на 6, то оно лежит между 70 и 79 включительно.
Чему может быть равно число n?
42
7. У паука есть 8 одинаковых носков и 8 одинаковых ботинок. Паук каждую секунду
либо надевает на одну из своих ног носок, либо натягивает ботинок на какую-нибудь из ног,
на которую носок уже надет (у паука 8 ног; на каждую ногу он надевает один носок и один
ботинок). Два способа обувания паука считаются различными, если паук хотя бы в одну из
16 секунд делает различные действия. Сколькими различными способами паук может обуться?
8. Дан клетчатый квадрат 99. В центре одного из квадратиков сидит квадрогрызка,
которая может перебираться по прямой в центр соседнего по стороне квадратика. Добравшись до центра нового квадратика, квадрогрызка поворачивает на 90. Какую наибольшую
длину может иметь замкнутый маршрут квадрогрызки, не проходящий ни через какой квадратик, кроме начального, больше одного раза, и возвращающийся в начальный только один
раз?
ГРУППА «СТАРТ», ПЕРВАЯ ЛИГА
1. На столе лежат 5 красных, 6 синих и 6 белых бусин. Петя и Вася ходят по очереди.
За каждый ход игрок берет со стола две или три бусины. Первым ходит Петя. Побеждает тот
игрок, после хода которого со стола исчезнут бусины хотя бы одного из цветов. Кто выигрывает при правильной игре?
2. Существуют ли такие натуральные числа a и b, что сумма цифр каждого из чисел a,
b, a+b равна 999?
3. Какое наибольшее число прямых можно провести на плоскости таким образом,
чтобы среди любых трех из них нашлись две перпендикулярные?
4. В стране 100 городов. Некоторые пары городов соединены дорогами, причем из
каждого города выходит ровно 5 дорог. Страна разделилась на 2 республики по 50 городов в
каждой. Докажите, что в первой республике дорог, соединяющих её города, столько же,
сколько во второй.
5. За круглым столом сидят 180 человек, каждый из которых — рыцарь или лжец (рыцари всегда говорят правду, лжецы всегда лгут). Каждый из них произнес фразу: «Среди 17
человек, сидящих следом за мной по часовой стрелке, не менее 9 лжецов». Сколько рыцарей
может сидеть за этим столом?
6. Про натуральное число n известно три факта:
1) Если оно делится на 3, то оно лежит между 50 и 59 включительно.
2) Если оно не делится на 4, то оно лежит между 60 и 69 включительно.
3) Если оно не делится на 6, то оно лежит между 70 и 79 включительно.
Чему может быть равно число n?
7. В бассейн ведут две одинаковых трубы. Одна труба заполняет бассейн за 3 часа.
Сначала включили обе трубы, но через час одна из труб засорилась и через нее вода стала
поступать вдвое медленнее. Через сколько времени бассейн заполнится?
8. Дан клетчатый квадрат 88. В центре одного из квадратиков сидит квадрогрызка,
которая может перебираться по прямой в центр соседнего по стороне квадратика. Добравшись до центра нового квадратика, квадрогрызка поворачивает на 90. Может ли квадрогрызка посетить ровно по одному разу все клетки доски?
Старшая группа, высшая лига, 1 тур, решения и указания для жюри.
1. Куб 333 состоит из 27 единичных кубиков. В центре одного из кубиков сидит
кубогрызка, которая может перебираться по прямой в центр соседнего кубика (то есть
имеющего с ее нынешним кубиком общую грань). Добравшись до центра нового кубика, ку-
43
богрызка поворачивает на 90. Какую наибольшую длину может иметь замкнутый маршрут кубогрызки, не проходящий ни через какой кубик, кроме начального, больше одного раза,
и возвращающийся в начальный только один раз под прямым углом к начальному ходу?
Ответ: 24. Решение. Оценка. Покрасим единичные кубики в черный и белый цвета в шахматном порядке. Заметим, что наша зверушка за ход меняет цвет
кубика в котором находится, откуда следует, что длина ее пути будет четная. Допустим, она равна 26. Тогда кубогрызка посетила по крайней мере 7 угловых кубиков. Заметим, что путь между двумя угловыми кубиками имеет четную длину и
не может быть равен 2, и, значит, не короче 4. Но тогда длина пути не меньше 28
— противоречие. Пример — на рисунке справа (показаны три слоя куба и последовательность обхода кубиков).
1
8
1
1
1
1
2
2
0
1
2
4
 Пример — 4 балла, оценка — 6 баллов. Только ответ — 0 баллов.
2. В равнобочной трапеции ABCD из точки B опущена высота на большее
основание AD и на ее продолжении взята точка E. Отрезки EC и BD пересекаются в точке P. Докажите, что SPED = SEBA+SBPC.
9
3
2
4
3
0
7
1
9
Решение. Добавим к обеим частям SEBP. Получим, что требуется доказать равенство
6 1
площадей четырехугольника ABCE и треугольника BED. Площадь последнего равна
5 6
BEHD/2, где H — точка пересечения BE и AD. Площадь ABCE равна BE(AH+BC)/2. Осталось заметить, что AH+BC = HD, поскольку трапеция равнобокая.
2
2
Ответ: x = 0, y = 7; x = 7, y = 0; x = 3, y = 4; x = 4, y = 3. Решение. Исходное уравнение
2 3
2
2
нетрудно привести к виду (x+y) –(xy–6) = 13  (x+y+xy–6)(x+y–xy+6) = 13. Допустим, xy > 6. 8
Тогда
3 4
x+y+xy–6 > 1, откуда x+y+xy–6 = 13 и x+y–xy+6 = 1. Складывая уравнения и деля на 2, получа7
ем x+y = 7, откуда коротким перебором находим решения (3, 4) и (4, 3). Перебирая же случаи,
когда xy ≤ 6, находим решения (0, 7) и (7, 0).
1
3. Решите в целых неотрицательных числах уравнение (xy–7)2 = x2+y2.
1
1
2
 Вся логика решения верна, но при разборе вариантов потеряна одна из двух пар ответов — дыра в 4 балла. Найдено разумное разложение на множители, дальнейшего содержательного продвижения нет — 4 балла. Только ответ — 0 баллов.
4. Докажите, что
a2
b2
3a  2b  c


при всех положительных a, b и c.
ab bc
4
a2
3a  b

. В самом деле, умножив обе части на
ab
4
a+b, после приведения подобных получаем a2+b2  2ab. Складывая теперь это неравенство с
b2
3b  c
аналогичным неравенством
, получаем искомое.

bc
4
Решение. Докажем сначала, что
5. На столе лежат n камней. Два игрока по очереди берут камни из кучи; каждый
игрок при своем ходе может взять любое количество камней, являющееся квадратом натурального числа. Проигрывает не имеющий хода. Докажите, что для бесконечно многих n
второй игрок может обеспечить себе победу.
Решение. Пусть утверждение задачи неверно. Тогда найдется такое натуральное число a, что при любом исходном числе камней, большем a, побеждает первый. Возьмём кучу
из (a+1)2–1 камней. После первого хода в ней останется не меньше, чем (a+1)2–1 –a2 = 2a
камней, и, следовательно, в получившейся позиции должен побеждать тот, чья очередь ходить. Но это — второй. Противоречие.
44
1
1
6. Дано натуральное число k > 1. Какое наибольшее количество прямых можно расположить на плоскости таким образом, чтобы среди любых k из них нашлись две прямые,
образующие угол 60?
Ответ: 3k–3. Решение. Пример. Проведем через одну точку три прямые, делящие полный угол на шесть углов по 60, затем добавим к каждой k–2 параллельные ей прямые. Оценка. Пусть даны 3k–2 прямые. Разобьем их на несколько семейств по следующему принципу:
две прямые принадлежат одному семейству, если они параллельны или угол между ними равен 60 (легко проверить, что это действительно разбиение). Каждое семейство состоит из
трех наборов параллельных прямых. Очевидно, один из этих наборов содержит не меньше
трети всех прямых этого семейства. Далее, выбрав из каждого семейства по такому набору,
мы получим набор из прямых, содержащий не менее трети из 3k–2 прямых, то есть не менее
k прямых, и угол между любыми двумя из них не равен 60.
 Ответ с примером — 2 балла, без примера — 0 баллов. Только оценка — 6 баллов.
7. В треугольнике ABC угол B равен 120 и BC = 2AB. Найдите угол между медианами AM и BK.
Ответ: 60. Решение. Достроим треугольник ABC до
параллелограмма ABCD, а точку пересечения медиан обозначим через P. Получаем что, точки B, P и D лежат на одной
прямой, а BCD = 60. Из условия следует, что MC = CD, поэтому треугольник MCD — равносторонний. Поэтому
BM = MD, а BMD=120. Из равнобедренности треугольников ABM и BMD получаем BMA = DBM = 30, откуда
BPA = 60.
 Только ответ — 0 баллов.
8. Существует ли такое нецелое положительное число x, что [x] 3+x2 = x3+[x] 2? (Через [x] обозначается целая часть числа x, то есть наибольшее целое число, не превосходящее x).
Ответ: Нет. Решение. Пусть для краткости [x] = a, x–a = b. По условию (a+b)3–
a = (a+b)2–a2. Раскладывая обе части последнего равенства на множители и сокращая на
b  0, получаем равенство (a+b)2+a(a+b)+a2 = 2a+b (*). По условию a  0. Тогда
(a+b)2+a(a+b)+a2 > 3a2, а 2a+b < 2a+1. При a  1 имеем 3a2–2a–1 = (a–1)2+2a2–2  0, и потому равенство (*) тут невозможно. Подставляя в уравнение (*) a = 0, убеждаемся, что в этом
случае оно также не имеет решений, лежащих между 0 и 1, что и завершает доказательство.
3
 Только ответ — 0 баллов.
Старшая группа, первая лига, 1 тур, решения и указания для жюри.
1. Дан клетчатый квадрат 99. В центре одного из квадратиков сидит квадрогрызка, которая может перебираться по прямой в центр соседнего (по стороне) квадратика.
Добравшись до центра нового квадратика, квадрогрызка поворачивает на 90. Какую
наибольшую длину может иметь замкнутый маршрут квадрогрызки, не проходящий ни через какой квадратик больше одного раза?
1 6
4 4
4
Ответ: 64. Решение. Раскрасим клеточки в 4 цвета: четные горизонтали — 121212121, нечетные —
434343434. Легко убедиться, что при такой раскраске
цвета на маршруте квадратогрызки чередуются с периодом 4: …123412341… . Заметим, что клеточек цвета 2
всего 16. Поэтому замкнутый маршрут квадратогрызки
3
1
2
3
4
2
5
0
7
6
0
4
1
4
9
4
7
4
6
5
5
5
7
1
8
5
0
5
5
3
3
3
3
3
3
3
3
2
2
6
4
4
4
9
3
4
5
5
3
8
5
7
1
5
1
6
4
5
0
4
2
8
9
1
1
6
6
8
6
3
1
9
45
6
5
4
3
2
0
1
5
не может быть длиннее 64 клеточек. Пример на 64 — на рисунке справа (клетки пронумерованы в порядке обхода).
 Пример — 4 балла, оценка — 6 баллов. Только ответ — 0 баллов.
2. Точка D лежит внутри треугольника ABC. На сторонах этого треугольника построены внешним образом прямоугольники BCEF, CAGH и ABKL, площадь каждого из которых вдвое больше площади треугольника ABC. Докажите, что сумма площадей треугольников DEF, DGH и DKL в четыре раза больше площади треугольника ABC.
Решение. Пусть перпендикуляр DD1, опущенный из точки D на прямую EF, пересекает прямую BC в точке D2. SDEF = DD1EF/2 = DD2BC/2+D1D2BC/2 = SDBC+SBCEF/2 = SDBC+SABC.
Аналогично SDGH = SDAC+SABC и SDKL = SDAB+SABC. Осталось сложить три полученных равенства.
3. Решите в целых неотрицательных числах уравнение (xy–7)2 = x2+y2.
Ответ: x = 0, y = 7; x = 7, y = 0; x = 3, y = 4; x = 4, y = 3. Решение. Исходное уравнение
нетрудно привести к виду (x+y)2–(xy–6)2 = 13  (x+y+xy–6)(x+y–xy+6) = 13. Допустим, xy > 6.
Тогда
x+y+xy–6 > 1, откуда x+y+xy–6 = 13 и x+y–xy+6 = 1. Складывая уравнения и деля на 2, получаем x+y = 7, откуда коротким перебором находим решения (3, 4) и (4, 3). Перебирая же случаи,
когда xy ≤ 6, находим решения (0, 7) и (7, 0).
 Вся логика решения верна, но при разборе вариантов потеряна одна из двух пар ответов — дыра в 4 балла. Найдено разумное разложение на множители, дальнейшего содержательного продвижения нет — 4 балла. Только ответ — 0 баллов.
4. Докажите, что
a2
b2
3a  2b  c
при всех положительных a, b и c.


ab bc
4
a2
3a  b

Решение. Докажем сначала, что
. В самом деле, умножив обе части на
ab
4
a+b, после приведения подобных получаем a2+b2  2ab. Складывая теперь это неравенство с
b2
3b  c

аналогичным неравенством
, получаем искомое.
bc
4
5. На столе лежат n камней. Два игрока по очереди берут камни из кучи; каждый
игрок при своем ходе может взять любое количество камней, являющееся квадратом натурального числа. Проигрывает не имеющий хода. Докажите, что для некоторого
n > 1000000 второй игрок может обеспечить себе победу.
Решение. Допустим, при всех n > 1000000 побеждает первый. Возьмём кучу из
100000012–1 камней. После первого хода первого игрока в куче останется не меньше, чем
100000012–1 –10000002 = 2000000 камней, и в получившейся позиции должен побеждать тот,
чья очередь ходить. Но это — второй.
6. Дано натуральное число k > 1. Какое наибольшее количество прямых можно расположить на плоскости таким образом, что среди любых k из них нашлись две прямые, образующие угол 90?
Ответ: 2k–2. Решение. Пример: k–1 параллельных прямых и k–1 прямая, перпендикулярная им. Оценка. Пусть есть n прямых, обладающих указанным в задаче свойством. Покрасим одну из них в синий цвет, все перпендикулярные ей — в красный, а все параллельные
— в синий. Если после этого остались непокрашенные прямые — будем повторять процедуру, пока все прямые не будут покрашены. Заметим, что перпендикулярными могут быть
46
только разноцветные прямые. Если n  2k–1, у нас найдутся k одноцветных прямых. Поэтому
n не превосходит 2k–2.
 Ответ с примером — 2 балла, без примера — 0 баллов. Только оценка — 6 баллов.
7. В треугольнике ABC угол B равен 120 и BC = 2AB. Найдите угол между медианами AM и BK.
Ответ: 60. Решение. Достроим треугольник ABC до
параллелограмма ABCD, а точку пересечения медиан обозначим через P. Получаем что, точки B, P и D лежат на одной
прямой, а BCD = 60. Из условия следует, что MC = CD, поэтому треугольник MCD — равносторонний. Поэтому
BM = MD, а BMD=120. Из равнобедренности треугольников ABM и BMD получаем BMA = DBM = 30, откуда
BPA = 60.
 Только ответ — 0 баллов.
8. Существует ли такое нецелое положительное число x. что [x]+x2 = x+[x] 2? (Через
[x] обозначается целая часть числа x, то есть наибольшее целое число, не превосходящее x).
Ответ: Нет. Решение. Пусть для краткости [x] = a, x–a = b. По условию (a+b)2–
a2 = (a+b)–a. Раскладывая обе части последнего равенства на множители и сокращая на b  0,
получаем равенство (a+b)+a = 1  b = 1–2a. Но нецелое число b не может равняться целому
числу 1–2a.
 Только ответ — 0 баллов.
Старшая группа, вторая лига, 1 тур, решения и указания для жюри.
1. Квадрат 99 состоит из 81 единичного квадратика. В центре одного из квадратиков сидит квадрогрызка, которая может перебираться по прямой в центр соседнего по
стороне квадратика. Добравшись до центра нового квадратика, квадрогрызка поворачивает на 90. Какую наибольшую длину может иметь замкнутый маршрут квадрогрызки, не
проходящий ни через какой квадратик, кроме начального, больше одного раза, и возвращающийся в начальный только один раз?
Ответ: 64. Решение. Раскрасим клеточки в 4 цвета: четные горизонтали — 121212121, нечетные —
434343434. Легко убедиться, что при такой раскраске
цвета на маршруте квадратогрызки чередуются с периодом 4: …123412341… . Заметим, что клеточек цвета 2
всего 16. Поэтому замкнутый маршрут квадратогрызки
не может быть длиннее 64 клеточек. Пример на 64 — на
рисунке справа (клетки пронумерованы в порядке обхода).
1
6
4
4
3
1
2
3
4
6
2
5
0
7
6
1
6
9
8
6
3
6
5
9
1
5
1
1
1 —00 баллов.
5 4
 Пример — 4 балла, оценка — 6 баллов. Только ответ
4
7
4
6
7
9
8
5
5
5
0
5
3
3
3
3
3
3
3
3
3
2
2
6
4
4
4
5
1
3
8
4
5
7
1
4
9
5
8
6
4
2
4
4
0
5
4
3
2
5
0
5
1
2. Точка D лежит внутри треугольника ABC. На сторонах
2 3 этого
3 треугольника
2
9по- 8
строены внешним образом прямоугольники BCEF, CAGH и ABKL, площадь
каждого
из
1 1 1 1 2 ко-2 2
торых вдвое больше площади треугольника ABC. Докажите, 4что5 сумма
8 9площадей
2 3 тре6 7
угольников DEF, DGH и DKL в четыре раза больше площади треугольника ABC.
1 1 2 2 2 2
6 прямую
7 0 EF,
1 пересека4 5
Решение. Пусть перпендикуляр DD1, опущенный из точки D на
ет прямую BC в точке D2. SDEF = DD1EF/2 = DD2BC/2+D1D2BC/2 = SDBC+SBCEF/2 = SDBC+SABC.
Аналогично SDGH = SDAC+SABC и SDKL = SDAB+SABC. Осталось сложить три полученных равенства.
47
2
3. Над пятизначным числом проводят следующую операцию: заменяют одну из его
цифр на последнюю цифру суммы цифр данного числа. Какое наименьшее число можно получить из числа 13579 несколькими такими операциями?
Ответ: 11111. Решение. Очевидно, можно получить только пятизначные числа, у которых все цифры нечетны. 11111 самое маленькое из них. Последовательность такая 13579,
15579, 15779, 15799, 11599, 11559, 11155, 11135, 11113, 11117, 11111.
 Только ответ — 0 баллов.
4. Докажите, что
a2
1
3a  b  2
при всех положительных a и b.


a 1 1 b
4
a2
3a  1
Решение. Искомое неравенство получается сложением неравенств
и

a 1
4
1
3b

. Оба эти неравенства сводятся к очевидным применением правила пропорции с
1 b
4
последующим приведением подобных.
5. Участники олимпиады устроили вечеринку, договорившись разделить расходы поровну. Получив счет на $1680, они посчитали было, сколько нужно заплатить каждому
участнику, но вдруг заметили, что четыре человека уже ушли, из-за чего взнос каждого из
оставшихся возрос на $1. Сколько человек участвовало в вечеринке?
Ответ: 84. Решение. Пусть x — число участников вечеринки. По условию
1+1680/x = 1680/(x–4). Это уравнение приводится к квадратному, единственным положительным корнем которого является число 84.
 Только ответ — 2 балла.
6. Какое наибольшее число прямых можно провести на плоскости таким образом,
чтобы среди любых пяти из них нашлись две, образующие угол в 90?
Ответ: 8. Решение. Пример: 4 параллельных прямых и 4 прямых, перпендикулярных
им. Оценка. Пусть есть n прямых, обладающих указанным в задаче свойством. Покрасим одну из них в синий цвет, все перпендикулярные ей — в красный, а все параллельные — в синий. Если после этого остались непокрашенные прямые — будем повторять процедуру, пока
все прямые не будут покрашены. Заметим, что перпендикулярными могут быть только разноцветные прямые. Если n  9, у нас найдутся 5 одноцветных прямых. Поэтому n не превосходит 8.
 Ответ с примером — 2 балла, без примера — 0 баллов. Только оценка — 6 баллов.
7. В треугольнике ABC угол B равен 120 и BC = 2AB. Найдите угол между медианами AM и BK.
Ответ: 60. Решение. Достроим треугольник ABC до
параллелограмма ABCD, а точку пересечения медиан обозначим через P. Получаем что, точки B, P и D лежат на одной
прямой, а BCD = 60. Из условия следует, что MC = CD, поэтому треугольник MCD — равносторонний. Поэтому
BM = MD, а BMD=120. Из равнобедренности треугольников ABM и BMD получаем BMA = DBM = 30, откуда
BPA = 60.
 Только ответ — 0 баллов.
48
8. Существует ли такое нецелое положительное число x, что [x] 2+x = x2+[x]? (Через
[x] обозначается целая часть числа x, то есть наибольшее целое число, не превосходящее x).
Ответ: Нет. Решение. Пусть для краткости [x] = a, x–a = b. По условию (a+b)2–
a = (a+b)–a. Раскладывая обе части последнего равенства на множители и сокращая на b  0,
получаем равенство (a+b)+a = 1  b = 1–2a. Но нецелое число b не может равняться целому
числу 1–2a.
2
 Только ответ — 0 баллов.
Младшая группа, высшая лига, 1 тур, решения и указания для жюри.
1. На столе лежат a красных, b синих и c белых бусин (причем есть хотя бы по одной
бусине каждого цвета). Петя и Вася ходят по очереди. За каждый ход игрок берет со стола две или три бусины. Первым ходит Петя. Побеждает тот игрок, после хода которого со
стола исчезнут бусины хотя бы одного из цветов. При каких значениях a, b и c Петя имеет
выигрышную стратегию?
Ответ: При тех, где одно из чисел a, b и c не больше 3, а если все эти числа больше 3
— когда a+b+c дает остаток 0, 1 или 4 при делении на 5. Решение. Если одно из чисел a, b и
c не больше 3, Петя выигрывает первым же ходом. Далее будем считать, что все эти числа
больше 3. В этом случае сумма a+b+c не меньше 12. Суммы 12 и 13 дают расклады 4,4,4 и
5,4,4, где тот, чья очередь ходить, очевидно, проигрывает. Из позиций с суммами 14, 15 и 16
тот, чья очередь ходить, всегда может перевести партнера в позицию с суммой 12 или 13 (и
каждым из чисел, не меньшим 4), и победить. Из позиций с суммами 17 и 18 тот, чья очередь
ходить, может попасть только в позиции с суммами 14, 15 или 16, и потому проигрывает, а
из позиций с суммами 19, 20 и 21 тот, чья очередь ходить, всегда может перевести партнера в
позицию с суммой 17 или 18 и выиграть. Продолжая по индукции, убеждаемся в правильности ответа.
2. Даны различные простые
abc+bcd+cda+abd+173 ≤ 2abcd.
числа
a,
b,
c
и
d.
Докажите,
что
Решение.
Поделив
обе
части
неравенства
на
abcd,
получим
1/a+1/b+1/c+1/d+173/abcd ≤ 2. Очевидно, наибольшее значение левая часть последнего неравенства принимает, когда данные простые числа равны 2, 3, 5 и 7. Нетрудно проверить, что в
этом случае она равна 2, откуда и вытекает нужное нам неравенство.
3. Какое наибольшее число прямых можно провести на плоскости таким образом,
чтобы среди любых ста из них нашлись две перпендикулярные?
Ответ: 198. Решение. Пример: 99 параллельных прямых и 99 прямых, перпендикулярных им. Оценка. Пусть есть n прямых, обладающих указанным в задаче свойством. Покрасим
одну из них в синий цвет, все перпендикулярные ей — в красный, а все параллельные — в
синий. Если после этого остались непокрашенные прямые — будем повторять процедуру,
пока все прямые не будут покрашены. Заметим, что перпендикулярными могут быть только
разноцветные прямые. Если n  199, у нас найдутся 100 одноцветных прямых. Поэтому n не
превосходит 198.
 Ответ с примером — 2 балла, без примера — 0 баллов. Только оценка — 6 баллов.
4. Существует ли такое нецелое положительное число x, что [x] 3+x2 = x3+[x] 2? (Через [x] обозначается целая часть числа x, то есть наибольшее целое число, не превосходящее x).
Ответ: Нет. Решение. Пусть для краткости [x] = a, x–a = b. По условию (a+b)3–
a = (a+b)2–a2. Раскладывая обе части последнего равенства на множители и сокращая на
b  0, получаем равенство (a+b)2+a(a+b)+a2 = 2a+b (*). По условию a  0. Тогда
(a+b)2+a(a+b)+a2 > 3a2, а 2a+b < 2a+1. При a  1 имеем 3a2–2a–1 = (a–1)2+2a2–2  0, и пото3
49
му равенство (*) тут невозможно. Подставляя в уравнение (*) a = 0, убеждаемся, что в этом
случае оно также не имеет решений, лежащих между 0 и 1, что и завершает доказательство.
 Только ответ — 0 баллов.
5. Найдите все пары натуральных чисел a и b, для которых 4a+4a2+4 = b2.
Ответ: a = 2, b = 6; a = 4, b = 18. Решение. Заметим, что 4a = (2a)2, а b четно. Поэтому
b  (2 +2)2, откуда 4a2+4  2a+2+4  a2  2a. Последнее неравенство, как легко показать, выполняется только при a, равном 2, 3 или 4. Подстановкой убеждаемся, что 4a+4a2+4 будет
точным квадратом только при a = 2 и a = 4, откуда и получаем ответ.
2
a
 Только ответ — 2 балла. Без обоснования используется, что a2  2a только при a = 2,
3, 4 — дыра в 4 балла.
6. Про натуральное число n известно три факта:
1) Если оно делится на 3, то оно лежит между 50 и 59 включительно;
2) Если оно не делится на 4, то оно лежит между 60 и 69 включительно;
3) Если оно не делится на 6, то оно лежит между 70 и 79 включительно.
Чему может быть равно число n?
Ответ: 76. Решение. Допустим n делится на 3. Тогда, в силу 1), оно равно 51, 54 или
57. Но ни одно из этих чисел не делится на 4, и второе утверждение для каждого из них не
выполнено. Итак, искомое число не делится на 3. Но тогда оно не делится и на 6, и, значит,
лежит между 70 и 79. Отсюда, в свою очередь, следует, что искомое число делится на 4. Такое число только одно — 76.
 Только ответ — 0 баллов. Только ответ с проверкой — 2 балла.
7. На стороне CD выпуклого четырехугольника ABCD отмечена такая точка E, что
AD = DE. На отрезке AE отмечена такая точка F, что AF = EC. Известно, что
ADB = BDC = 90–ABE. Докажите, что BF = BC.
Решение. По условию, треугольник EDA — равнобедренный с биссектрисой DB. Поэтому DB — серединный перпендикуляр отрезков EA. Треугольники ADB и EDB равны по
двум сторонам и углу между ними, следовательно, BE = BA и ABD = EBD. Тогда
BEC = EDB+EBD = 90–ABE +EBD = 90–ABD = BAF. Соединяя это равенство с
равенствами AF = EC и BE = BA получаем, что треугольники BEC и BAF равны, откуда и следует, что BF = BC.
8. Дан клетчатый квадрат 99. В центре одного из квадратиков сидит квадрогрызка, которая может перебираться по прямой в центр соседнего по стороне квадратика. Добравшись до центра нового квадратика, квадрогрызка поворачивает на 90. Какую
наибольшую длину может иметь замкнутый маршрут квадрогрызки, не проходящий ни через какой квадратик, кроме начального, больше одного раза, и возвращающийся в начальный
только один раз?
Ответ: 64. Решение. Раскрасим клеточки в 4 цвета: четные горизонтали — 121212121, нечетные —
434343434. Легко убедиться, что при такой раскраске
цвета на маршруте квадратогрызки чередуются с периодом 4: …123412341… . Заметим, что клеточек цвета 2
всего 16. Поэтому замкнутый маршрут квадратогрызки
не может быть длиннее 64 клеточек. Пример на 64 — на
рисунке справа (клетки пронумерованы в порядке обхода).
1
6
4
4
3
1
2
3
4
6
2
5
0
7
6
1
6
9
8
6
3
6
5
5
5
5
7
50
0
5
4
1
2
3
3
2
1
1
4
5
1
8
7
5
3
2
0
1
9
8
5
1
2
2
3
2
6
3
3
3
3
3
3
3
2
2
2
2
5
4
5
0
1
9
4
8
3
6
4
4
5
3
4
4
5
1
3
8
7
9
 Пример — 4 балла, оценка — 6 баллов. Только ответ — 0 баллов.
1 1 5
1
4
6
6
4
9
5
8
9
4
2
4
4
0
7
Младшая группа, первая лига, 1 тур, решения и указания для жюри.
1. На столе лежат a красных, b синих и 4 белых бусины (причем есть хотя бы по одной красной и синей бусине). Петя и Вася ходят по очереди. За каждый ход игрок берет со
стола две или три бусины. Первым ходит Петя. Побеждает тот игрок, после хода которого со стола исчезнут бусины хотя бы одного из цветов. При каких значениях a и b Петя
имеет выигрышную стратегию?
Ответ: Если одно из чисел a, b не больше 3 или a+b+4 дает остаток 0, 1 или 4 при делении на 5, побеждает Петя, иначе — Вася. Решение. Если одно из чисел a или b не больше
3, Петя выигрывает первым же ходом. Далее будем считать, что оба этих числа больше 3. В
этом случае сумма a+b+4 не меньше 12. Суммы 12 и 13 дают расклады 4,4,4 и 5,4,4, где тот,
чья очередь ходить, очевидно, проигрывает. Из позиций с суммами 14, 15 и 16 тот, чья очередь ходить, всегда может перевести партнера в позицию с суммой 12 или 13 (и каждым из
чисел, не меньшим 4), и победить. Из позиций с суммами 17 и 18 тот, чья очередь ходить,
может попасть только в позиции с суммами 14, 15 или 16, и потому проигрывает, а из позиций с суммами 19, 20 и 21 тот, чья очередь ходить, всегда может перевести партнера в позицию с суммой 17 или 18 и выиграть. Продолжая по индукции, убеждаемся в правильности
ответа.
2. Даны различные простые числа a, b и c. Докажите, что ab+bc+ca+29 ≤ 2abc.
Решение. Поделив обе части неравенства на abc, получим 1/a+1/b+1/c+29/abc ≤ 2.
Очевидно, наибольшее значение левая часть последнего неравенства принимает, когда данные простые числа равны 2, 3 и 5. Нетрудно проверить, что в этом случае она равна 2, откуда
и вытекает нужное нам неравенство.
3. Какое наибольшее число прямых можно провести на плоскости таким образом,
чтобы среди любых трех из них нашлись две перпендикулярные?
Ответ: 4. Решение. Пример для 4: продолжения сторон прямоугольника. Оценка.
Пусть есть пять прямых. Среди них найдутся две не перпендикулярных — обозначим их a и
b. Каждая из трёх оставшихся прямых перпендикулярна либо a, либо b, и найдутся две, перпендикулярные одной и той же — пусть это a. Тогда среди этих двух прямых и прямой b нет
двух перпендикулярных.
 Ответ с примером — 2 балла, без примера — 0 баллов. Только оценка — 6 баллов.
4. Существует ли такое нецелое положительное число x, что [x] 3+x2 = x3+[x] 2? (Через [x] обозначается целая часть числа x, то есть наибольшее целое число, не превосходящее x).
Ответ: Нет. Решение. Пусть для краткости [x] = a, x–a = b. По условию (a+b)3–
a3 = (a+b)2–a2. Раскладывая обе части последнего равенства на множители и сокращая на
b  0, получаем равенство (a+b)2+a(a+b)+a2 = 2a+b (*). По условию a  0. Тогда
(a+b)2+a(a+b)+a2 > 3a2, а 2a+b < 2a+1. При a  1 имеем 3a2–2a–1 = (a–1)2+2a2–2  0, и потому равенство (*) тут невозможно. Подставляя в уравнение (*) a = 0, убеждаемся, что в этом
случае оно также не имеет решений, лежащих между 0 и 1, что и завершает доказательство.
 Только ответ — 0 баллов.
5. Найдите все пары (m,n) натуральных чисел, для которых число 4(mn+1) делится на
(m+n) .
2
Ответ: m = n+2 (n  1), n = m+2 (m  1), m = n = 1. Решение. Если частное от деления
4(mn+1) на (m+n)2 равно k  2, имеем: 4(mn+1) = k(m+n)2  4kmn, откуда (k–1)mn ≤ 1, и
m = n = 1. Равенство же 4(mn+1) = (m+n)2 равносильно равенству (m–n)2 = 4, что дает первые
две серии ответов.
51
 Только ответ — 2 балла. Потеря ответа (1, 1) — дыра в 2 балла.
6. Про натуральное число n известно три факта:
1) Если оно делится на 3, то оно лежит между 50 и 59 включительно;
2) Если оно не делится на 4, то оно лежит между 60 и 69 включительно;
3) Если оно не делится на 6, то оно лежит между 70 и 79 включительно.
Чему может быть равно число n?
Ответ: 76. Решение. Допустим n делится на 3. Тогда, в силу 1), оно равно 51, 54 или
57. Но ни одно из этих чисел не делится на 4, и второе утверждение для каждого из них не
выполнено. Итак, искомое число не делится на 3. Но тогда оно не делится и на 6, и, значит,
лежит между 70 и 79. Отсюда, в свою очередь, следует, что искомое число делится на 4. Такое число только одно — 76.
 Только ответ — 0 баллов. Только ответ с проверкой — 2 балла.
7. На стороне CD выпуклого четырехугольника ABCD отмечена такая точка E, что
AD = DE. На отрезке AE отмечена такая точка F, что AF = EC. Известно, что
ADB = BDC = 90–ABE. Докажите, что BF = BC.
Решение. По условию, треугольник EDA — равнобедренный с биссектрисой DB. Поэтому DB — серединный перпендикуляр отрезков EA. Треугольники ADB и EDB равны по
двум сторонам и углу между ними, следовательно, BE = BA и ABD = EBD. Тогда
BEC = EDB+EBD = 90–ABE +EBD = 90–ABD = BAF. Соединяя это равенство с
равенствами AF = EC и BE = BA получаем, что треугольники BEC и BAF равны, откуда и следует, что BF = BC.
8. За круглым столом сидят 180 человек, каждый из которых — рыцарь или лжец.
Каждый из них произнес фразу: «Среди 17 человек, сидящих следом за мной по часовой
стрелке, не менее 9 лжецов». Сколько рыцарей может сидеть за этим столом?
Ответ: 90. Решение. Из условия ясно, что из 17 человек, сидящих следом за лжецом,
не менее 9 — рыцари, а из 17 человек, сидящих следом за рыцарем, не менее 9 — лжецы.
Допустим, среди 17 человек, сидящих за каким-то рыцарем, хотя бы 10 — лжецы. Но тогда
среди 17 человек, сидящих следом за первым из этих лжецов, есть хотя бы 9 лжецов, и получается, что этот лжец сказал правду. Стало быть, среди 17 человек, сидящих следом за каждым рыцарем, ровно 9 лжецов (и, стало быть, 8 рыцарей). Аналогично, среди 17 человек, сидящих следом за каждым лжецом, ровно 9 рыцарей и 8 лжецов. Поэтому среди любых 18 человек, сидящих подряд, 9 рыцарей и 9 лжецов, откуда и следует ответ.
 Предъявлять схему рассадки 90 рыцарей и 90 лжецов не требуется. Только ответ с
примером — 0 баллов.
Младшая группа, вторая лига, 1 тур, решения и указания для жюри.
1. На столе лежат 5 красных, 6 синих и 6 белых бусин. Петя и Вася ходят по очереди. За каждый ход игрок берет со стола две или три бусины. Первым ходит Петя. Побеждает тот игрок, после хода которого со стола исчезнут бусины хотя бы одного из цветов.
Кто выигрывает при правильной игре?
Ответ: Вася. Решение. Если после первого хода Пети появится кучка, где не больше
трех бусин, Вася, очевидно, выиграет своим первым ходом. В противном случае Вася может
своим первым ходом создать позицию 4, 4, 4 (потому что 5 = 4+1, 6 = 4+2, 2+2+1 = 5, и Вася
всегда может своим ходом дополнить количество спичек, взятых Петей, до 5), которая, как
легко показать, проигрышна для того, чья очередь ходить.
2. Даны различные простые числа p и q. Докажите, что p+q+7 ≤ 2pq.
52
Решение. Поделив обе части неравенства на pq, получим 1/p+1/q+7/pq ≤ 2. Очевидно,
наибольшее значение левая часть последнего неравенства принимает, когда простые числа
равны 2 и 3. Нетрудно проверить, что в этом случае она равна 2, откуда и вытекает нужное
нам неравенство.
3. Какое наибольшее число прямых можно провести на плоскости таким образом,
чтобы среди любых трех из них нашлись две перпендикулярные?
Ответ: 4. Решение. Пример для 4: продолжения сторон прямоугольника. Оценка.
Пусть есть пять прямых. Среди них найдутся две не перпендикулярных — обозначим их a и
b. Каждая из трёх оставшихся прямых перпендикулярна либо a, либо b, и найдутся две, перпендикулярные одной и той же — пусть это a. Тогда среди этих двух прямых и прямой b нет
двух перпендикулярных.
 Ответ с примером — 2 балла, без примера — 0 баллов. Только оценка — 6 баллов.
4. Существует ли такое нецелое положительное число x, что [x] 2+x = x2+[x]? (Через
[x] обозначается целая часть числа x, то есть наибольшее целое число, не превосходящее x).
Ответ: Нет. Решение. Пусть для краткости [x] = a, x–a = b. По условию (a+b)2–
a2 = (a+b)–a. Раскладывая обе части последнего равенства на множители и сокращая на b  0,
получаем равенство (a+b)+a = 1  b = 1–2a. Но нецелое число b не может равняться целому
числу 1–2a.
Только ответ — 0 баллов.
5. Найдите все пары (m,n) натуральных чисел, для которых число 4(mn+1) делится на
(m+n) .
2
Ответ: m = n+2 (n  1), n = m+2 (m  1), m = n = 1. Решение. Если частное от деления
4(mn+1) на (m+n)2 равно k  2, имеем: 4(mn+1) = k(m+n)2  4kmn, откуда (k–1)mn ≤ 1, и
m = n = 1. Равенство же 4(mn+1) = (m+n)2 равносильно равенству (m–n)2 = 4, что дает первые
две серии ответов.
 Только ответ — 2 балла. Потеря ответа (1, 1) — дыра в 2 балла.
6. Про натуральное число n известно три факта:
1) Если оно делится на 3, то оно лежит между 50 и 59 включительно;
2) Если оно не делится на 4, то оно лежит между 60 и 69 включительно;
3) Если оно не делится на 6, то оно лежит между 70 и 79 включительно.
Чему может быть равно число n?
Ответ: 76. Решение. Допустим n делится на 3. Тогда, в силу 1), оно равно 51, 54 или
57. Но ни одно из этих чисел не делится на 4, и второе утверждение для каждого из них не
выполнено. Итак, искомое число не делится на 3. Но тогда оно не делится и на 6, и, значит,
лежит между 70 и 79. Отсюда, в свою очередь, следует, что искомое число делится на 4. Такое число только одно — 76.
 Только ответ — 0 баллов. Только ответ с проверкой — 2 балла.
7. Чтобы пронумеровать страницы объемистого тома по порядку, начиная с первой,
печатник использовал 2010 цифр. Сколько страниц содержит том?
Ответ: 706. Решение. На страницы с первой по девятую понадобилось 9 цифр, с десятой по 99-ю — 290 = 180 цифр. Трехзначные числа от 100 до 999 содержат вместе
3900 = 2700 цифр, что больше, чем 2010. Поэтому в книге меньше 1000 страниц, и номер
последней
страницы
равен
99+(2010–9–180)/3 = 1821/3 = 706.
53
 Ход решения верен, но забыли прибавить 99 (ответ — 607) — дыра в 4 балла. Только ответ — 0 баллов.
8. За круглым столом сидят 180 человек, каждый из которых — рыцарь или лжец.
Каждый из них произнес фразу: «Среди 17 человек, сидящих следом за мной по часовой
стрелке, не менее 9 лжецов». Сколько рыцарей может сидеть за этим столом?
Ответ: 90. Решение. Из условия ясно, что из 17 человек, сидящих следом за лжецом,
не менее 9 — рыцари, а из 17 человек, сидящих следом за рыцарем, не менее 9 — лжецы.
Допустим, среди 17 человек, сидящих за каким-то рыцарем, хотя бы 10 — лжецы. Но тогда
среди 17 человек, сидящих следом за первым из этих лжецов, есть хотя бы 9 лжецов, и получается, что этот лжец сказал правду. Стало быть, среди 17 человек, сидящих следом за каждым рыцарем, ровно 9 лжецов (и, стало быть, 8 рыцарей). Аналогично, среди 17 человек, сидящих следом за каждым лжецом, ровно 9 рыцарей и 8 лжецов. Поэтому среди любых 18 человек, сидящих подряд, 9 рыцарей и 9 лжецов, откуда и следует ответ.
 Предъявлять схему рассадки 90 рыцарей и 90 лжецов не требуется. Только ответ с
примером — 0 баллов.
Группа «Старт», высшая лига, 1 тур, решения и указания для жюри.
1. На столе лежат 17 красных и 37 синих бусин. Петя и Вася ходят по очереди. За
каждый ход игрок берет со стола две или три бусины. Первым ходит Петя. Побеждает
тот игрок, после хода которого со стола исчезнут бусины хотя бы одного из цветов. Кто
из игроков может обеспечить себе выигрыш, независимо от игры соперника?
Ответ: Вася. Решение. Если после очередного хода Пети бусин какого-то цвета осталось не больше 3, Вася победит следующим ходом. В противном случае Вася каждый раз берёт 2 бусины, если Петя перед этим взял 3, и 3 бусины, если Петя взял две, следя за тем, чтобы бусин каждого цвета оставалось не меньше 4 (что всегда возможно, пока бусин остается
не меньше 11). После 9 пар таких ходов на столе останется 9 бусин, причем 5 будут одного
цвета, а 4 — другого, и будет Петина очередь ходить. После любого его хода на столе останется не больше 7 бусин, то есть бусин какого-то цвета будет не больше 3, и Вася выиграет
следующим ходом.
 Только ответ — 0 баллов.
2. Для каких натуральных n найдутся натуральные числа a и b такие, что сумма
цифр каждого из чисел a, b, a+b равна n?
Ответ: Для всех n, делящихся на 9, и только для них. Решение. Пример.
9...9  9...9  19...98 — сумма цифр равна 9k. Оценка. Пусть число n дает при делении на 9
k
k
k 1
ненулевой остаток l. Тогда если суммы цифр чисел a и b равны n, то сумма цифр числа a+b
дает при делении на 9 остаток 2l, не равный l.
 Только ответ — 0 баллов. Ответ с примером без оценки — 4 балла. Оценка без
примера — 6 баллов.
3. Какое наибольшее число прямых можно провести на плоскости таким образом,
чтобы среди любых десяти из них нашлись две перпендикулярные?
Ответ: 18. Решение. Пример: 9 параллельных прямых и 9 прямых, перпендикулярных
им. Оценка. Пусть есть n прямых, обладающих указанным в задаче свойством. Покрасим одну из них в синий цвет, все перпендикулярные ей — в красный, а все параллельные — в синий. Если после этого остались непокрашенные прямые — будем повторять процедуру, пока
все прямые не будут покрашены. Заметим, что перпендикулярными могут быть только разноцветные прямые. Если n  19, у нас найдутся 10 одноцветных прямых. Поэтому n не превосходит 18.
54
 Ответ с примером — 2 балла, без примера — 0 баллов. Только оценка — 6 баллов.
4. В стране 100 городов. Некоторые пары городов соединены дорогами, причем из
каждого города выходит ровно 5 дорог. Страна разделилась на 2 республики по 50 городов в
каждой. Докажите, что в первой республике дорог, соединяющих её города, столько же,
сколько во второй.
Решение. Пусть первую республику со второй соединяет k дорог. Тогда дорог, соединяющих ее города, как в первой, так и во второй республике — по (550–k)/2.
5. За круглым столом сидят 180 человек, каждый из которых — рыцарь или лжец.
Каждый из них произнес фразу: «Среди 17 человек, сидящих следом за мной по часовой
стрелке, не менее 9 лжецов». Сколько рыцарей может сидеть за этим столом?
Ответ: 90. Решение. Из условия ясно, что из 17 человек, сидящих следом за лжецом,
не менее 9 — рыцари, а из 17 человек, сидящих следом за рыцарем, не менее 9 — лжецы.
Допустим, среди 17 человек, сидящих за каким-то рыцарем, хотя бы 10 — лжецы. Но тогда
среди 17 человек, сидящих следом за первым из этих лжецов, есть хотя бы 9 лжецов, и получается, что этот лжец сказал правду. Стало быть, среди 17 человек, сидящих следом за каждым рыцарем, ровно 9 лжецов (и, стало быть, 8 рыцарей). Аналогично, среди 17 человек, сидящих следом за каждым лжецом, ровно 9 рыцарей и 8 лжецов. Поэтому среди любых 18 человек, сидящих подряд, 9 рыцарей и 9 лжецов, откуда и следует ответ.
 Предъявлять схему рассадки 90 рыцарей и 90 лжецов не требуется. Только ответ с
примером — 0 баллов.
6. Про натуральное число n известно три факта:
1) Если оно делится на 3, то оно лежит между 50 и 59 включительно.
2) Если оно не делится на 4, то оно лежит между 60 и 69 включительно.
3) Если оно не делится на 6, то оно лежит между 70 и 79 включительно.
Чему может быть равно число n?
Ответ: 76. Решение. Допустим n делится на 3. Тогда, в силу 1), оно равно 51, 54 или
57. Но ни одно из этих чисел не делится на 4, и второе утверждение для каждого из них не
выполнено. Итак, искомое число не делится на 3. Но тогда оно не делится и на 6, и, значит,
лежит между 70 и 79. Отсюда, в свою очередь, следует, что искомое число делится на 4. Такое число только одно — 76.
 Только ответ — 0 баллов. Только ответ с проверкой — 2 балла.
7. У паука есть 8 одинаковых носков и 8 одинаковых ботинок. Паук каждую секунду
либо надевает на одну из своих ног носок, либо натягивает ботинок на какую-нибудь из ног,
на которую носок уже надет (у паука 8 ног; на каждую ногу он надевает один носок и один
ботинок). Два способа обувания паука считаются различными, если паук хотя бы в одну из
16 секунд делает различные действия. Сколькими различными способами паук может
обуться?
Ответ: 16!/28. Решение. Обуть первую лапу паук может в любые две секунды из шестнадцати (в первую из них он надевает на лапу носок, во вторую – ботинок). Выбрать две секунды из шестнадцати можно 1615/2 способами. Аналогично, для обувания второй лапы
надо выбрать две секунды из оставшихся четырнадцати. Это можно сделать 1413/2 способами. Продолжая рассуждения для остальных лап и применяя правило умножения вариантов,
получаем ответ.
 Только ответ — 2 балла.
8. Дан клетчатый квадрат 99. В центре одного из квадратиков сидит квадрогрызка, которая может перебираться по прямой в центр соседнего по стороне квадратика. До-
55
бравшись до центра нового квадратика, квадрогрызка поворачивает на 90. Какую
наибольшую длину может иметь замкнутый маршрут квадрогрызки, не проходящий ни через какой квадратик, кроме начального, больше одного раза, и возвращающийся в начальный
только один раз?
Ответ: 64. Решение. Раскрасим клеточки в 4 цвета: четные горизонтали — 121212121, нечетные —
434343434. Легко убедиться, что при такой раскраске
цвета на маршруте квадратогрызки чередуются с периодом 4: …123412341… . Заметим, что клеточек цвета 2
всего 16. Поэтому замкнутый маршрут квадратогрызки
не может быть длиннее 64 клеточек. Пример на 64 — на
рисунке справа (клетки пронумерованы в порядке обхода).
1
6
4
4
3
1
2
3
4
6
2
5
0
7
6
1
6
9
8
6
3
6
5
9
4
5
4
6
7
9
8
5
1
3
3
3
3
3
3
3
3
2
2
6
4
4
4
5
3
4
5
7
3
8
7
 Пример — 4 балла, оценка — 6 баллов. Только ответ — 0 баллов.
1 1 5 5 5
Группа «Старт», первая лига, 1 тур, решения и1указания
0 5 для
4 жюри.
1 0
1
4
9
5
8
6
4
2
4
4
0
5
4
3
2
0
1
5
5
1. На столе лежат 5 красных, 6 синих и 6 белых бусин.
и Вася
по очере2 Петя
3
3 ходят
2
9 8
ди. За каждый ход игрок берет со стола две или три бусины. Первым ходит
1 1 Петя.
1 1Побеж2 2 2
5 бы
8 одного
9 2 из цветов.
3 6 7
дает тот игрок, после хода которого со стола исчезнут бусины4 хотя
1
1
2
2 2 2
Кто выигрывает при правильной игре?
6
7
0
1
4
5
Ответ: Вася. Решение. Если после первого хода Пети появится кучка, где не больше
трех бусин, Вася, очевидно, выиграет своим первым ходом. В противном случае Вася может
своим первым ходом создать позицию 4, 4, 4 (потому что 5 = 4+1, 6 = 4+2, 2+2+1 = 5, и Вася
всегда может своим ходом дополнить количество спичек, взятых Петей, до 5), которая, как
легко показать, проигрышна для того, чья очередь ходить.
2. Существуют ли такие натуральные числа a и b, что сумма цифр каждого из чисел
a, b, a+b равна 999?
Ответ: Да. Решение. 9...9  9...9  1 9...9 8 .
111 раз
111 раз
110 раз
 Только ответ «да» — 0 баллов.
3. Какое наибольшее число прямых можно провести на плоскости таким образом,
чтобы среди любых трех из них нашлись две перпендикулярные?
Ответ: 4. Решение. Пример для 4: продолжения сторон прямоугольника. Оценка.
Пусть есть пять прямых. Среди них найдутся две не перпендикулярных — обозначим их a и
b. Каждая из трёх оставшихся прямых перпендикулярна либо a, либо b, и найдутся две, перпендикулярные одной и той же — пусть это a. Тогда среди этих двух прямых и прямой b нет
двух перпендикулярных.
 Ответ с примером — 2 балла, без примера — 0 баллов. Только оценка — 6 баллов.
4. В стране 100 городов. Некоторые пары городов соединены дорогами, причем из
каждого города выходит ровно 5 дорог. Страна разделилась на 2 республики по 50 городов в
каждой. Докажите, что в первой республике дорог, соединяющих её города, столько же,
сколько во второй.
Решение. Пусть первую республику со второй соединяет k дорог. Тогда дорог, соединяющих ее города, как в первой, так и во второй республике — по (550–k)/2.
5. За круглым столом сидят 180 человек, каждый из которых — рыцарь или лжец.
Каждый из них произнес фразу: «Среди 17 человек, сидящих следом за мной по часовой
стрелке, не менее 9 лжецов». Сколько рыцарей может сидеть за этим столом?
56
2
Ответ: 90. Решение. Из условия ясно, что из 17 человек, сидящих следом за лжецом,
не менее 9 — рыцари, а из 17 человек, сидящих следом за рыцарем, не менее 9 — лжецы.
Допустим, среди 17 человек, сидящих за каким-то рыцарем, хотя бы 10 — лжецы. Но тогда
среди 17 человек, сидящих следом за первым из этих лжецов, есть хотя бы 9 лжецов, и получается, что этот лжец сказал правду. Стало быть, среди 17 человек, сидящих следом за каждым рыцарем, ровно 9 лжецов (и, стало быть, 8 рыцарей). Аналогично, среди 17 человек, сидящих следом за каждым лжецом, ровно 9 рыцарей и 8 лжецов. Поэтому среди любых 18 человек, сидящих подряд, 9 рыцарей и 9 лжецов, откуда и следует ответ.
 Предъявлять схему рассадки 90 рыцарей и 90 лжецов не требуется. Только ответ с
примером — 0 баллов.
6. Про натуральное число n известно три факта:
1) Если оно делится на 3, то оно лежит между 50 и 59 включительно.
2) Если оно не делится на 4, то оно лежит между 60 и 69 включительно.
3) Если оно не делится на 6, то оно лежит между 70 и 79 включительно.
Чему может быть равно число n?
Ответ: 76. Решение. Допустим n делится на 3. Тогда, в силу 1), оно равно 51, 54 или
57. Но ни одно из этих чисел не делится на 4, и второе утверждение для каждого из них не
выполнено. Итак, искомое число не делится на 3. Но тогда оно не делится и на 6, и, значит,
лежит между 70 и 79. Отсюда, в свою очередь, следует, что искомое число делится на 4. Такое число только одно — 76.
 Только ответ — 0 баллов. Только ответ с проверкой — 2 балла.
7. В бассейн ведут две одинаковых трубы. Одна труба заполняет бассейн за 3 часа.
Сначала включили обе трубы, но через час одна из труб засорилась и через нее вода стала
поступать вдвое медленнее. Через сколько времени бассейн заполнится?
Ответ: Через 1 час 40 минут. Решение. Каждая труба за час заполняет треть бассейна.
За первый час две трубы заполнили 2/3 бассейна. После этого вторая труба стала заполнять
бассейн со скоростью 1/6 бассейна в час, стало быть, две трубы вместе — со скоростью
1/6+1/3 = 1/2 бассейна в час. Поэтому оставшуюся треть бассейна они заполнят за
1/3:1/2 = 2/3 часа.
8. Дан клетчатый квадрат 88. В центре одного из квадратиков сидит квадрогрызка, которая может перебираться по прямой в центр соседнего по стороне квадратика. Добравшись до центра нового квадратика, квадрогрызка поворачивает на 90. Может ли квадрогрызка посетить ровно по одному разу все клетки доски?
Ответ: Нет. Решение. Легко убедиться, что если квадратогрызка вошла в угловую
клетку квадрата, то после следующего хода она не сможет никуда свернуть.
МАТЕМАТИЧЕСКИЙ БОЙ №2. 17.02.2011
СТАРШАЯ ГРУППА, ВЫСШАЯ ЛИГА
1. В компании 2011 человек. Оказалось, что любые двое имеют хотя бы двух общих
знакомых. Назовем трио любую тройку попарно знакомых. Докажите, что можно выбрать
3333 различных трио.
2. Докажите, что если числа ab, cd и ac+bd делятся на k, то ac и bd тоже делятся на k
(a, b, c, d, k — натуральные числа)
3. Можно ли вырезать из доски 88 восемь клеток так, чтобы из оставшихся клеток
нельзя было вырезать по клеткам прямоугольник площади, не меньшей 8?
57
x13  x23  ...  xn3   x1  x2  ...  xn 
4. Решите уравнение
2
в различных нату-
ральных числах.
5. Точки K и L расположены на сторонах AD и BC выпуклого четырехугольника
ABCD соответственно так, что AK/KD = CL/LB. Прямая KL пересекает отрезки AC и BD в
точках P и Q соответственно. Докажите, что KP/QL = SACD/SBCD.
6. Докажите, что a +b +c
2
2
2
3(a  b) 2
 ab+bc+ca+
4
при любых a, b и c.
7. Внутри треугольника ABC (AB < BC) лежит точка O, равноудаленная от трех его
вершин. BD — биссектриса угла B. Точка M — середина стороны AC, а точка P на луче MO
такова, что APC = ABC. Точка N — основание перпендикуляра, опущенного из P на BC.
Докажите, что каждая из диагоналей четырехугольника BDMN делит треугольник ABC на
две равновеликие части.
8. Дано натуральное число N. Два игрока по очереди делают такие ходы: A пишет на
доске число 1 или –1, B прибавляет к этому числу 2 или вычитает из него 2, потом A прибавляет или вычитает 3 и т.д. (на k-м ходу игрок прибавляет k или вычитает k). Игрок, после хода которого число впервые будет делиться на N, выигрывает. Для каждого N, большего 10,
определите, есть ли у одного из игроков выигрышная стратегия, и если есть, то у какого.
СТАРШАЯ ГРУППА, ПЕРВАЯ ЛИГА
1. В компании 2011 человек. Оказалось, что любые двое имеют хотя бы двух общих
знакомых. Назовем трио любую тройку попарно знакомых. Докажите, что можно выбрать
3333 различных трио.
2. Докажите, что если числа ab, cd и ac+bd делятся на k, то ac и bd тоже делятся на k
(a, b, c, d, k — натуральные числа).
3. Можно ли вырезать из доски 88 восемь клеток так, чтобы из оставшихся клеток
нельзя было вырезать по клеткам прямоугольник площади, не меньшей 8?
4. Решите уравнение
x3  y 3  z 3  t 3   x  y  z  t 
2
в различных натураль-
ных числах.
5. Через вершину C равнобедренного треугольника ABC (AB = AC) с острым углом
при вершине A провели перпендикуляр к BC и на этом перпендикуляре отметили точку P,
лежащую с той же стороны от прямой BC, что и A, и с той же стороны от прямой AB, что и C.
Точка D такова, что ABPD — параллелограмм. M — точка пересечения прямой PC и отрезка
AD. Найдите отношение DM/DA.
6. Докажите, что a +b +c
2
2
2
3(a  b) 2
 ab+bc+ca+
4
при любых a, b и c.
7. Внутри треугольника ABC (AB < BC) лежит точка O, равноудаленная от трех его
вершин. BD — биссектриса угла B. Точка M — середина стороны AC, а точка P на луче MO
такова, что APC = ABC. Точка N — основание перпендикуляра, опущенного из P на BC.
Докажите, что каждая из диагоналей четырехугольника BDMN делит треугольник ABC на
две равновеликие части.
8. В коробке лежат 50 синих и 50 красных шариков. Вася хочет набрать 25 синих шариков из коробки. Для этого он вслепую вытаскивает по два шарика. Если он вытащил два
синих, то берет оба. Если синий и красный, то берет синий, а красный кладет обратно в ко-
58
робку. Наконец, если он вытащил два красных, то оба откладывает в сторону. За какое
наименьшее число вытаскиваний Вася заведомо сможет набрать 25 синих шариков?
СТАРШАЯ ГРУППА, ВТОРАЯ ЛИГА
1. Дорогу разделили на 2011 частей (не обязательно равной длины). Яна знает длину
дороги. Ей разрешено спросить, чему равно расстояние между серединами любых двух частей, при этом количество вопросов неограничено. Длину каких частей она сможет узнать?
2. Докажите, что если числа ab, cd и ac+bd делятся на k, то ac и bd тоже делятся на k.
3. Можно ли вырезать из доски 88 восемь клеток так, чтобы из оставшихся клеток
нельзя было вырезать по клеткам прямоугольник площади, не меньшей 8?
4. Решите уравнение x3+y3 = (x+y)2 в различных натуральных числах.
5. Через вершину C равнобедренного треугольника ABC (AB = AC) с острым углом
при вершине A провели перпендикуляр к BC и на этом перпендикуляре отметили точку P,
лежащую с той же стороны от прямой BC, что и A, и с той же стороны от прямой AB, что и C.
Точка D такова, что ABPD — параллелограмм. M — точка пересечения прямой PC и отрезка
AD. Найдите отношение DM/DA.
6. В двух коробках лежат конфеты. Если к коробке подходит Вася, то он берет 1/17
часть конфет, имеющихся в данный момент в этой коробке, если Петя — то 1/12, а Коля всегда берет 1/4 часть. Если у берущего конфеты не получается взять ровно столько, сколько
надо, то он воздерживается от сладкого. Вначале в коробках конфет было поровну, по истечении некоторого срока оказалось, что конфет снова поровну. Можно ли утверждать, что
каждый из мальчиков лакомился из первой коробки столько же раз, сколько из второй?
7. Внутри угла отмечена точка P, не лежащая на его биссектрисе. Отрезки AB и CD
проходят через точку P, при этом A и C лежат на одной стороне угла, B и D на другой, отрезок CD перпендикулярен биссектрисе угла, а отрезок AB делится точкой P пополам. Докажите, что AB > CD.
8. В коробке лежат 50 синих и 50 красных шариков. Вася хочет набрать 25 синих шариков из коробки. Для этого он вслепую вытаскивает по два шарика. Если он вытащил два
синих, то берет оба. Если синий и красный, то берет синий, а красный кладет обратно в коробку. Наконец, если он вытащил два красных, то оба откладывает в сторону. За какое
наименьшее число вытаскиваний Вася заведомо сможет набрать 25 синих шариков?
МЛАДШАЯ ГРУППА, ВЫСШАЯ ЛИГА
1. В компании 2011 человек. Оказалось, что любые двое имеют хотя бы двух общих
знакомых. Назовем трио любую тройку попарно знакомых. Докажите, что можно выбрать
3333 различных трио.
2. Можно ли квадрат разрезать на 17 прямоугольников, у каждого из которых меньшая сторона относится к большей, как 3:5?
3. В коробке лежат 50 синих и 50 красных шариков. Вася хочет набрать 25 синих шариков из коробки. Для этого он вслепую вытаскивает по два шарика. Если он вытащил два
синих, то берет оба. Если синий и красный, то берет синий, а красный кладет обратно в коробку. Наконец, если он вытащил два красных, то оба откладывает в сторону. За какое
наименьшее число вытаскиваний Вася заведомо сможет набрать 25 синих шариков?
4. Найдите все натуральные a, b, c и d, для которых справедливо равенство: a+b+c+d–
3 = ab = cd.
59
5. В треугольнике ABC угол A равен 60. Точка M — середина стороны BC. Докажите,
что AB+BC > 2AM.
6. Найдите все натуральные n, для которых
двух натуральных чисел.
7. Докажите, что a +b +c
2
2
2
4n  2
n5
3(a  b) 2
 ab+bc+ca+
4
равно отношению квадратов
при любых a, b и c.
8. На острове живут три племени: рыцари, которые всегда говорят правду, лжецы, которые всегда лгут, и хитрецы, которые иногда говорят правду, а иногда лгут. За круглым
столом сидят представители этих племен. Каждый из сидящих за столом произнес две фразы: 1) «Слева от меня сидит лжец.» 2) «Справа от меня сидит хитрец.» Докажите, что за этим
столом рыцарей сидит столько же, сколько и лжецов.
МЛАДШАЯ ГРУППА, ПЕРВАЯ ЛИГА
1. В компании 2011 человек. Оказалось, что любые двое имеют хотя бы двух общих
знакомых. Назовем трио любую тройку попарно знакомых человек. Докажите, что можно
выбрать 2011 различных трио.
2. Можно ли квадрат разрезать на 17 прямоугольников, у каждого из которых одна из
сторон в 2,5 раза больше другой?
3. В коробке лежат 10 синих и 10 красных шариков. Вася хочет набрать 5 синих шариков из коробки. Для этого он вслепую вытаскивает по два шарика. Если он вытащил два
синих, то берет оба. Если синий и красный, то берет синий, а красный кладет обратно в коробку. Наконец, если он вытащил два красных, то оба откладывает в сторону. За какое
наименьшее число вытаскиваний Вася заведомо сможет набрать 5 синих шариков?
4. Найдите все натуральные a, b, c и d, для которых справедливо равенство: a+b+c+d–
3 = ab = cd.
5. В остроугольном треугольнике ABC проведена высота BH. На стороне BC отмечена
такая точка K, что KH = KC. А на стороне AB отмечена такая точка L, что KL — биссектриса
угла BKH. Докажите, что AL = LH.
6. Верно ли, что среди любых шести натуральных чисел найдется три числа,
наименьшее общее кратное которых делится на наибольший общий делитель остальных трех
чисел?
2
3(
a

b
)
7. Докажите, что a2+b2+c2  ab+bc+ca+
4
при любых a, b и c.
8. На острове живут три племени: рыцари, которые всегда говорят правду, лжецы, которые всегда лгут, и хитрецы, которые иногда говорят правду, а иногда лгут. За круглым
столом сидят представители этих племен. Каждый из сидящих за столом произнес две фразы: 1) «Слева от меня сидит лжец.» 2) «Справа от меня сидит хитрец.» Докажите, что за этим
столом рыцарей сидит столько же, сколько и лжецов.
60
МЛАДШАЯ ГРУППА, ВТОРАЯ ЛИГА
1. Докажите, что a +b +c
2
2
2
3(a  b) 2
 ab+bc+ca+
4
при любых a, b и c.
2. Найдите все натуральные числа n, квадратный корень из которых равен числу, полученному удалением пяти последних цифр числа n (в десятичной системе счисления).
3. В коробке лежат 10 синих и 10 красных шариков. Вася хочет набрать 5 синих шариков из коробки. Для этого он вслепую вытаскивает по два шарика. Если он вытащил два
синих, то берет оба. Если синий и красный, то берет синий, а красный кладет обратно в коробку. Наконец, если он вытащил два красных, то оба откладывает в сторону. За какое
наименьшее число вытаскиваний Вася заведомо сможет набрать 5 синих шариков?
4. Найдите все четверки различных натуральных чисел a, b, c и d, для которых выполнено равенство a+b+c+d–3 = ab = cd.
5. В остроугольном треугольнике ABC проведена высота BH. На стороне BC отмечена
такая точка K, что KH = KC. А на стороне AB отмечена такая точка L, что KL — биссектриса
угла BKH. Докажите, что AL = LH.
6. В двух коробках лежат конфеты. Если к коробке подходит Вася, то он берет 1/17
часть конфет, имеющихся в данный момент в этой коробке, если Петя — то 1/12, а Коля всегда берет 1/4 часть. Если у берущего конфеты не получается взять ровно столько, сколько
надо, то он воздерживается от сладкого. Вначале в коробках конфет было поровну, по истечении некоторого срока оказалось, что конфет снова поровну. Можно ли утверждать, что
каждый из мальчиков лакомился из первой коробки столько же раз, сколько из второй?
7. Можно ли разрезать квадрат на 17 прямоугольников, у каждого из которых одна из
сторон вдвое длиннее другой?
8. На острове живут три племени: рыцари, которые всегда говорят правду, лжецы, которые всегда лгут, и хитрецы, которые иногда говорят правду, а иногда лгут. За круглым
столом сидят представители этих племен. Каждый из сидящих за столом произнес две фразы: 1) «Слева от меня сидит лжец.» 2) «Справа от меня сидит хитрец.» Докажите, что за этим
столом рыцарей сидит столько же, сколько и лжецов.
ГРУППА «СТАРТ», ВЫСШАЯ ЛИГА
1. Назовем соседними такие клетки квадрата 66, у которых есть общая сторона или
общая вершина. Можно ли так расставить все числа от 1 до 36 в клетках этого квадрата, чтобы разность между числами в любых соседних клетках была не меньше 9?
2. Найдите все натуральные числа n  100000 такие, что если отбросить у n пять последних цифр, а затем полученное число возвести в квадрат, то получится снова n. (Квадрат
числа — это число, умноженное на себя.)
3. В коробке лежат 10 синих и 10 красных шариков. Вася хочет набрать 5 синих шариков из коробки. Для этого он вслепую вытаскивает по два шарика. Если он вытащил два
синих, то берет оба. Если синий и красный, то берет синий, а красный кладет обратно в коробку. Наконец, если он вытащил два красных, то оба откладывает в сторону. За какое
наименьшее число вытаскиваний Вася заведомо сможет набрать 5 синих шариков?
4. Верно ли, что среди любых 100 натуральных чисел найдется число, которое делится
на наибольший общий делитель всех остальных?
61
5. В двух коробках лежат конфеты. Если к коробке подходит Вася, то он берет 1/17
часть конфет, имеющихся в данный момент в этой коробке, если Петя — то 1/12, а Коля всегда берет 1/4 часть. Если у берущего конфеты не получается взять ровно столько, сколько
надо, то он воздерживается от сладкого. Вначале в коробках конфет было поровну, по истечении некоторого срока оказалось, что конфет снова поровну. Можно ли утверждать, что
каждый из мальчиков лакомился из первой коробки столько же раз, сколько из второй?
6. В группе из 25 школьников любые двое имеют общего знакомого. Докажите, что из
этой группы можно не менее, чем 36 способами выбрать пару знакомых школьников.
7. На острове живут три племени: рыцари, которые всегда говорят правду, лжецы, которые всегда лгут, и хитрецы, которые иногда говорят правду, а иногда лгут. За круглым
столом сидят представители этих племен. Каждый из сидящих за столом произнес две фразы: 1) «Слева от меня сидит лжец.» 2) «Справа от меня сидит хитрец.» Докажите, что за этим
столом рыцарей сидит столько же, сколько и лжецов.
8. Докажите, что для любого целого n > 4 квадрат можно разрезать на n прямоугольников, у каждого из которых одна из сторон вдвое длиннее другой.
ГРУППА «СТАРТ», ПЕРВАЯ ЛИГА
1. Назовем соседними такие клетки квадрата 44, у которых есть общая сторона или
общая вершина. Можно ли так расставить все числа от 1 до 16 в клетках этого квадрата, чтобы разность между числами в любых соседних клетках была не меньше 3?
2. Найдите все натуральные числа n  100 такие, что если отбросить у n две последние
цифры, а затем полученное число возвести в квадрат, то получится снова n. (Квадрат числа
— это число, умноженное на себя.)
3. В коробке лежат 10 синих и 10 красных шариков. Вася хочет набрать 5 синих шариков из коробки. Для этого он вслепую вытаскивает по два шарика. Если он вытащил два
синих, то берет оба. Если синий и красный, то берет синий, а красный кладет обратно в коробку. Наконец, если он вытащил два красных, то оба откладывает в сторону. За какое
наименьшее число вытаскиваний Вася заведомо сможет набрать 5 синих шариков?
4. Верно ли, что среди любых четырех натуральных чисел найдется число, которое
делится на наибольший общий делитель всех остальных?
5. В двух коробках лежат конфеты. Если к коробке подходит Вася, то он берет 1/3
часть конфет, имеющихся в данный момент в этой коробке, если Петя — то 1/5. Если у берущего конфеты не получается взять ровно столько, сколько надо, то он воздерживается от
сладкого. Вначале в коробках конфет было поровну, по истечении некоторого срока оказалось, что конфет снова поровну. Можно ли утверждать, что каждый из мальчиков лакомился
из первой коробки столько же раз, сколько из второй?
6. В группе из 7 школьников любые двое имеют общего знакомого. Докажите, что из
этой группы можно не менее, чем 9 способами выбрать пару знакомых школьников.
7. На острове живут три племени: рыцари, которые всегда говорят правду, лжецы, которые всегда лгут, и хитрецы, которые иногда говорят правду, а иногда лгут. За круглым
столом сидят представители этих племен. Каждый из сидящих за столом произнес две фразы: 1) «Слева от меня сидит лжец.» 2) «Справа от меня сидит хитрец.» Докажите, что за этим
столом рыцарей сидит столько же, сколько и лжецов.
8. Можно ли разрезать квадрат на 17 прямоугольников, у каждого из которых одна из
сторон вдвое длиннее другой?
XXXVII УРАЛЬСКИЙ ТУРНИР ЮНЫХ МАТЕМАТИКОВ. КИРОВ, 14-21.02.2011
62
Старшая группа, высшая лига, 2 тур, решения и указания для жюри.
1. В компании 2011 человек. Оказалось, что любые двое имеют хотя бы двух общих
знакомых. Назовем трио любую тройку попарно знакомых. Докажите, что можно выбрать
3333 различных трио.
Решение. Построим граф, вершины которого соответствуют людям, а рёбра — знакомствам. Трио — это треугольник в нашем графе. Пусть в графе e рёбер, t треугольников и
n = 2011 вершин. Мы докажем, что e  (5n–8)/2. Из условия очевидно следует, что каждое
ребро входит хотя бы в два треугольника, поэтому степень каждой вершины хотя бы 3. Если
нет вершин степени 3 и 4, то e  5n/2 и доказываемое утверждение верно.
Пусть a — вершина степени 3, соединённая с вершинами b1, b2, b3, а U — множество
оставшихся n–4 вершин. Каждое из рёбер ab1, ab2, ab3 входит хотя бы в два треугольника,
поэтому вершины a, b1, b2, b3 попарно смежны. Тогда любая вершина из U соединена хотя бы
с двумя из вершин b1, b2, b3, поэтому от вершин b1, b2, b3 отходит хотя бы 2(n–4) рёбер к
вершинам множества U. Следовательно, сумма степеней вершин графа хотя бы 3n+2(n–
4) = 5n–8, откуда следует доказываемое утверждение.
Пусть вершин степени 3 нет, но есть a — вершина степени 4, соединённая с вершинами b1, b2, b3, b4, а U — множество оставшихся n–5 вершин. Тогда любая вершина из U соединена хотя бы с двумя из вершин b1, b2, b3, b4, поэтому от вершин b1, b2 ,b3, b4 отходит хотя бы
2(n–5) рёбер к вершинам множества U. Следовательно, сумма степеней вершин графа хотя
бы
4(n–4)+2(n–5) = 6n–26 > 5n–8, откуда следует доказываемое утверждение.
Итак, e  (5n–8)/2. Так как каждое ребро входит хотя бы в два треугольника, а каждый
треугольник содержит ровно три ребра, t  2e/3  (5n–8)/3 > 3333.
 Получена оценка больше 2000 — 2 балла, больше 3000, но меньше 3325 — 4 балла,
больше 3325, но меньше 3333 — 6 баллов. Показано только, что на каждом ребре есть 2 треугольника — 0 баллов. Показано только, что нет вершин степени 2 — 2 балла.
2. Докажите, что если числа ab, cd и ac+bd делятся на k, то ac и bd тоже делятся
на k (a, b, c, d, k — натуральные числа).
Решение. Достаточно показать, что если p — простое число, и k делится на pn, то ac и
bd тоже делятся на pn. Докажем это. Пусть p входит в разложения на простые множители чисел a, b, c, d в степенях q, r, s, t соответственно. Из условия следует, что q+r  n и s+t  n. Допустим, q+s = r+t. Тогда эти суммы не меньше n (поскольку q+r+s+t  2n), и все доказано.
Пусть q+s > r+t. Тогда ac+bd делится на pr+t, но не делится на pr+t+1. Отсюда, поскольку
ac+bd делится на k, следует, что q+s > r+t  n, и снова все доказано. Случай q+s < r+t аналогичен.
 Пропущен случай q+s = r+t — не более 4 баллов. Доказано только 2ac делится на k
— 6 баллов.
3. Можно ли вырезать из доски 88 восемь клеток так, чтобы из оставшихся клеток
нельзя было вырезать по клеткам прямоугольник площади, не меньшей 8?
Ответ: Нет. Решение. Допустим, вырезать удалось. Тогда в каждой вертикали и каждой горизонтали должно быть вырезано ровно по одной клетке. Разделим каждую вертикаль
и горизонталь на две половинки длины 4. Половинку, из которой вырезана клетка, назовем
отмеченной. Пусть в первой вертикали отмечена верхняя половинка (если нижняя — перевернем доску). Тогда во всех четных вертикалях отмечена верхняя, а в нечетных — нижняя
половинка. Аналогично, можно считать, что во всех четных горизонталях отмечена правая
половинка, а в нечетных — левая. Заметим, что вырезанные клетки обязаны находиться на
пересечениях отмеченных половинок. Возьмем пересечения третьей вертикали с пятой и
63
седьмой горизонталями. Одно из них не вырезано, и в квадрате 33 с центром в нем нет вырезанных клеток.
 Только ответ — 0 баллов. Перебор с хотя бы одной дырой — 0 баллов.
4. Решите уравнение x13  x23  ...xn3   x1  x2  ...  xn  в различных натуральных чис2
лах.
Ответ: (1, 2, …, n) и все перестановки этих чисел. Решение. Индукцией по n легко
убедиться, что числа от 1 до n действительно дают решение уравнения. Далее, не умаляя
общности, будем считать, что x1 < x2 < ... < xn. Теперь, также индукцией по n, докажем, что
2
x13  x23  ...xn3   x1  x2  ...  xn  , причем равенство имеет место только в случае xi = i при всех
i от 1 до n. База n = 1 очевидна. Пусть утверждение доказано для n–1 числа. Заметим, что
x ( x  1)
xn3  xn2  2 xn n n
 2 xn (1  2  ...  ( xn  1))  2 xn ( x1  ...  xn 1 ) , причем последнее неравен2
ство может обратиться в равенство только тогда, когда xi = i при всех i от 1 до n. Прибавляя к
2
полученному неравенству индукционное предположение x13  x23  ...xn31   x1  x2  ...  xn 1  ,
получаем требуемое неравенство, причем в равенство оно может обратиться только в вышеуказанном случае.
 Только ответ — 2 балла. Нет проверки ответа — дыра в 2 балла.
5. Точки K и L расположены на сторонах AD и BC выпуклого четырехугольника
ABCD соответственно так, что AK/KD = CL/LB. Прямая KL пересекает отрезки AC и BD в
точках P и Q соответственно. Докажите, что KP/QL = SACD/SBCD.
Решение. Пусть KP/QL = k, а R — такая точка на стороне CD, что CR/RD = k. Тогда,
как легко видеть, LR параллельна BD, а KR параллельна AC. Заметим, что KP/QL = SKPR/SQLR.
Покажем, что SBCDk(1–k) = SQLR, а SACD k(1–k) = SKPR, из этого будет следовать утверждение
задачи. Поскольку, LR параллельна BD, SQLR = SBLR. Далее, SBLR = (1–k)SBCR = (1–k)kSBCD.
Другое равенство доказывается аналогично.
 Не разобран один из случаев расположения точек P и Q — дыра не более 2 баллов.
6. Докажите, что a2+b2+c2  ab+bc+ca+
Решение.
a  c
2
2

c  b
2
Исходное
2

a  b
3(a  b) 2
при любых a, b и c.
4
неравенство
нетрудно
привести
к
виду
2
4
. Полагая u = a–c, v = c–b, приводим последнее неравенство к оче-
u 2 v2 u  v 
 
видному
.
2 2
4
2
7. Внутри треугольника ABC (AB < BC) лежит точка O,
равноудаленная от трех его вершин. BD — биссектриса угла B.
Точка M — середина стороны AC, а точка P на луче MO такова,
что APC = ABC. Точка N — основание перпендикуляра, опущенного из P на BC. Докажите, что каждая из диагоналей четырехугольника BDMN делит треугольник ABC на две равновеликие
части.
Решение. Для решения задачи достаточно доказать, что
2CNCD = ACCB. Поскольку CAB+BCA = CAP+PCA, имеем BAP = BCP. Пусть K — проекция точки P на прямую AB.
64
Заметим, что прямоугольные треугольники AKP и NPC равны по гипотенузе и острому углу.
Поэтому KP = NP, а CN = KA. Кроме того, нетрудно видеть, что KB = NB, следовательно
CN = AB+BN. То есть 2CN = AB+BC. Далее, поскольку BD — биссектриса угла B, имеем
CD = BC/(AB+BC). Таким образом, 2CNCD = (AB+BC)(ACBC/(AB+BC)) = ACCB.
 Разобрана только диагональ BM — 2 балла. Разобрана только диагональ DN — 6
баллов.
8. Дано натуральное число N. Два игрока по очереди делают такие ходы: A пишет на
доске число 1 или –1, B прибавляет к этому числу 2 или вычитает из него 2, потом A прибавляет или вычитает 3 и т.д. (на k-м ходу игрок прибавляет k или вычитает k). Игрок, после хода которого число впервые будет делиться на N, выигрывает. Для каждого N, большего 10, определите, есть ли у одного из игроков выигрышная стратегия, и если есть, то у какого.
Ответ: Выигрышной стратегии нет ни у кого ни при каких N > 10. Решение. Действительно, пусть игрок (назовем его X), у которого есть выигрышная стратегия, впервые получает кратное N после k–го хода, а после k–2–го получил число a. Тогда его противник Y мог получить любое из чисел a+k–1 и a+1–k, Поскольку X должен выиграть, у него есть выигрывающий ответ на любой из этих ходов. Это значит, что и среди чисел a+2k–1, a–1, и среди чисел a–2k+1, a+1 есть число, кратное N. Поскольку числа a–1 и a+1 оба делиться на N не могут, у N есть кратные, отличающиеся либо на 2k–2, либо на 4k–2. Разберем эти две возможности отдельно.
1. Пусть 2k–2 делится на N. Тогда число, получаемое после k–2-го хода, отличается от
кратного N на 1. Если после k–4-го хода получилось число b, игрок Y мог получить любое из
чисел b+k–3 и b+3–k, и в каждом из этих случаев X мог получить число, сравнимое с 1 (mod
N). Значит, в каждой из пар b+2k–5  b–3 (mod N), b–1 и b–2k+5  b+3 (mod N), b+1 есть число, сравнимое с 1 (mod N). Поскольку N > 10, это возможно только при b, кратном N — то
есть кратное N получилось уже после k–4-го хода, что противоречит нашему предположению.
2. Пусть 4k–2 делится на N. Тогда число, получаемое после k–2-го хода, сравнимо с
2k–1 (mod N), но не делится на N. Если после k–4-го хода получилось число b, то в каждой из
пар b+2k–5, b–1 и b–2k+5, b+1 есть число, сравнимое с 2k–1 (mod N), что, как легко видеть,
невозможно.
 Задача решена для все достаточно больших N — 6 баллов.
Старшая группа, первая лига, 2 тур, решения и указания для жюри.
1. В компании 2011 человек. Оказалось, что любые двое имеют хотя бы двух общих
знакомых. Назовем трио любую тройку попарно знакомых. Докажите, что можно выбрать
3333 различных трио.
Решение. Построим граф, вершины которого соответствуют людям, а рёбра — знакомствам. Трио — это треугольник в нашем графе. Пусть в графе e рёбер, t треугольников и
n = 2011 вершин. Мы докажем, что e  (5n–8)/2. Из условия очевидно следует, что каждое
ребро входит хотя бы в два треугольника, поэтому степень каждой вершины хотя бы 3. Если
нет вершин степени 3 и 4, то e  5n/2 и доказываемое утверждение верно.
Пусть a — вершина степени 3, соединённая с вершинами b1, b2, b3, а U — множество
оставшихся n–4 вершин. Каждое из рёбер ab1, ab2, ab3 входит хотя бы в два треугольника,
поэтому вершины a, b1, b2, b3 попарно смежны. Тогда любая вершина из U соединена хотя бы
с двумя из вершин b1, b2, b3, поэтому от вершин b1, b2, b3 отходит хотя бы 2(n–4) рёбер к
вершинам множества U. Следовательно, сумма степеней вершин графа хотя бы 3n+2(n–
4) = 5n–8, откуда следует доказываемое утверждение.
Пусть вершин степени 3 нет, но есть a — вершина степени 4, соединённая с вершинами b1, b2, b3, b4, а U — множество оставшихся n–5 вершин. Тогда любая вершина из U соеди-
65
нена хотя бы с двумя из вершин b1, b2, b3, b4, поэтому от вершин b1, b2 ,b3, b4 отходит хотя бы
2(n–5) рёбер к вершинам множества U. Следовательно, сумма степеней вершин графа хотя
бы
4(n–4)+2(n–5) = 6n–26 > 5n–8, откуда следует доказываемое утверждение.
Итак, e  (5n–8)/2. Так как каждое ребро входит хотя бы в два треугольника, а каждый
треугольник содержит ровно три ребра, t  2e/3  (5n–8)/3 > 3333.
 Получена оценка больше 2000 — 2 балла, больше 3000, но меньше 3325 — 6 баллов, больше 3325, но меньше 3333 — 8 баллов. Показано только, что на каждом ребре есть 2
треугольника — 0 баллов. Показано только, что нет вершин степени 2 — 2 балла.
2. Докажите, что если числа ab, cd и ac+bd делятся на k, то ac и bd тоже делятся
на k (a, b, c, d, k — натуральные числа).
Решение. Достаточно показать, что если p — простое число, и k делится на pn, то ac и
bd тоже делятся на pn. Докажем это. Пусть p входит в разложения на простые множители чисел a, b, c, d в степенях q, r, s, t соответственно. Из условия следует, что q+r  n и s+t  n. Допустим, q+s = r+t. Тогда эти суммы не меньше n (поскольку q+r+s+t  2n), и все доказано.
Пусть q+s > r+t. Тогда ac+bd делится на pr+t, но не делится на pr+t+1. Отсюда, поскольку
ac+bd делится на k, следует, что q+s > r+t  n, и снова все доказано. Случай q+s < r+t аналогичен.
 Пропущен случай q+s = r+t — не более 4 баллов. Доказано только 2ac делится на k
— 6 баллов.
3. Можно ли вырезать из доски 88 восемь клеток так, чтобы из оставшихся клеток
нельзя было вырезать по клеткам прямоугольник площади, не меньшей 8?
Ответ: Нет. Решение. Допустим, вырезать удалось. Тогда в каждой вертикали и каждой горизонтали должно быть вырезано ровно по одной клетке. Разделим каждую вертикаль
и горизонталь на два половинки длины 4. Половинку, из которой вырезана клетка, назовем
отмеченной. Пусть в первой вертикали отмечена верхняя половинка (если нижняя — перевернем доску). Тогда во всех четных вертикалях отмечена верхняя, а в нечетных — нижняя
половинка. Аналогично, можно считать, что во всех четных горизонталях отмечена правая
половинка, а в нечетных — левая. Заметим, что вырезанные клетки обязаны находиться на
пересечениях отмеченных половинок. Возьмем пересечения третьей вертикали с пятой и
седьмой горизонталями. Одно из них не вырезано, и в квадрате 33 с центром в нем нет вырезанных клеток.
 Только ответ — 0 баллов. Перебор с хотя бы одной дырой — 0 баллов.
4. Решите уравнение x3  y 3  z 3  t 3   x  y  z  t  в различных натуральных числах.
2
Ответ: (1, 2, 3, 4) и всевозможные его перестановки. Решение. См. решение более общей задачи 4 высшей лиги.
 Только ответ — 0 баллов.
5. Через вершину C равнобедренного треугольника ABC
(AB = AC) с острым углом при вершине A провели перпендикуляр к BC
и на этом перпендикуляре отметили точку P, лежащую с той же
стороны от прямой BC, что и A, и с той же стороны от прямой AB,
что и C. Точка D такова, что ABPD — параллелограмм. M — точка
пересечения прямой PC и отрезка AD. Найдите отношение DM/DA.
Ответ: 1/2. Решение. Пусть H — середина BC, а Q — точка пересечения AH и BP. Тогда AH параллельно CP. Поэтому по теореме
Фалеса BQ = QP. AMPQ — параллелограмм, следовательно, AM = QP.
66
Но, поскольку BP = AD, получаем, что AM = MD.
6. Докажите, что a2+b2+c2  ab+bc+ca+
Решение.
a  c
2
2

c  b
2
Исходное
2

a  b
3(a  b) 2
при любых a, b и c.
4
неравенство
нетрудно
привести
к
виду
2
4
. Полагая u = a–c, v = c–b, приводим последнее неравенство к оче-
u 2 v2 u  v 
видному
.
 
2 2
4
2
7. Внутри треугольника ABC (AB < BC) лежит точка O,
равноудаленная от трех его вершин. BD — биссектриса угла B.
Точка M — середина стороны AC, а точка P на луче MO такова,
что APC = ABC. Точка N — основание перпендикуляра, опущенного из P на BC. Докажите, что каждая из диагоналей четырехугольника BDMN делит треугольник ABC на две равновеликие
части.
Решение. Для решения задачи достаточно доказать, что
2CNCD = ACCB. Поскольку CAB+BCA = CAP+PCA, имеем BAP = BCP. Пусть K — проекция точки P на прямую AB.
Заметим, что прямоугольные треугольники AKP и NPC равны по гипотенузе и острому углу.
Поэтому KP = NP, а CN = KA. Кроме того, нетрудно видеть, что KB = NB, следовательно
CN = AB+BN. То есть 2CN = AB+BC. Далее, поскольку BD — биссектриса угла B, имеем
CD = BC/(AB+BC). Таким образом, 2CNCD = (AB+BC)(ACBC/(AB+BC)) = ACCB.
 Разобрана только диагональ BM — 2 балла. Разобрана только диагональ DN — 6
баллов.
8. В коробке лежат 50 синих и 50 красных шариков. Вася хочет набрать 25 синих
шариков из коробки. Для этого он вслепую вытаскивает по два шарика. Если он вытащил
два синих, то берет оба. Если синий и красный, то берет синий, а красный кладет обратно
в коробку. Наконец, если он вытащил два красных, то оба откладывает в сторону. За какое
наименьшее число вытаскиваний Вася заведомо сможет набрать 25 синих шариков?
Ответ: 50. Решение. Пример. Первые 24 раза вытаскиваются красный и синий, потом
25 раз подряд по два красных, и в конце — два синих. Оценка. По два красных шарика можно вытащить не больше 25 раз. Все остальные вытаскивания добавляют хотя бы по одному
синему шарику, поэтому их понадобится не больше 25.
 Только оценка или только пример — 4 балла. Только ответ — 0 баллов.
Старшая группа, вторая лига, 2 тур, решения и указания для жюри.
1. Дорогу разделили на 2011 частей (не обязательно равной длины). Яна знает длину
дороги. Ей разрешено спросить, чему равно расстояние между серединами любых двух частей, при этом количество вопросов неограничено. Длину каких частей она сможет
узнать?
Ответ: Всех. Решение. Найдем расстояние между серединами первой и второй частей
и удвоим его. Это сумма их длин. Аналогично найдем суммы дли третьей и четвертой, …,
2009-й и 2010-й частей. Вычитая все эти суммы из длины дороги, найдем длину 2011-й части. Аналогично найдем длину первой части. Удалим эти крайние части из дороги и, повторив процедуру, найдем длины второй и 2010-й частей. Повторяя процедуру дальше, последовательно найдем длины всех частей, кроме средней, 1006-й. Длину средней части найдем,
вычитая из длины дороги сумму длин всех остальных частей.
67
2. Докажите, что если числа ab, cd и ac+bd делятся на k, то ac и bd тоже делятся
на k (a, b, c, d, k — натуральные числа).
Решение. Достаточно показать, что если p — простое число, и k делится на pn, то ac и
bd тоже делятся на pn. Докажем это. Пусть p входит в разложения на простые множители чисел a, b, c, d в степенях q, r, s, t соответственно. Из условия следует, что q+r  n и s+t  n. Допустим, q+s = r+t. Тогда эти суммы не меньше n (поскольку q+r+s+t  2n), и все доказано.
Пусть q+s > r+t. Тогда ac+bd делится на pr+t, но не делится на pr+t+1. Отсюда, поскольку
ac+bd делится на k, следует, что q+s > r+t  n, и снова все доказано. Случай q+s < r+t аналогичен.
 Пропущен случай q+s = r+t — не более 4 баллов. Доказано только 2ac делится на k
— 6 баллов.
3. Можно ли вырезать из доски 88 восемь клеток так, чтобы из оставшихся клеток
нельзя было вырезать по клеткам прямоугольник площади, не меньшей 8?
Ответ: Нет. Решение. Допустим, вырезать удалось. Тогда в каждой вертикали и каждой горизонтали должно быть вырезано ровно по одной клетке. Разделим каждую вертикаль
и горизонталь на два половинки длины 4. Половинку, из которой вырезана клетка, назовем
отмеченной. Пусть в первой вертикали отмечена верхняя половинка (если нижняя — перевернем доску). Тогда во всех четных вертикалях отмечена верхняя, а в нечетных — нижняя
половинка. Аналогично, можно считать, что во всех четных горизонталях отмечена правая
половинка, а в нечетных — левая. Заметим, что вырезанные клетки обязаны находиться на
пересечениях отмеченных половинок. Возьмем пересечения третьей вертикали с пятой и
седьмой горизонталями. Одно из них не вырезано, и в квадрате 33 с центром в нем нет вырезанных клеток.
 Только ответ — 0 баллов. Перебор с хотя бы одной дырой — 0 баллов.
4. Решите уравнение x3+y3 = (x+y)2 в различных натуральных числах.
Ответ: (1, 2) и (2, 1). Решение. Сокращая на x+y, получаем x2–xy+y2 = x+y. Пусть x > y.
Тогда
x2–xy = x(x–y)  x, а y2  y, причем оба неравенства обращаются в равенства только при y = 1,
x = 2.
5. Через вершину C равнобедренного треугольника ABC
(AB = AC) с острым углом при вершине A провели перпендикуляр к BC
и на этом перпендикуляре отметили точку P, лежащую с той же
стороны от прямой BC, что и A, и с той же стороны от прямой AB,
что и C. Точка D такова, что ABPD — параллелограмм. M — точка
пересечения прямой PC и отрезка AD. Найдите отношение DM/DA.
Ответ: 1/2. Решение. Пусть H — середина BC, а Q — точка пересечения AH и BP. Тогда AH параллельно CP. Поэтому по теореме
Фалеса BQ = QP. AMPQ — параллелограмм, следовательно, AM = QP.
Но, поскольку BP = AD, получаем, что AM = MD.
6. В двух коробках лежат конфеты. Если к коробке подходит Вася, то он берет 1/17
часть конфет, имеющихся в данный момент в этой коробке, если Петя — то 1/12, а Коля
всегда берет 1/4 часть. Если у берущего конфеты не получается взять ровно столько,
сколько надо, то он воздерживается от сладкого. Вначале в коробках конфет было поровну,
по истечении некоторого срока оказалось, что конфет снова поровну. Можно ли утверждать, что каждый из мальчиков лакомился из первой коробки столько же раз, сколько из
второй?
68
Ответ: Можно. Решение. Если конфеты брал Вася, в коробке остается 16/17, если Петя — 11/12, если Коля — 3/4 от их предыдущего количества. Если Вася, Петя и Коля брали
конфеты из данной коробки несколько раз, их первоначальное количество умножается на
16/17, 11/12 и 3/4 соответствующее количество раз. Если сначала конфет в обеих коробках
было поровну, и в конце — тоже, то дроби, на которые умножалось количество конфет в
первой и второй коробках, также равны. Но тогда у них в несократимой записи в числителях
должно быть поровну множителей 11 и поровну множителей 3, а в знаменателях — поровну
множителей 17. А так как всем этим множителям не с чем сокращаться, то их было поровну
и в исходной записи, откуда и следует справедливость утверждения задачи.
7. Внутри угла отмечена точка P, не лежащая на его биссектрисе. Отрезки AB и CD
проходят через точку P, при этом A и C лежат на одной стороне угла, B и D на другой, отрезок CD перпендикулярен биссектрисе угла, а отрезок AB делится точкой P пополам. Докажите, что AB > CD.
Решение. Выберем на сторонах угла такие точки A1 и B1, что CD||AA1||BB1. Тогда
AA1BB1 будет равнобокой трапецией, а CD — её средней линией. Заметим, что
2CD = AA1+BB1, а AB = A1B1. Осталось заметить, что AB+A1B1 > AA1+BB1, поскольку сумма
длин диагоналей всегда больше суммы длин противоположных сторон.
8. В коробке лежат 50 синих и 50 красных шариков. Вася хочет набрать 25 синих
шариков из коробки. Для этого он вслепую вытаскивает по два шарика. Если он вытащил
два синих, то берет оба. Если синий и красный, то берет синий, а красный кладет обратно
в коробку. Наконец, если он вытащил два красных, то оба откладывает в сторону. За какое
наименьшее число вытаскиваний Вася заведомо сможет набрать 25 синих шариков?
Ответ: 50. Решение. Пример. Первые 24 раза вытаскиваются красный и синий, потом
25 раз подряд по два красных, и в конце — два синих. Оценка. По два красных шарика можно вытащить не больше 25 раз. Все остальные вытаскивания добавляют хотя бы по одному
синему шарику, поэтому их понадобится не больше 25.
Младшая группа, высшая лига, 2 тур, решения и указания для жюри.
1. В компании 2011 человек. Оказалось, что любые двое имеют хотя бы двух общих
знакомых. Назовем трио любую тройку попарно знакомых человек. Докажите, что можно
выбрать 3333 различных трио.
Решение. Построим граф, вершины которого соответствуют людям, а рёбра — знакомствам. Трио — это треугольник в нашем графе. Пусть в графе e рёбер, t треугольников и
n = 2011 вершин. Мы докажем, что e  (5n–8)/2. Из условия очевидно следует, что каждое
ребро входит хотя бы в два треугольника, поэтому степень каждой вершины хотя бы 3. Если
нет вершин степени 3 и 4, то e  5n/2 и доказываемое утверждение верно.
Пусть a — вершина степени 3, соединённая с вершинами b1, b2, b3, а U — множество
оставшихся n–4 вершин. Каждое из рёбер ab1, ab2, ab3 входит хотя бы в два треугольника,
поэтому вершины a, b1, b2, b3 попарно смежны. Тогда любая вершина из U соединена хотя бы
с двумя из вершин b1, b2, b3, поэтому от вершин b1, b2, b3 отходит хотя бы 2(n–4) рёбер к
вершинам множества U. Следовательно, сумма степеней вершин графа хотя бы 3n+2(n–
4) = 5n–8, откуда следует доказываемое утверждение.
Пусть вершин степени 3 нет, но есть a — вершина степени 4, соединённая с вершинами b1, b2, b3, b4, а U — множество оставшихся n–5 вершин. Тогда любая вершина из U соединена хотя бы с двумя из вершин b1, b2, b3, b4, поэтому от вершин b1, b2 ,b3, b4 отходит хотя бы
2(n–5) рёбер к вершинам множества U. Следовательно, сумма степеней вершин графа хотя
бы
4(n–4)+2(n–5) = 6n–26 > 5n–8, откуда следует доказываемое утверждение.
Итак, e  (5n–8)/2. Так как каждое ребро входит хотя бы в два треугольника, а каждый
треугольник содержит ровно три ребра, t  2e/3  (5n–8)/3 > 3333.
69
 Только оценка 2011 — 2 балла. Разобран только один из двух случаев: есть вершины степени 3 и все вершины степени не менее 4 — 4 балла.
2. Можно ли квадрат разрезать на 17 прямоугольников, у каждого из которых
меньшая сторона относится к большей, как 3:5?
Ответ: Можно. Решение. Пример будет нарисован на доске.
3. В коробке лежат 50 синих и 50 красных шариков. Вася хочет набрать 25 синих
шариков из коробки. Для этого он вслепую вытаскивает по два шарика. Если он вытащил
два синих, то берет оба. Если синий и красный, то берет синий, а красный кладет обратно
в коробку. Наконец, если он вытащил два красных, то оба откладывает в сторону. За какое
наименьшее число вытаскиваний Вася заведомо сможет набрать 25 синих шариков?
Ответ: 50. Решение. Пример. Первые 24 раза вытаскиваются красный и синий, потом
25 раз подряд по два красных, и в конце — два синих. Оценка. По два красных шарика можно вытащить не больше 25 раз. Все остальные вытаскивания добавляют хотя бы по одному
синему шарику, поэтому их понадобится не больше 25.
 Только оценка — 4 балла. Только пример — 4 балла.
4. Найдите все натуральные a, b, c и d, для которых справедливо равенство:
a+b+c+d–3 = ab = cd.
Ответ: (1, 1, 1, 1), (3, 3, 3, 3), (6, 2, 3, 4) и все четверки, получаемые из последней операциями перестановки двух первых чисел, двух последних чисел и первой пары чисел со второй.
Решение. Не умаляя общности будем считать, что a — наибольшее из данных чисел. Тогда
a+b+c+d–3 < 4a, откуда b ≤ 3. Допустим, b = 3. Тогда a+c+d = 3a, откуда c = d = a = 3. Допустим, b = 2. Тогда a+c+d = 2a+1 = cd+1, откуда, как легко проверить, следует, что (c–2)(d–
2) = 2. Поскольку обе скобки в левой части последнего равенства больше –2, имеем c–2 = 1,
d–2 = 2 или c–2 = 2, d–2 = 1. Это дает решения (6, 2, 3, 4) и (6, 2, 5, 4). Решения, где наибольшим является не a, а другое из данных чисел, получаются из него описанными выше перестановками. Наконец, пусть b = 1. Тогда, как легко показать, все числа равны 1.
 Только ответы (все) — 2 балла. Потерян ответ (1, 1, 1, 1) — дыра в 4 балла. Любая
другая потеря ответа — дыра не меньше, чем в 6 баллов.
5. В треугольнике ABC угол A равен 60. Точка M — середина стороны BC. Докажите, что AB+BC > 2AM.
Решение. Рассмотрим высоту BH. Треугольник ABH — прямоугольный с углом 60
при вершине A. Поэтому AH = AB/2. Далее, HM = BC/2 как медиана в прямоугольном треугольнике. Наконец, AM < AH+HM = (AB+BC)/2.
6. Найдите все натуральные n, для которых
4n  2
равно отношению квадратов двух
n5
натуральных чисел.
Ответ: 13. Решение. Если n четно, то n+5 нечетно, а 4n–2 делится на 2, но не делится
на 4. В этом случае дробь из условия не может равняться отношению двух квадратов. Поэтому можно считать, что n = 2m–1 при некотором натуральном m. Тогда дробь из условия после сокращения на 2 принимает вид (4m–3)/(m+2). Заметим, что 4(m+2)–(4m–3) = 11. Поэтому
НОД(4m–3, m+2) может равняться только 1 или 11. В первом случае имеем 4m–3 = a2,
m+2 = b2, откуда 4b2–a2 = 11. Решая в натуральных числах уравнение (2b–a)(2b+a) = 11, получаем b = 3, a = 5, m = b2–2 = 7, n = 13. Во втором случае имеем 4m–3 = 11a2, m+2 = 11b2,
откуда получаем не имеющее решений в натуральных числах уравнение 4b2–a2 = 1.
 Только ответ — 0 баллов.
70
7. Докажите, что a2+b2+c2  ab+bc+ca+
Решение.
a  c
2
2

c  b
2
Исходное
2

a  b
3(a  b) 2
при любых a, b и c.
4
неравенство
нетрудно
привести
к
виду
2
4
. Полагая u = a–c, v = c–b, приводим последнее неравенство к оче-
u 2 v2 u  v 
видному
.
 
2 2
4
2
8. На острове живут три племени: рыцари, которые всегда говорят правду, лжецы,
которые всегда лгут и хитрецы, которые иногда говорят правду, а иногда лгут. За круглым
столом сидят представители этих племен. Каждый из сидящих за столом произнес две
фразы: 1) «Слева от меня сидит лжец.» 2) «Справа от меня сидит хитрец.» Докажите,
что за этим столом рыцарей сидит столько же, сколько и лжецов.
Решение. Из условия следует, что слева от каждого рыцаря сидит лжец. Справа же от
лжеца не могут сидеть ни хитрец (тогда этот лжец сказал бы про своего правого соседа правду), ни лжец (тогда тот сказал бы правду про своего левого соседа). Поэтому правым соседом
лжеца может быть только рыцарь. Таким образом, все рыцари и лжецы разбиваются на пары,
где лжец — левый сосед рыцаря, откуда и вытекает утверждение задачи.
Младшая группа, первая лига, 2 тур, решения и указания для жюри.
1. В компании 2011 человек. Оказалось, что любые двое имеют хотя бы двух общих
знакомых. Назовем трио любую тройку попарно знакомых человек. Докажите, что можно
выбрать 2011 различных трио.
Решение. Построим граф, вершины которого соответствуют людям, а рёбра — знакомствам. Трио — это треугольник в нашем графе. Пусть в графе e рёбер и t треугольников и
n = 2011 вершин. Из условия очевидно следует, что каждое ребро входит хотя бы в два треугольника, поэтому степень каждой вершины хотя бы 3. Следовательно, e  3n/2. Так как
каждое ребро входит хотя бы в два треугольника, а каждый треугольник содержит ровно три
ребра, t  2e/3  n = 2011.
2. Можно ли квадрат разрезать на 17 прямоугольников, у каждого из которых одна
из сторон в 2,5 раза больше другой?
Ответ: Можно. Решение. Существуют разрезания квадрата на 4 прямоугольника с отношением сторон 2:5. Прямоугольник же с отношением сторон 2:5 нетрудно разрезать как на
4, так и на 6 прямоугольников с тем же отношением сторон. Таким образом мы можем увеличивать число прямоугольников в наших разрезаниях на 3 и на 5. 17 = 4+3+5+5.
3. В коробке лежат 10 синих и 10 красных шариков. Вася хочет набрать 5 синих шариков из коробки. Для этого он вслепую вытаскивает по два шарика. Если он вытащил два
синих, то берет оба. Если синий и красный, то берет синий, а красный кладет обратно в
коробку. Наконец, если он вытащил два красных, то оба откладывает в сторону. За какое
наименьшее число вытаскиваний Вася заведомо сможет набрать 5 синих шариков?
Ответ: 10. Решение. Пример. Первые 4 раза вытаскиваются красный и синий, потом
пять раз подряд по два красных, и в конце — два синих. Оценка. По два красных шарика
можно вытащить не больше 5 раз. Все остальные вытаскивания добавляют хотя бы по одному синему шарику, поэтому их понадобится не больше пяти.
 Только оценка — 4 балла. Только пример — 4 балла.
4. Найдите все натуральные a, b, c и d, для которых справедливо равенство:
a+b+c+d–3 = ab = cd.
71
Ответ: (1, 1, 1, 1), (3, 3, 3, 3), (6, 2, 3, 4) и все четверки, получаемые из последней операциями перестановки двух первых чисел, двух последних чисел и первой пары чисел со второй.
Решение. Не умаляя общности будем считать, что a — наибольшее из данных чисел. Тогда
a+b+c+d–3 < 4a, откуда b ≤ 3. Допустим, b = 3. Тогда a+c+d = 3a, откуда c = d = a = 3. Допустим, b = 2. Тогда a+c+d = 2a+1 = cd+1, откуда, как легко проверить, следует, что (c–2)(d–
2) = 2. Поскольку обе скобки в левой части последнего равенства больше –2, имеем c–2 = 1,
d–2 = 2 или c–2 = 2, d–2 = 1. Это дает решения (6, 2, 3, 4) и (6, 2, 5, 4). Решения, где наибольшим является не a, а другое из данных чисел, получаются из него описанными выше перестановками. Наконец, пусть b = 1. Тогда, как легко показать, все числа равны 1.
 Только ответы (все) — 2 балла. Потерян ответ (1, 1, 1, 1) — дыра в 4 балла. Любая
другая потеря ответа — дыра не меньше, чем в 6 баллов.
5. В остроугольном треугольнике ABC проведена высота BH. На стороне BC отмечена такая точка K, что KH = KC. А на стороне AB отмечена такая точка L, что KL —
биссектриса угла BKH. Докажите, что AL = LH.
Решение. Поскольку KH = KC, точка K лежит на серединном перпендикуляре к отрезку HC. Поскольку этот перпендикуляр параллелен высоте BH, точка K — середина стороны
BC. Поэтому HK = BK (как медиана из вершины прямого угла прямоугольного треугольника
BHC). Но это значит, что биссектриса KL является высотой в равнобедренном треугольнике
BKH, откуда KL||AC. Следовательно, L — середина AB и потому лежит на серединном перпендикуляре отрезка AH, откуда и следует, что AL = LH.
6. Верно ли, что среди любых шести натуральных чисел найдется три числа,
наименьшее общее кратное которых делится на наибольший общий делитель остальных
трех чисел?
Ответ: Нет. Решение. Возьмём шесть разных букв и запишем всевозможные наборы,
состоящие из трех различных букв из этих шести. Каждому набору сопоставим простое число так, чтобы все эти простые числа были различны. Затем составим 6 чисел, перемножив
для каждой буквы все простые числа, сопоставленные наборам, куда входит эта буква. Тогда
НОД любых трех из этих чисел будет равен простому числу, сопоставленному набору из
трех соответствующих букв, а три других числа из построенных нами шести на это простое
число не делятся, стало быть, на него не делится и их НОК.
3(a  b) 2
7. Докажите, что a +b +c  ab+bc+ca+
при любых a, b и c.
4
2
Решение.
a  c
2
2

c  b
2
2
Исходное
2

a  b
2
неравенство
нетрудно
привести
к
виду
2
4
. Полагая u = a–c, v = c–b, приводим последнее неравенство к оче-
u 2 v2 u  v 
 
видному
.
2 2
4
2
8. На острове живут три племени: рыцари, которые всегда говорят правду, лжецы,
которые всегда лгут и хитрецы, которые иногда говорят правду, а иногда лгут. За круглым
столом сидят представители этих племен. Каждый из сидящих за столом произнес две
фразы: 1) «Слева от меня сидит лжец.» 2) «Справа от меня сидит хитрец.» Докажите,
что за этим столом рыцарей сидит столько же, сколько и лжецов.
Решение. Из условия следует, что слева от каждого рыцаря сидит лжец. Справа же от
лжеца не могут сидеть ни хитрец (тогда этот лжец сказал бы про своего правого соседа правду), ни лжец (тогда тот сказал бы правду про своего левого соседа). Поэтому правым соседом
лжеца может быть только рыцарь. Таким образом, все рыцари и лжецы разбиваются на пары,
где лжец — левый сосед рыцаря, откуда и вытекает утверждение задачи.
72
Младшая группа, вторая лига, 2 тур, решения и указания для жюри.
1. Докажите, что a2+b2+c2  ab+bc+ca+
Решение.
a  c
2
2

c  b
2
Исходное
2

a  b
3(a  b) 2
при любых a, b и c.
4
неравенство
нетрудно
привести
к
виду
2
4
. Полагая u = a–c, v = c–b, приводим последнее неравенство к оче-
u 2 v2 u  v 
видному
.
 
2 2
4
2
2. Найдите все натуральные числа n, квадратный корень из которых равен числу, полученному удалением пяти последних цифр числа n (в десятичной системе счисления).
Ответ: 10000000000. Решение. Пусть a — число, которое получается отбрасыванием у
числа n пяти последних цифр. По условию a2 = 100000a+b, где b < 100000. Поделив на a, получаем a = 100000+b/a. Поскольку a — целое и не меньше 100000, то 0 ≤ b/a < 1. Поэтому
равенство a = 100000+b/a возможно только при b = 0, что и дает ответ.
 Только ответ — 0 баллов.
3. В коробке лежат 10 синих и 10 красных шариков. Вася хочет набрать 5 синих шариков из коробки. Для этого он вслепую вытаскивает по два шарика. Если он вытащил два
синих, то берет оба. Если синий и красный, то берет синий, а красный кладет обратно в
коробку. Наконец, если он вытащил два красных, то оба откладывает в сторону. За какое
наименьшее число вытаскиваний Вася заведомо сможет набрать 5 синих шариков?
Ответ: 10. Решение. Пример. Первые 4 раза вытаскиваются красный и синий, потом
пять раз подряд по два красных, и в конце — два синих. Оценка. По два красных шарика
можно вытащить не больше 5 раз. Все остальные вытаскивания добавляют хотя бы по одному синему шарику, поэтому их понадобится не больше пяти.
 Только оценка — 4 балла. Только пример — 4 балла.
4. Найдите все четверки различных натуральных чисел a, b, c и d, для которых выполнено равенство a+b+c+d–3 = ab = cd.
Ответ: (6, 2, 3, 4) и все четверки, получаемые из последней операциями перестановки
двух первых чисел, двух последних чисел и первой пары чисел со второй. Решение. Не умаляя
общности будем считать, что a — наибольшее из данных чисел. Тогда a+b+c+d–3 < 4a, откуда b ≤ 3. Допустим, b = 3. Тогда a+c+d = 3a, откуда c = d = a = 3 — не подходит, есть одинаковые числа. Допустим, b = 2. Тогда a+c+d = 2a+1 = cd+1, откуда, как легко проверить, следует, что (c–2)(d–2) = 2. Поскольку обе скобки в левой части последнего равенства больше –
2, имеем c–2 = 1, d–2 = 2 или c–2 = 2, d–2 = 1. Это дает решения (6, 2, 3, 4) и (6, 2, 5, 4). Решения, где наибольшим является не a, а другое из данных чисел, получаются из него описанными
выше перестановками. Наконец, пусть b = 1. Тогда, как легко показать, все числа равны 1 — не
подходит, есть одинаковые числа.
 Только ответы (все) — 2 балла. Любая потеря ответа — дыра не меньше, чем в 6 баллов.
5. В остроугольном треугольнике ABC проведена высота BH. На стороне BC отмечена такая точка K, что KH = KC. А на стороне AB отмечена такая точка L, что KL —
биссектриса угла BKH. Докажите, что AL = LH.
Решение. Поскольку KH = KC, точка K лежит на серединном перпендикуляре к отрезку HC. Поскольку этот перпендикуляр параллелен высоте BH, точка K — середина стороны
73
BC. Поэтому HK = BK (как медиана из вершины прямого угла прямоугольного треугольника
BHC). Но это значит, что биссектриса KL является высотой в равнобедренном треугольнике
BKH, откуда KL||AC. Следовательно, L — середина AB и потому лежит на серединном перпендикуляре отрезка AH, откуда и следует, что AL = LH.
6. В двух коробках лежат конфеты. Если к коробке подходит Вася, то он берет 1/17
часть конфет, имеющихся в данный момент в этой коробке, если Петя — то 1/12, а Коля
всегда берет 1/4 часть. Если у берущего конфеты не получается взять ровно столько,
сколько надо, то он воздерживается от сладкого. Вначале в коробках конфет было поровну,
по истечении некоторого срока оказалось, что конфет снова поровну. Можно ли утверждать, что каждый из мальчиков лакомился из первой коробки столько же раз, сколько из
второй?
Ответ: Можно. Решение. Если конфеты брал Вася, в коробке остается 16/17, если Петя — 11/12, если Коля — 3/4 от их предыдущего количества. Если Вася, Петя и Коля брали
конфеты из данной коробки несколько раз, их первоначальное количество умножается на
16/17, 11/12 и 3/4 соответствующее количество раз. Если сначала конфет в обоих коробках
было поровну, и в конце — тоже, то дроби, на которые умножалось количество конфет в
первой и второй коробках, также равны. Но тогда у них в несократимой записи в числителях
должно быть поровну множителей 11, а в знаменателях — поровну множителей 17. А так как
всем этим множителям не с чем сокращаться, то их было поровну и в исходной записи. Следовательно, Петя и Вася брали из обоих коробок поровну раз. Поделив на 16/17 и 11/12 в
нужных степенях, увидим, что поровну раз брал и Коля.
 Нет объяснения, почему Коля брал поровну раз — дыра в 6 баллов.
7. Можно ли разрезать квадрат на 17 прямоугольников, у каждого из которых одна
из сторон вдвое длиннее другой?
Ответ: Можно. Решение. Разрежем квадрат 44 на два прямоугольника 24, а потом
один из них — на четыре прямоугольника 12. Получили разрезание квадрата на 5 прямоугольников с отношением сторон 1:2. Заметим теперь, что если разрезать такой прямоугольник средними линиями на 4 равных прямоугольника, то отношение сторон у каждого из четырех тоже будет 1:2. Каждое такое разрезание увеличивает число прямоугольников на 3.
Проделав его 4 раза, получим разрезание на 17 искомых прямоугольников.
8. На острове живут три племени: рыцари, которые всегда говорят правду, лжецы,
которые всегда лгут и хитрецы, которые иногда говорят правду, а иногда лгут. За круглым
столом сидят представители этих племен. Каждый из сидящих за столом произнес две
фразы: 1) «Слева от меня сидит лжец.» 2) «Справа от меня сидит хитрец.» Докажите,
что за этим столом рыцарей сидит столько же, сколько и лжецов.
Решение. Из условия следует, что слева от каждого рыцаря сидит лжец. Справа же от
лжеца не могут сидеть ни хитрец (тогда этот лжец сказал бы про своего правого соседа правду), ни лжец (тогда тот сказал бы правду про своего левого соседа). Поэтому правым соседом
лжеца может быть только рыцарь. Таким образом, все рыцари и лжецы разбиваются на пары,
где лжец — левый сосед рыцаря, откуда и вытекает утверждение задачи.
Группа «Старт», высшая лига, 2 тур, решения и указания для жюри.
1. Назовем соседними такие клетки квадрата 66, у которых есть общая сторона
или общая вершина. Можно ли так расставить все числа от 1 до 36 в клетках этого квадрата, чтобы разность между числами в любых соседних клетках была не меньше 9?
Ответ: Нельзя. Решение. Заметим, что если взять квадрат 22, содержащий число 9, то
в нем же окажутся и числа 18, 27 и 36. Поскольку такой квадрат может быть только один,
девятка должна стоять в углу. Но тем же свойством обладает и любой квадрат 22, содержащий число 18, и потому 18 тоже должно стоять в углу, что несовместимо с девяткой в углу.
74
2. Найдите все натуральные числа n  100000 такие, что если отбросить у n пять
последних цифр, а затем полученное число возвести в квадрат, то получится снова n.
Ответ: 10000000000. Решение. Пусть a — число, которое получается отбрасыванием у
числа n пяти последних цифр. По условию a2 = 100000a+b, где b < 100000. Поделив на a, получаем a = 100000+b/a. Поскольку a — целое и не меньше 100000, то 0 ≤ b/a < 1. Поэтому
равенство a = 100000+b/a возможно только при b = 0, что и дает ответ.
 Только ответ — 0 баллов.
3. В коробке лежат 10 синих и 10 красных шариков. Вася хочет набрать 5 синих шариков из коробки. Для этого он вслепую вытаскивает по два шарика. Если он вытащил два
синих, то берет оба. Если синий и красный, то берет синий, а красный кладет обратно в
коробку. Наконец, если он вытащил два красных, то оба откладывает в сторону. За какое
наименьшее число вытаскиваний Вася заведомо сможет набрать 5 синих шариков?
Ответ: 10. Решение. Пример. Первые 4 раза вытаскиваются красный и синий, потом
пять раз подряд по два красных, и в конце — два синих. Оценка. По два красных шарика
можно вытащить не больше 5 раз. Все остальные вытаскивания добавляют хотя бы по одному синему шарику, поэтому их понадобится не больше пяти.
 Только оценка — 6 баллов, только пример — 4 балла.
4. Верно ли, что среди любых 100 натуральных чисел найдется число, которое делится на наибольший общий делитель всех остальных?
Ответ: Нет. Решение. Контрпример: 100 различных числа, каждое из которых равно
произведению 99 из 100 данных различных простых чисел. НОД любых 99 из этих произведений равен как раз тому простому числу, на которое не делится оставшееся произведение.
 Только ответ — 0 баллов.
5. В двух коробках лежат конфеты. Если к коробке подходит Вася, то он берет 1/17
часть конфет, имеющихся в данный момент в этой коробке, если Петя — то 1/12, а Коля
всегда берет 1/4 часть. Если у берущего конфеты не получается взять ровно столько,
сколько надо, то он воздерживается от сладкого. Вначале в коробках конфет было поровну,
по истечении некоторого срока оказалось, что конфет снова поровну. Можно ли утверждать, что каждый из мальчиков лакомился из первой коробки столько же раз, сколько из
второй?
Ответ: Можно. Решение. Если конфеты брал Вася, в коробке остается 16/17, если Петя — 11/12, если Коля — 3/4 от их предыдущего количества. Если Вася, Петя и Коля брали
конфеты из данной коробки несколько раз, их первоначальное количество умножается на
16/17, 11/12 и 3/4 соответствующее количество раз. Если сначала конфет в обоих коробках
было поровну, и в конце — тоже, то дроби, на которые умножалось количество конфет в
первой и второй коробках, также равны. Но тогда у них в несократимой записи в числителях
должно быть поровну множителей 11, а в знаменателях — поровну множителей 17. А так как
всем этим множителям не с чем сокращаться, то их было поровну и в исходной записи. Следовательно, Петя и Вася брали из обоих коробок поровну раз. Значит, поровну раз брал и
Коля.
6. В группе из 25 школьников любые двое имеют общего знакомого. Докажите, что
из этой группы можно не менее, чем 36 способами выбрать пару знакомых школьников.
Решение. 1) Никто из семерых не может иметь только одного знакомого, потому что
тогда у него и его знакомого не будет общего знакомого. 2) Пусть есть школьник А, имеющий ровно двух знакомых Б и В. Тогда Б и В должны быть знакомы, иначе у А и Б не будет
общего знакомого. По аналогичной причине каждый из оставшихся 22 школьников должен
быть знаком либо с Б, либо с В, то есть у Б и В суммарно не меньше 26 знакомств. Кроме то-
75
го, у каждого из оставшихся 23 школьников (включая А) не менее двух знакомств. Всего
знакомств получается не менее 72, причем каждое сосчитано дважды, откуда и вытекает
утверждение задачи. 3) Если у каждого школьника А хотя бы трое знакомых, всего знакомств не меньше, чем 325/2 = 37,5.
7. На острове живут три племени: рыцари, которые всегда говорят правду, лжецы,
которые всегда лгут и хитрецы, которые иногда говорят правду, а иногда лгут. За круглым
столом сидят представители этих племен. Каждый из сидящих за столом
произнес две фразы: 1) «Слева от меня сидит лжец.» 2) «Справа от меня сидит хитрец.» Докажите, что за этим столом рыцарей сидит столько же,
сколько и лжецов.
Решение. Из условия следует, что слева от каждого рыцаря сидит
лжец. Справа же от лжеца не могут сидеть ни хитрец (тогда этот лжец сказал
бы про своего правого соседа правду), ни лжец (тогда тот сказал бы правду
про своего левого соседа). Поэтому правым соседом лжеца может быть только рыцарь. Таким образом, все рыцари и лжецы разбиваются на пары, где
лжец — левый сосед рыцаря, откуда и вытекает утверждение задачи.
8. Докажите, что для любого целого n > 4 квадрат можно разрезать
на n прямоугольников, у каждого из которых одна из сторон вдвое длиннее
другой.
Решение. На рисунках справа показано, как нужным образом разрезать квадрат на 5, 6 и 7 прямоугольников. Заметим теперь, что если разрезать
такой прямоугольник средними линиями на 4 равных прямоугольника, то
отношение сторон у каждого из четырех будет 1:2. Каждое такое разрезание
увеличивает число прямоугольников на 3, и мы можем, отправляясь от чисел
5, 6 и 7, замостить весь натуральный ряд, начиная с 5.
 Только ответ — 0 баллов.
Группа «Старт», первая лига, 2 тур, решения и указания для жюри.
1. Назовем соседними такие клетки квадрата 44, у которых есть
общая сторона или общая вершина. Можно ли так расставить все числа от
1 до 16 в клетках этого квадрата, чтобы разность между числами в любых
соседних клетках была не меньше 3?
Ответ: Да. Решение. См. рисунок справа.
9
13
10
14
5
1
6
2
11
15
12
16
 Только ответ — 0 баллов.
2. Найдите все натуральные числа n  100 такие, что если отбросить у n две последние цифры, а затем полученное число возвести в квадрат, то получится снова n.
Ответ: 10000. Решение. Пусть a — число, которое получается отбрасыванием у числа
n пяти последних цифр. По условию a2 = 100a+b, где b < 100. Поделив на a, получаем
a = 100000+b/a. Поскольку a — целое и не меньше 100, то 0 ≤ b/a < 1. Поэтому равенство
a = 100+b/a возможно только при b = 0, что и дает ответ.
 Только ответ — 0 баллов.
3. В коробке лежат 10 синих и 10 красных шариков. Вася хочет набрать 5 синих шариков из коробки. Для этого он вслепую вытаскивает по два шарика. Если он вытащил два
синих, то берет оба. Если синий и красный, то берет синий, а красный кладет обратно в
коробку. Наконец, если он вытащил два красных, то оба откладывает в сторону. За какое
наименьшее число вытаскиваний Вася заведомо сможет набрать 5 синих шариков?
76
8
4
7
3
Ответ: 10. Решение. Пример. Первые 4 раза вытаскиваются красный и синий, потом
пять раз подряд по два красных, и в конце — два синих. Оценка. По два красных шарика
можно вытащить не больше 5 раз. Все остальные вытаскивания добавляют хотя бы по одному синему шарику, поэтому их понадобится не больше пяти.
 Только оценка — 6 баллов, только пример — 4 балла.
4. Верно ли, что среди любых четырех натуральных чисел найдется число, которое
делится на наибольший общий делитель всех остальных?
Ответ: Нет. Решение. Контрпример: четыре различных числа, каждое из которых равно произведению трех из четырех данных различных простых чисел. НОД любых трех из
этих произведений равен как раз тому простому числу, на которое не делится четвертое произведение.
 Только ответ — 0 баллов.
5. В двух коробках лежат конфеты. Если к коробке подходит Вася, то он берет 1/3
часть конфет, имеющихся в данный момент в этой коробке, если Петя — то 1/5. Если у берущего конфеты не получается взять ровно столько, сколько надо, то он воздерживается
от сладкого. Вначале в коробках конфет было поровну, по истечении некоторого срока оказалось, что конфет снова поровну. Можно ли утверждать, что каждый из мальчиков лакомился из первой коробки столько же раз, сколько из второй?
Ответ: Можно. Решение. Если конфеты брал Вася, в коробке остается 2/3, а если Петя
— 4/5 от их предыдущего количества. Если Вася и Петя брали конфеты из данной коробки
несколько раз, их первоначальное количество умножается на 2/3 и 4/5 столько раз, сколько
раз конфеты брали Вася и Петя соответственно. Если сначала конфет в обоих коробках было
поровну, и в конце — тоже, то дроби, на которые умножалось количество конфет в первой и
второй коробках, также равны. Но обе эти дроби несократимы, поэтому их знаменатели
должны быть равны, то есть троек в них должно быть поровну, и пятёрок — тоже, что и требовалось доказать.
 Только ответ — 0 баллов.
6. В группе из 7 школьников любые двое имеют общего знакомого. Докажите, что из
этой группы можно не менее, чем 9 способами выбрать пару знакомых школьников.
Решение. 1) Никто из семерых не может иметь только одного знакомого, потому что
тогда у этих двоих не будет общего знакомого. 2) Пусть есть школьник А, имеющий ровно
двух знакомых Б и В. Тогда Б и В должны быть знакомы, иначе у А и Б не будет общего знакомого. По аналогичной причине каждый из оставшихся четверых должен быть знаком либо
с Б, либо с В. При этом если он не знаком одновременно с Б и В, он должен быть знаком с
одним из оставшихся четверых. Если такое знакомство внутри четверки только одно, то хотя
бы двое из четверки знакомы и с Б, и с В, и всего знакомств получается не меньше 10, если
же таких знакомств хотя бы два, то не меньше 9, что и требовалось доказать. 3) Если у каждого школьника А хотя бы трое знакомых, всего знакомств не меньше, чем 37/2 = 10,5.
7. На острове живут три племени: рыцари, которые всегда говорят правду, лжецы,
которые всегда лгут и хитрецы, которые иногда говорят правду, а иногда лгут. За круглым
столом сидят представители этих племен. Каждый из сидящих за столом произнес две
фразы: 1) «Слева от меня сидит лжец.» 2) «Справа от меня сидит хитрец.» Докажите,
что за этим столом рыцарей сидит столько же, сколько и лжецов.
Решение. Из условия следует, что слева от каждого рыцаря сидит лжец. Справа же от
лжеца не могут сидеть ни хитрец (тогда этот лжец сказал бы про своего правого соседа правду), ни лжец (тогда тот сказал бы правду про своего левого соседа). Поэтому правым соседом
77
лжеца может быть только рыцарь. Таким образом, все рыцари и лжецы разбиваются на пары,
где лжец — левый сосед рыцаря, откуда и вытекает утверждение задачи.
8. Можно ли разрезать квадрат на 17 прямоугольников, у каждого из которых одна
из сторон вдвое длиннее другой?
Ответ: Можно. Решение. Разрежем квадрат 44 на два прямоугольника 24, а потом
один из них — на четыре прямоугольника 12. Получили разрезание квадрата на 5 прямоугольников с отношением сторон 1:2. Заметим теперь, что если разрезать такой прямоугольник средними линиями на 4 равных прямоугольника, то отношение сторон у каждого из четырех тоже будет 1:2. Каждое такое разрезание увеличивает число прямоугольников на 3.
Проделав его 4 раза, получим разрезание на 17 искомых прямоугольников.
 Только ответ — 0 баллов.
МАТЕМАТИЧЕСКИЙ БОЙ №3. 19.02.2011
СТАРШАЯ ГРУППА, ВЫСШАЯ ЛИГА
1. Пусть n — натуральное число. Для каждого его простого делителя p рассмотрим
наибольшую его степень, не превосходящую n. Сумму всех таких степеней назовём степенной суммой числа n. Докажите, что существует бесконечно много натуральных чисел, степенные суммы которых превосходят их более, чем в полтора раза.
2. На стороне BC треугольника ABC лежат точки D и E, причем BD < BE. Докажите,
что разность периметров треугольников ABC и ADE больше, чем удвоенная длина меньшего
из отрезков BD и EC.
3. Существуют ли на плоскости четыре бесконечных семейства параллельных прямых
такие, что расстояние между любыми двумя соседними прямыми одного семейства больше 1
и меньше 2, а через каждую точку пересечения двух прямых из разных семейств проходят
прямые двух других семейств?
4. В каждом из 30 сундуков лежит по 13 монет. Монеты весят натуральное число
граммов, не большее 30, причем монет каждого веса ровно 13. Известно, что веса любых
двух монет, лежащих в одном сундуке, отличаются не более, чем на 4 грамма. Назовем сундук максимальным, если суммарный вес монет в нем не меньше, чем в других сундуках. Какой наименьший суммарный вес может быть у монет в максимальном сундуке?
5. В бесконечной последовательности a1, a2, a3, ... первый член равен 1, а каждый следующий получается из предыдущего умножением на одно из чисел 2, 3, …, 9, причем умножение на каждое число из них производилось хотя бы один раз. Докажите, что для бесконечно многих n сумма цифр числа an+1 не превосходит суммы цифр числа an.
6. В тупоугольном треугольнике ABC с тупым углом C угол B в два раза больше угла
A. На стороне AB отмечена точка P такая, что BP = 2BC. Известно, что середина M стороны
AB лежит между P и B. Докажите, что перпендикуляр, опущенный из M на сторону AC, делит
отрезок PC пополам.
7. В каждой клетке таблицы kn (k строк, n столбцов) стоит 1 или 0. Известно, что в
каждой строчке есть хотя бы две единицы и хотя бы один ноль. При каком наибольшем k заведомо можно так переставить столбцы, чтобы в результате ни в одной строчке все единицы
не стояли подряд?
8. В кружок записались 2011 мальчиков и 2011 девочек, причем каждый из мальчиков
знаком хотя бы с 10 девочками. Докажите, что можно выбрать 100 мальчиков и 10 девочек
78
так, чтобы каждый из выбранных мальчиков был знаком хотя бы с одной из выбранных девочек.
СТАРШАЯ ГРУППА, ПЕРВАЯ ЛИГА
1. Пусть n — натуральное число. Для каждого его простого делителя p рассмотрим
наибольшую его степень, не превосходящую n. Сумму всех таких степеней назовём степенной суммой числа n. Докажите, что существует бесконечно много натуральных чисел, меньших, чем их степенные суммы.
2. На стороне BC треугольника ABC лежат точки D и E, причем BD < BE. Докажите,
что разность периметров треугольников ABC и ADE больше, чем удвоенная длина меньшего
из отрезков BD и EC.
3. Существуют ли на плоскости четыре конечных семейства параллельных прямых (в
каждом — хотя бы две прямые) такие, что расстояние между любыми двумя соседними прямыми одного семейства больше 1 и меньше 2, а через каждую точку пересечения двух прямых из разных семейств проходят прямые из двух других семейств?
4. В каждом из 30 сундуков лежит по 13 монет. Монеты весят натуральное число
граммов, не большее 30, причем монет каждого веса ровно 13. Известно, что веса любых
двух монет, лежащих в одном сундуке, отличаются не более, чем на 2 грамма. Назовем сундук максимальным, если суммарный вес монет в нем не меньше, чем в других сундуках. Какой наименьший суммарный вес может быть у монет в максимальном сундуке?
5. В бесконечной последовательности a1, a2, a3, ... первый член равен 1, а каждый следующий получается из предыдущего умножением на одно из чисел 2, 3, …, 9, причем умножение на каждое число из них производилось хотя бы один раз. Докажите, что для бесконечно многих n сумма цифр числа an+1 не превосходит суммы цифр числа an.
6. Даны натуральное n и простое p. Докажите, что если n! делится на pp, то оно делится и на pp+1.
7. В равнобедренном треугольнике ABC (AB = AC) A = 30. На медиане AD взята
точка P, а на стороне AB — точка Q таким образом, что PB = PQ. Найдите угол PQC.
8. В кружок записались 2011 мальчиков и 2011 девочек, причем каждый из мальчиков
знаком хотя бы с 10 девочками. Докажите, что можно выбрать 100 мальчиков и 10 девочек
так, чтобы каждый из выбранных мальчиков был знаком хотя бы с одной из выбранных девочек.
СТАРШАЯ ГРУППА, ВТОРАЯ ЛИГА
1. Пусть n — натуральное число. Для каждого его простого делителя p рассмотрим
наибольшую его степень, не превосходящую n. Сумму всех таких степеней назовём степенной суммой числа n. Докажите, что существует бесконечно много натуральных чисел, меньших, чем их степенные суммы.
2. В трапеции ABCD с основаниями AB и CD ABC = 65 и ADC = 130. Биссектриса
угла ADC пересекает отрезок AB в точке E. Известно, что AD = 12 и BE = 15. Найдите среднюю линию трапеции.
3. Существуют ли на плоскости четыре конечных семейства параллельных прямых (в
каждом — хотя бы две прямые) таких, что расстояние между любыми двумя соседними прямыми одного семейства больше 1 и меньше 2, а через каждую точку пересечения двух прямых из разных семейств проходят прямые из двух других семейств?
79
4. Даны натуральное n и простое p. Докажите, что если n! делится на pp, то оно делится и на pp+1.
5. В каждом из 30 сундуков лежит по 13 монет. Монеты весят натуральное число
граммов, не большее 30, причем монет каждого веса ровно 13. Известно, что веса любых
двух монет, лежащих в одном сундуке, отличаются не более, чем на 2 грамма. Назовем сундук максимальным, если суммарный вес монет в нем не меньше, чем в других сундуках. Какой наименьший суммарный вес может быть у монет в максимальном сундуке?
6. В строчку выписывают натуральные числа. Каждое число, начиная с третьего –
наибольший нечетный делитель суммы двух предыдущих чисел, уменьшенный на 1. Докажите, что рано или поздно в строчке появится 0.
7. В равнобедренном треугольнике ABC (AB = AC) A = 30. На медиане AD взята
точка P, а на стороне AB — точка Q таким образом, что PB = PQ. Найдите угол PQC.
8. В кружок записались 2011 мальчиков и 2011 девочек, причем каждый из мальчиков
знаком хотя бы с 10 девочками. Докажите, что можно выбрать 19 мальчиков и 2 девочек так,
чтобы каждый из выбранных мальчиков был знаком хотя бы с одной из выбранных девочек.
МЛАДШАЯ ГРУППА, ВЫСШАЯ ЛИГА
1. На каждой клетке доски 89 (8 строк, 9 столбцов) стоит по фишке. В некоторый
момент каждая фишка сдвинулась на соседнее поле по горизонтали или диагонали. Докажите, что в результате этого образовалось хотя бы 8 пустых клеток.
2. Петя и Вася играют в такую игру. На крайнем левом поле клетчатой ленты длины
20 лежит кучка из 2011 камней. Игроки ходят по очереди, начинает Петя. Каждый из ребят
своим ходом может сдвинуть любой камень на одно или два поля вправо. Выигрывает тот,
кто первым поставит камень на крайнюю правую клетку. Кто выиграет при правильной игре?
3. На доске написаны числа 1, 2, ..., 33. За один шаг можно стереть любые два числа,
произведение которых — квадрат натурального числа, и вместо них записать квадратный
корень из их произведения (в частности можно заменить два равных числа на одно). После
нескольких шагов на доске остались числа, произведение любых двух из которых — не точный квадрат. Докажите, что на доске осталось не менее 16 чисел.
4. Найдите все натуральные n, для которых число 5n+12n является точным квадратом.
5. В каждом из 30 сундуков лежит по 13 монет. Монеты весят натуральное число
граммов, не большее 30, причем монет каждого веса ровно 13. Известно, что веса любых
двух монет, лежащих в одном сундуке, отличаются не более, чем на 2 грамма. Назовем сундук наибольшим, если суммарный вес монет в нем не меньше, чем в других сундуках. Какой
наименьший суммарный вес может быть у монет в наибольшем сундуке?
6. Для чисел a и b, не меньших 1, докажите неравенство
(a2+1)(b2+1)–(a–1)2(b–1)2  4.
7. O — точка пересечения диагоналей CE и AD выпуклого пятиугольника ABCDE. Известно, что AB = DE, BC = AD = CE/2 и ADE = BAC+BCA. Докажите, что AO = OE.
8. В кружок записались 2011 мальчиков и 2011 девочек, причем каждый из мальчиков
знаком хотя бы с 10 девочками. Докажите, что можно выбрать 100 мальчиков и 10 девочек
так, чтобы каждый из выбранных мальчиков был знаком хотя бы с одной из выбранных девочек.
80
МЛАДШАЯ ГРУППА, ПЕРВАЯ ЛИГА
1. На каждой клетке доски 89 (8 строк, 9 столбцов) стоит по фишке. В некоторый
момент каждая фишка сдвинулась на соседнее поле по горизонтали или диагонали. Докажите, что в результате этого образовалось хотя бы 8 пустых клеток.
2. Петя и Вася играют в такую игру. На крайнем левом поле клетчатой ленты длины
20 лежит кучка из 2010 камней. Игроки ходят по очереди, начинает Петя. Каждый из ребят
своим ходом может сдвинуть любой камень на одно или два поля вправо. Выигрывает тот,
кто первым поставит камень на крайнюю правую клетку. Кто выиграет при правильной игре?
3. На доске написаны числа 1, 2, ..., 33. За один шаг можно стереть любые два числа,
произведение которых — квадрат натурального числа, и вместо них записать квадратный
корень из их произведения (в частности можно заменить два равных числа на одно). После
нескольких шагов на доске остались числа, произведение любых двух из которых — не точный квадрат. Докажите, что на доске осталось не менее 15 чисел.
4. Найдите все натуральные n, для которых число 5n+12n является точным квадратом.
5. При каких натуральных n > 1 натуральные числа от 1 до n можно так покрасить в
красный, синий, желтый и зеленый цвета, что чисел каждого цвета будет поровну и суммы
синих, красных, желтых и зеленых чисел одинаковы.
6. Для чисел a и b, не меньших 1, докажите неравенство
(a2+1)(b2+1)–(a–1)2(b–1)2  4.
7. O — точка пересечения диагоналей CE и AD выпуклого пятиугольника ABCDE. Известно, что AB = DE, BC = AD = CE/2 и ADE = BAC+BCA. Докажите, что AO = OE.
8. В кружок записались 15 мальчиков и 15 девочек, причем каждый из мальчиков знаком хотя бы с тремя девочками. Докажите, что можно выбрать шестерых мальчиков и двух
девочек так, чтобы каждый из выбранных мальчиков был знаком хотя бы с одной из выбранных девочек.
МЛАДШАЯ ГРУППА, ВТОРАЯ ЛИГА
1. На каждой клетке доски 89 (8 строк, 9 столбцов) стоит по фишке. В некоторый
момент каждая фишка сдвинулась на соседнее поле по горизонтали или диагонали. Докажите, что в результате этого образовалось хотя бы одна пустая клетка.
2. Петя и Вася играют в такую игру. На крайнем левом поле клетчатой ленты длины
19 лежит кучка из 2011 камней. Игроки ходят по очереди, начинает Петя. Каждый из ребят
своим ходом может сдвинуть любой камень на одно или два поля вправо. Выигрывает тот,
кто первым поставит камень на крайнюю правую клетку. Кто выиграет при правильной игре?
3. На доске написаны числа 1, 2, ..., 33. За один шаг можно стереть любые два числа,
произведение которых — квадрат натурального числа, и вместо них записать квадратный
корень из их произведения (в частности можно заменить два равных числа на одно). После
нескольких шагов на доске остались числа, произведение любых двух из которых — не точный квадрат. Докажите, что на доске осталось не менее 9 чисел.
4. Вася перемножил все натуральные числа от 1000 до 2000 включительно и прибавил
к этому произведению единицу. Докажите, что все простые делители полученного числа
больше 2000.
81
5. При каких натуральных n > 1 натуральные числа от 1 до n можно так покрасить в
красный, синий, желтый и зеленый цвета, что чисел каждого цвета будет поровну и суммы
синих, красных, желтых и зеленых чисел одинаковы.
6. Для чисел a и b, не меньших 1, докажите неравенство
(a2+1)(b2+1)–(a–1)2(b–1)2  4.
7. Существуют ли два таких картонных треугольника, из которых, разными способами прикладывая их друг к другу без наложений, можно получить треугольник, четырёхугольник, пятиугольник и шестиугольник?
8. В кружок записались 15 мальчиков и 15 девочек, причем каждый из мальчиков знаком хотя бы с тремя девочками. Докажите, что можно выбрать пятерых мальчиков и двух
девочек так, чтобы каждый из выбранных мальчиков был знаком хотя бы с одной из выбранных девочек.
ГРУППА «СТАРТ», ВЫСШАЯ ЛИГА
1. Петя и Вася играют в такую игру. На крайнем левом поле клетчатой ленты длины
20 лежит кучка из 2011 камней. Игроки ходят по очереди, начинает Петя. Каждый из ребят
своим ходом может сдвинуть любой камень на одно или два поля вправо. Выигрывает тот,
кто первым поставит камень на крайнюю правую клетку. Кто выиграет при правильной игре?
2. В кружок записались 15 мальчиков и 15 девочек, причем каждый из мальчиков знаком хотя бы с тремя девочками. Докажите, что можно выбрать пятерых мальчиков и двух
девочек так, чтобы каждый из выбранных мальчиков был знаком хотя бы с одной из выбранных девочек.
3. На доске написано несколько (больше одного) различных натуральных чисел. Известно, что из любых трех написанных чисел можно выбрать два таких, что одно из них делится на другое. Докажите, что можно выбрать такие два написанных числа a и b, что любое
из написанных чисел делится хотя бы на одно из этих двух.
4. На каждой клетке доски 89 (8 строк, 9 столбцов) стоит по фишке. В некоторый
момент каждая фишка сдвинулась на соседнее поле по горизонтали или диагонали. Докажите, что в результате этого образовалось хотя бы 8 пустых клеток.
5. На доске написаны все натуральные числа от 1 до 100. За одну операцию любые
два написанных числа заменяют на сумму цифр их суммы. В результате 99 таких операций
на доске осталось одно число. Может ли оно равняться 5?
6. Вася перемножил все натуральные числа от 100 до 200 включительно и прибавил к
этому произведению единицу. Докажите, что все делители полученного числа, кроме единицы, больше 200.
7. Среди семи монет есть одна фальшивая, которая на 1 грамм легче настоящих, и одна фальшивая, которая на 1 грамм тяжелее настоящих. Как за 4 взвешивания на чашечных
весах без гирь найти и лёгкую, и тяжёлую монеты?
8. Существуют ли два таких картонных треугольника, из которых, разными способами прикладывая их друг к другу без наложений, можно получить треугольник, четырёхугольник, пятиугольник и шестиугольник?
ГРУППА «СТАРТ», ПЕРВАЯ ЛИГА
1. На первой горизонтали клетчатой доски 99 стоят пешки. Пешкой разрешается ходить только вперёд на одно или два поля. Выходить за пределы доски нельзя. Петя и Вася
82
ходят по очереди, начинает Петя. Проигрывает тот, кто не может сделать хода. Кто выиграет
при правильной игре?
2. В кружок записались 15 мальчиков и 15 девочек, причем каждый из мальчиков знаком хотя бы с тремя девочками. Докажите, что можно выбрать 5 мальчиков и двух девочек
так, чтобы каждый из выбранных мальчиков был знаком хотя бы с одной из выбранных девочек.
3. Даны 2011 различных натуральных чисел. Известно, что для любых двух из этих
чисел одно делится на другое. Докажите, что у какого-то из этих 2011 чисел среди остальных
данных чисел столько же делителей, сколько чисел делятся на него.
4. На каждой клетке доски 89 (8 строк, 9 столбцов) стоит по фишке. В некоторый
момент каждая фишка сдвинулась на соседнее поле по горизонтали или диагонали. Докажите, что в результате этого образовалось хотя бы 8 пустых клеток.
5. На доске написаны все натуральные числа от 1 до 99. За одну операцию любые два
написанных числа заменяют на сумму цифр их суммы. В результате 98 таких операций на
доске осталось одно число. Может ли оно равняться 5?
6. Вася перемножил все натуральные числа от 100 до 200 включительно и прибавил к
этому произведению единицу. Докажите, что все простые делители полученного числа, кроме единицы, больше 200.
7. Среди шести монет есть одна фальшивая, которая на 1 грамм легче настоящих, и
одна фальшивая, которая на 1 грамм тяжелее настоящих. Как за 4 взвешивания на чашечных
весах без гирь найти и лёгкую и тяжёлую монеты?
8. Существуют ли два таких картонных треугольника, из которых, разными способами прикладывая их друг к другу без наложений, можно получить треугольник, четырёхугольник, пятиугольник и шестиугольник?
МАТЕМАТИЧЕСКИЙ БОЙ №4. 20.02.2011
СТАРШАЯ ГРУППА, ВЫСШАЯ ЛИГА, БОИ ЗА 1-6 МЕСТА
1. Найдите наибольшее и наименьшее возможное значение суммы S = a/b+c/d, где a,
b, c, d — натуральные числа, удовлетворяющие условиям a+c = 20202, b+d = 20200.
2. В каждом из 100 сосудов лежит по 99 камней. Два игрока ходят по очереди. Каждый игрок при своем ходе должен взять по одному камню из 98 сосудов. Игрок, после хода
которого два сосуда оказались пусты, выигрывает. Кто выиграет при правильной игре —
начинающий или его партнер?
3. M и N — середины сторон CD и AD выпуклого четырехугольника ABCD соответственно. Перпендикуляр к AB, проведенный через M, и перпендикуляр к BC, проведенный
через N, пересекаются в точке P. Докажите, что P лежит на диагонали BD тогда и только тогда, когда диагонали AC и BD перпендикулярны.
4. По кругу записаны в порядке возрастания по часовой стрелке числа от 1 до 31. Разрешается взять любые три числа a, b, c (не обязательно соседние и стоящие в любом порядке) и заменить их числами c, a–1/10, и b+1/10 соответственно. Докажите, что, применяя такие
операции, можно добиться, чтобы числа стояли в порядке возрастания против часовой
стрелки.
5. В трапеции ABCD с основаниями BC и AD биссектриса угла CAD проходит через
середину отрезка BD. Известно, что BD = 2AB. Докажите, что AD = 3BC.
83
6. Найдите все такие натуральные m, n и простые p, что число
7 m  2n p
7 m  2n p
— нату-
ральное.
7. Из чисел 1, 2, ..., 37 произвольным образом выбраны 11 различных. Докажите, что
из этих 11 чисел можно выбрать четыре таких, что сумма двух из них равна сумме двух других.
8. Таблица 5050 заполнена натуральными числами таким образом, что все суммы
чисел по строкам и столбцам равны. Какое наименьшее количество чисел надо изменить,
чтобы все эти 100 сумм стали попарно различны?
СТАРШАЯ ГРУППА: ВЫСШАЯ ЛИГА, БОЙ ЗА 7 МЕСТО,
ПЕРВАЯ ЛИГА, БОИ 3А 1-6 МЕСТА
1. Найдите наибольшее и наименьшее возможное значение суммы S = a/b+c/d, где a,
b, c, d — натуральные числа, удовлетворяющие условиям a+c = 20202, b+d = 20200.
2. В каждом из 100 сосудов лежит по 100 камней. Два игрока ходят по очереди. Каждый игрок при своем ходе должен взять по одному камню из 98 сосудов. Игрок, после хода
которого два сосуда оказались пусты, выигрывает. Кто выиграет при правильной игре —
начинающий или его партнер?
3. Биссектрисы углов A и C треугольника ABC пересекаются в точке I. Известно, что
AI = BC и ICA = 2IAC. Найдите угол ABC.
4. Известно, что
a
b
c
(a  b)(b  c)(c  a) 1


.
 . Найдите
ab bc ca
(a  b)(b  c)(c  a) 11
5. В трапеции ABCD с основаниями BC и AD биссектриса угла CAD проходит через
середину отрезка BD. Известно, что BD = 2AB. Докажите, что AD = 3BC.
6. Найдите все такие натуральные m, n и простые p, что число
7 m  2n p
7 m  2n p
— нату-
ральное.
7. Из чисел 1, 2, ..., 37 произвольным образом выбраны 13 различных. Докажите, что
из этих 13 чисел можно выбрать четыре таких, что сумма двух из них равна сумме двух других.
8. Таблица 5050 заполнена натуральными числами таким образом, что все суммы
чисел по строкам и столбцам равны. Какое наименьшее количество чисел надо изменить,
чтобы все эти 100 сумм стали попарно различны?
СТАРШАЯ ГРУППА: ПЕРВАЯ ЛИГА, БОЙ ЗА 7 МЕСТО,
ВТОРАЯ ЛИГА
1. Найдите наибольшее и наименьшее возможное значение суммы S = a/b+c/d, где a,
b, c, d — натуральные числа, удовлетворяющие условиям a+c = 20202, b+d = 20200.
2. В каждом из 100 сосудов лежит по 100 камней. Два игрока ходят по очереди. Каждый игрок при своем ходе должен взять по одному камню из 98 сосудов. Игрок, после хода
которого два сосуда оказались пусты, выигрывает. Кто выиграет при правильной игре —
начинающий или его партнер?
84
3. На стороне AC прямоугольного треугольника ABC с прямым углом B взята точка D
такая, что CD = AB, а на стороне BC — точка E такая, что DB = DE. Известно, что
CAB = 2ABD. Найдите угол EDC.
4. Известно, что
a
b
c
(a  b)(b  c)(c  a) 1


.
 . Найдите
ab bc ca
(a  b)(b  c)(c  a) 11
5. Дана трапеция ABCD с основаниями BC и AD. Точка K — середина диагонали BD.
Оказалось, что AK — биссектриса угла CAD и AD = 3BC. Докажите, что AC = 2BC.
6. Найдите все такие натуральные m и простые p, что число
7m  2 p
7m  2 p
— натураль-
ное.
7. Из чисел 1, 2, ..., 37 произвольным образом выбраны 13 различных. Докажите, что
из этих 13 чисел можно выбрать четыре таких, что сумма двух из них равна сумме двух других.
8. Сколькими способами можно раскрасить клетки прямоугольника 22011 в два цвета так, чтобы никакие три клетки одного цвета не образовывали уголок из трех клеток?
МЛАДШАЯ ГРУППА, ВЫСШАЯ ЛИГА, БОИ ЗА 1-4 МЕСТА
1. Найдите наибольшее число, делящееся на 72, которое можно получить из числа
123...20092010 (все натуральные числа от 1 до 2010 записаны подряд) вычеркиванием некоторых цифр.
2. Таблица 5050 заполнена натуральными числами таким образом, что все суммы
чисел по строкам и столбцам равны. Какое наименьшее количество чисел надо изменить,
чтобы все эти 100 сумм стали попарно различны?
3. Для любых целых чисел a, b, c и d докажите неравенство a2+b2+c2+3d2  (2d–
1)(a+b+c)+3d.
4. Из чисел 1, 2, ..., 37 произвольным образом выбраны 11 различных. Докажите, что
из этих 11 чисел можно выбрать четыре таких, что сумма двух из них равна сумме двух других.
5. Дана трапеция ABCD с основаниями BC и AD. Точка K — середина диагонали BD.
Оказалось, что AK — биссектриса угла CAD и AD = 3BC. Докажите, что AC = 2BC.
6. В каждом из 100 сосудов лежит по 97 камней. Два игрока ходят по очереди. Каждый игрок при своем ходе должен взять по одному камню из 98 сосудов. Игрок, после хода
которого два сосуда оказались пусты, выигрывает. Кто выиграет при правильной игре —
начинающий или его партнер?
7. Аня, Боря, Вика и Галя навещали Диму. Договорились они прийти в один день, но в
разное время и пробыли у Димы по 15 минут. Аня посетила Диму в 8 часов, Боря пришел в 9
часов, Вика — в 10 часов, а Галя пришла в 11 часов, только неизвестно кто из них приходил
утром, а кто вечером. Известно, что: 1) К Диме кто-то приходил между Аней и Борей; 2) Какая-то из девочек пришла к Диме до Ани; 3) Вика не заходила к Диме между Борей и Галей.
Определите кто в какое время приходил к Диме.
8. Найдите все простые числа p такие, что число (p2–23)(9p+5) — точный квадрат.
85
МЛАДШАЯ ГРУППА: ВЫСШАЯ ЛИГА, БОИ ЗА 5-8 МЕСТА,
ПЕРВАЯ ЛИГА, БОИ 3А 1-6 МЕСТА
1. Найдите наибольшее число, делящееся на 72, которое можно получить из числа
123...20092010 (все натуральные числа от 1 до 2010 записаны подряд) вычеркиванием некоторых цифр.
2. Таблица 4848 заполнена натуральными числами таким образом, что все суммы
чисел по строкам и столбцам равны. Какое наименьшее количество чисел надо изменить,
чтобы все эти 96 сумм стали попарно различны?
3. Для любых целых чисел a, b и c докажите неравенство a2+b2+2c2  (2c–1)(a+b)+2c.
4. Из чисел 1, 2, ..., 37 произвольным образом выбраны 13 различных. Докажите, что
из этих 13 чисел можно выбрать четыре таких, что сумма двух из них равна сумме двух других.
5. Дана трапеция ABCD с основаниями BC и AD. Точка K — середина диагонали BD.
Оказалось, что AK — биссектриса угла CAD и AD = 3BC. Докажите, что AC = 2BC.
6. Дан прямоугольник 22012 (2 строки, 2012 столбцов). Два игрока ходят по очереди.
Первый своим ходом вычеркивает две соседние по вертикали клетки, а второй вычеркивает
две соседние по горизонтали клетки. Вычеркивать ранее вычеркнутые клетки нельзя. Игрок,
не имеющий хода, проигрывает. Кто выигрывает при правильной игре?
7. Аня, Боря, Вика и Галя навещали Диму. Договорились они прийти в один день, но в
разное время и пробыли у Димы по 15 минут. Аня посетила Диму в 8 часов, Боря пришел в 9
часов, Вика — в 10 часов, а Галя пришла в 11 часов, только неизвестно кто из них приходил
утром, а кто вечером. Известно, что: 1) К Диме кто-то приходил между Аней и Борей; 2) Какая-то из девочек пришла к Диме до Ани; 3) Вика не заходила к Диме между Борей и Галей.
Определите кто в какое время приходил к Диме.
8. Найдите все простые числа p такие, что число (p2–23)(9p+5) — точный квадрат.
МЛАДШАЯ ГРУППА: ПЕРВАЯ ЛИГА, БОЙ ЗА 7 МЕСТО,
ВТОРАЯ ЛИГА
1. Найдите наибольшее число, делящееся на 72, которое можно получить из числа
123...20092010 (все натуральные числа от 1 до 2010 записаны подряд) вычеркиванием некоторых цифр.
2. 100 депутатов Думы пользуются сетью Билайн, а 200 депутатов – сетью Мегафон.
За внутрисетевой звонок Билайн берёт 43 копейки, а Мегафон меньше, но целое число копеек. За звонок в другую сеть стоимость звонка возрастает в 3 раза. Все входящие звонки бесплатные. В течение дня каждый депутат звонит каждому по одному разу и от каждого один
раз получает встречный звонок. Сколько стоят звонки с Мегафона, если его ежедневные доходы с депутатов более чем на 11000 рублей превышают доходы Билайна?
3. Числа a, b, c и d удовлетворяют условию a+d = b+c. Докажите неравенство (a–b)(c–
d)+(a–c)(b–d)+(d–a)(b–c)  0.
4. Из чисел 1, 2, ..., 37 произвольным образом выбраны 13 различных. Докажите, что
из этих 13 чисел можно выбрать четыре таких, что сумма двух из них равна сумме двух других.
5. На диагонали BD выпуклого четырехугольника ABCD отмечена точка E. Известно,
что AB = CE, ABD = BCE = ECD и DAB = ABC. Докажите, что треугольник BCD равнобедренный.
86
6. Дан прямоугольник 22012 (2 строки, 2012 столбцов). Два игрока ходят по очереди.
Первый своим ходом вычеркивает две соседние по вертикали клетки, а второй вычеркивает
две соседние по горизонтали клетки. Вычеркивать ранее вычеркнутые клетки нельзя. Игрок,
не имеющий хода, проигрывает. Кто выигрывает при правильной игре?
7. Аня, Боря, Вика и Галя навещали Диму. Договорились они прийти в один день, но в
разное время и пробыли у Димы по 15 минут. Аня посетила Диму в 8 часов, Боря пришел в 9
часов, Вика — в 10 часов, а Галя пришла в 11 часов, только неизвестно кто из них приходил
утром, а кто вечером. Известно, что: 1) К Диме кто-то приходил между Аней и Борей; 2) Какая-то из девочек пришла к Диме до Ани; 3) Вика не заходила к Диме между Борей и Галей.
Определите кто в какое время приходил к Диме.
8. Трехзначные числа abc и cba делятся на простое число p. Докажите, что хотя бы
одно из чисел a+b+c, a–b+c или a–c также делится на p.
ГРУППА «СТАРТ», ВЫСШАЯ ЛИГА
1. Дано натуральное число n > 100. Докажите, что найдется число, которое меньше n,
больше единицы, и не имеет с числом n(n–1) общих делителей, кроме 1.
2. В некоторые клетки таблицы 1313 записали числа. Оказалось, что все суммы чисел по строкам и столбцам попарно различны. Какое наибольшее количество клеток могли
остаться свободными?
3. Из чисел 1, 2, ..., 100 произвольным образом выбраны 20 различных. Докажите, что
из этих 20 чисел можно выбрать четыре таких, что сумма двух из них равна сумме двух других.
4. На каждой клетке доски 88 лежит камень. Камни убывают по весу в каждой строке (слева направо) и каждом столбце (снизу вверх). Разрешается взвесить на весах (показывающих вес груза) любой камень (несколько камней сразу взвешивать нельзя). Покажите,
как за 15 таких взвешиваний определить, есть ли на доске камень, весящий 100 грамм.
5. На доске написано число 2011. Вася может написанное число умножать на 8 или
делить на 8 (если частное — целое число), либо переставлять в нем цифры (0 на первое место ставить нельзя). Может ли Вася такими операциями получить число 1?
6. На доску выписано несколько натуральных чисел, не превосходящих 100. Оказалось, что числа 1 и 2 выписаны и ни одно из выписанных чисел не равно сумме двух различных выписанных чисел. Какое наибольшее количество чисел могло быть выписано?
7. Как провести на круглом листе бумаги пять отрезков, каждый из которых соединяет
две точки на краю листа, чтобы среди частей, на которые эти отрезки делят лист, оказались
хотя бы один пятиугольник и хотя бы два четырехугольника?
8. В стране несколько городов. Некоторые пары городов соединены дорогами, причем
из любого города в любой другой можно добраться по этим дорогам. Оказалось, что из двух
городов выходят по две дороги, а из остальных — по три или по одной. Докажите, что можно закрыть несколько дорог на ремонт таким образом, чтобы из каждого города выходило по
три или по одной дороге.
ГРУППА «СТАРТ», ПЕРВАЯ ЛИГА
1. Назовём натуральное число n, большее 3, сложным, если с каждым натуральным
числом, меньшими n, кроме 1 и n–1, у него есть общий простой делитель. Найдите все сложные числа, меньшие 1000.
87
2. В некоторые клетки таблицы 55 записали числа. Оказалось, что все суммы чисел
по строкам и столбцам попарно различны. Какое наибольшее количество клеток могли
остаться свободными?
3. Определите максимально возможное число, делящееся на 6, которое может быть
получено из числа 123…200920102011 вычеркиванием некоторых из его цифр. .
4. На 49 карточках записаны числа от 1 до 49. Карточки выложены в квадрат 77 числами вниз. Известно, что в каждом горизонтальном ряду числа упорядочены по возрастанию
слева направо, а в вертикальном – сверху вниз. Можно ли последовательно открыть не более
12 карточек, чтобы найти карточку с числом 25?
5. В кружке 6 человек и у каждого ровно 3 друга в этом кружке. Верно ли что всех
кружковцев можно рассадить за 3 парты, за каждой из которых сидят 2 друга?
6. Петя и Вася по очереди умножают записанное на доске число на одно из трёх чисел: 2, 3 или 4. Сначала на доске написана единица. Выигрывает тот, кто первым получит результат больше 250000. Кто может выиграть независимо от игры соперника?
7. Как провести на круглом листе бумаги пять отрезков, каждый из которых соединяет
две точки на краю листа, чтобы среди частей, на которые эти отрезки делят лист, оказались
хотя бы один пятиугольник и хотя бы один четырехугольник?
8. 100 депутатов Думы пользуются сетью Билайн, а 200 депутатов – сетью Мегафон.
За внутрисетевой звонок Билайн берёт 43 копейки, а Мегафон меньше, но целое число копеек. За звонок в другую сеть стоимость звонка возрастает в 3 раза. Все входящие звонки бесплатные. В течение дня каждый депутат звонит каждому по одному разу и от каждого один
раз получает встречный звонок. Сколько стоят звонки с Мегафона, если его ежедневные доходы с депутатов более чем на 11000 рублей превышают доходы Билайна?
СТАРШАЯ ГРУППА
N Задача
1 Число x3+2x2+6x+1 является точным кубом (x- натураль.
ное число). Найдите все такие x.
.
.
.
.
2 В Трансильвании есть люди и упыри, причем люди всегда говорят правду, а упыри всегда лгут. Часть населения психически нормальна, а часть сошла с ума. Сумасшедшие все истинные суждения считают ложными, а все ложные утверждения — истинными (например, сумасшедший упырь будет говорить правду, СЧИТАЯ, что это ложь). Муж и жена либо оба
упыри, либо оба люди.
Жена: « Все, что говорит мой муж, правда».
Муж: «Моя жена свихнулась».
Напишите, кто они ( и каждому напишите его психическое состояние).
3 Найдите НОД всех чисел вида p4–1,где p – все простые
числа от 7 до 2011.
4 Король ходит вправо, вверх и вправо-вверх на доске
5×20. Он вышел из одного угла и пришел в другой угол. Какое
наименьшее количество ходов он мог сделать?
5 В трапеции ABCD основание ВС вдвое короче основания AD, а ВКС=90, где K – середина AD. Найдите отношение AD:KM, где М – точка пересечения диагоналей трапеции.
6 В треугольнике ABC проведены высоты BD и CE. Ока-
88
Ответ
.
.
залось, что CBD= 2ABD, ACE =3BCE. Найдите углы
треугольника ABC.
7 Из данных пяти различных отрезков надо выбрать три и
составить из них прямоугольный треугольник. Оказалось, что
можно сделать это четырьмя различными способами. Найдите
отношение длин наибольшего и наименьшего отрезков.
8 Вычислите,
чему
равно
.
.
89
МЛАДШАЯ ГРУППА
N Задача
1 Найдите все натуральные n, которые равны сумме некото.
рых трех различных натуральных делителей числа n – 1.
.
.
.
.
.
.
.
2 Кассир продал все билеты в первый ряд кинотеатра, причем по ошибке на одно из мест было продано два билета. Сумма
номеров мест на всех этих билетах равна 857. На какое место
продано два билета?
3 За один ход разрешается умножить число на 2 или прибавить к нему 3. За какое количество ходов можно из 3 получить
число 2010? Найдите все варианты ответов .
4 Король ходит вправо, вверх и вправо-вверх на доске 5×20.
Он вышел из одного угла и пришел в другой угол. Какое
наименьшее количество ходов он мог сделать?
5 Впишите в середину числа 2012 ( между 20 и 12)
наименьшее возможное число цифр так, чтобы полученное многозначное число делилось 2011.
6 Некоторые буквы заменили цифрами, причем одинаковые
– одинаковыми, разные – разными. Даны 4 числа 1234, 5678,
9278, 0834 и имена ВАЛЯ, КОЛЯ, ДИМА, РОМА. Запишите на
этом шифре КИРОВ..
7 Длина круга стадиона равна 400м. Три бегуна одновременно стартовали в часовом забеге с одной стартовой линии,
каждый – со своей постоянной скоростью. Первый бегун пробежал 20 км, второй – 19 км, третий – 18км. Сколько раз во время
этого забега один из бегунов обгонял другого? Одновременный
финиш не считается обгоном.
8 В треугольнике ABC проведены высоты BD и CE. Оказалось, что CBD= 2ABD, ACE =3BCE. Найдите углы треугольника ABC.
90
Ответ
ГРУППА «СТАРТ»
N
Задача
1
Ответ
.
. Разрежьте квадрат на 4 равные части
так, чтобы в каждой секции была гусеница со своим листком. У одной
гусеницы не будет листка, она садится на диету.
.
.
.
.
.
.
.
2
Замените звёздочки натуральными числами так, чтобы получилось верное равенство:
(9/*)-(*/21)=17/42.
3
За один ход разрешается умножить число на 2 или прибавить к
нему 3. За какое наименьшее количество ходов можно из 3 получить
число 300?
4
Длина круга стадиона равна 400м. Три бегуна одновременно
стартовали в часовом забеге с одной стартовой линии, каждый – со своей постоянной скоростью. Первый бегун пробежал 20 км, второй – 19
км, третий – 18км. Сколько раз во время этого забега один из бегунов
обгонял другого? Одновременный финиш не считается обгоном.
5
Впишите в середину числа 2012 ( между 20 и 12) наименьшее
возможное число цифр так, чтобы полученное многозначное число делилось 2011.
6
Некоторые буквы заменили цифрами, причем одинаковые –
одинаковыми, разные – разными. Даны 4 числа 1234, 5678, 9278, 0834 и
имена ВАЛЯ, КОЛЯ, ДИМА, РОМА. Запишите на этом шифре КИРОВ..
7
Дно квадратного бассейна выложено квадратными плитками двух цветов, как показано на рисунке. Прораб заказал плиток одного вида на 2011
больше, чем другого. Найдите размеры бассейна.
8
На четырех карточках с одной стороны написаны числа 2, 5, 7 и
12 (каждое – ровно на одной карточке), а с другой стороны - слова «делится на 7», «простое», «нечетное», «больше 100» (каждое – ровно на
одной карточке). Известно, что в каждой карточке число, написанное на
ней, НЕ соответствует слову на другой стороне. Найдите, какие числа
на каких карточках написаны.
91
Старшая группа, высшая лига, 4 тур, решения и указания для жюри.
1. Найдите наибольшее и наименьшее возможное значение суммы S = a/b+c/d, где a,
b, c, d — натуральные числа, удовлетворяющие условиям a+c = 20202, b+d = 20200.
Ответ: M = 20201+1/20199 и m = 1+20201/20199=1/141+20201/20059. Решение. Ответ:
Пусть b  d, тогда 1/d  1/b , а тогда если мы будем «перекидывать» 1 из a в b и обратно, то
чем больше a тем S меньше и чем меньше a тем S больше. Таким образом и для максимуму и
для минимума одно из чисел a и c должно быть 1. То есть и максимум достигается на
1/b+20201/d , а минимум на 20201/b+1/d . Максимум. Если d не равно 1, то есть оба и b и d
больше 1, то сумма a/b+c/d ≤ 20202/2<M. Если d = 1, то получаем S=1/20199+20201 = M. Минимум. S = 20201/b+1/d=1/d+20201/(20200–d) = 20200(d+1)/(d(20200–d)), пусть d+1=x , тогда
минимум достигается, когда у (x–1)(20201–x)/x достигается максимум. (x–1)(20201–
x)/x = 20201–x–20201/x , а здесь максимум, когда у x+20201/x минимум. а он достигается, когда x=142 . То есть d=141 , то есть b=20200–141=20059 и минимум 1/141+20201/20059
2. В каждом из 100 сосудов лежит по 99 камней. Два игрока ходят по очереди. Каждый игрок при своем ходе должен взять по одному камню из 98 сосудов. Игрок, после хода
которого два сосуда оказались пусты, выигрывает. Кто выиграет при правильной игре —
начинающий или его партнер?
Ответ: Второй. Решение. При каждом ходе игрок не берет камней из двух сосудов.
Будем называть эти сосуды пропущенными. Очевидно, пока не сделаны 99 ходов, ни один
сосуд не опустеет, поэтому при каждом из этих 99 ходов делающий ход игрок может сам выбирать, какие два сосуда пропустить. Пусть второй игрок каждым ходом пропускает два сосуда, которые еще не были пропущены (если такой остался только один — пропускает его и
любой другой; если таких вообще не осталось — делает любой ход). Тогда после 99 ходов
все сосуды будут пропущены хотя бы один раз (так как два сосуда пропустил первый игрок
начальным ходом, а потом второй игрок сделал еще 49 ходов, при каждом из которых он мог
увеличивать количество пропущенных сосудов на 2, пока оно не достигло 100).
После 99 ходов сосуд, который был пропущен хоть раз, будет содержать хотя бы один
камень. Поэтому после 99 ходов в наших 100 сосудах останется 99(100-99)98 = 198 камней,
причем в каждом сосуде будет не менее одного камня. Отсюда следует, что хотя бы в двух
сосудах будет ровно по одному камню (иначе общее число камней не меньше 1+299=199), и
сотым ходом второй игрок выиграет.
3. M и N — середины сторон CD и AD выпуклого четырехугольника ABCD соответственно. Перпендикуляр к AB, проведенный через M, и перпендикуляр к BC, проведенный через N, пересекаются в точке P. Докажите, что P лежит на диагонали BD тогда и только
тогда, когда диагонали AC и BD перпендикулярны.
Решение. Проведем через точки A и C перпендикуляры к BC и AB соответственно, а
точку их пересечения обозначим за Q. Легко видеть, что точка P лежит на прямой BD тогда и
только тогда, когда на ней лежит точка Q. Точка Q является ортоцентром треугольника ABC,
следовательно прямая BQ перпендикулярна AC. Поэтому BQ совпадает с BD тогда и только
тогда, когда BD перпендикулярна AC.
4. По кругу записаны в порядке возрастания по часовой стрелке числа от 1 до 31.
Разрешается взять любые три числа a, b, c (не обязательно соседние и стоящие в любом
порядке) и заменить их числами c, a–1/10, и b+1/10 соответственно. Докажите, что, применяя такие операции, можно добиться, чтобы числа стояли в порядке возрастания против часовой стрелки.
Ответ: Решение.
5. В трапеции ABCD с основаниями BC и AD биссектриса угла CAD проходит через
середину отрезка BD. Известно, что BD = 2AB. Докажите, что AD = 3BC.
92
Решение. Пусть K, L, M — середины отрезков ABL, AC и BD соответственно. Длины
сторон AD и BC обозначим через a и b. Поскольку LMA = MAD = CAM, получаем, что
AL = LM. А поскольку AB = BM, получаем, что AL — медиана в треугольнике ABM. Но MK
— тоже медиана в этом треугольнике, значит K — точка пересечения медиан. Следовательно, LK/MK = 1:3. Поскольку LK = BC/2, а MK = AD/2, получаем утверждение задачи.
6. Найдите все такие натуральные m, n и простые p, что число
7 m  2n p
— тоже
7 m  2n p
натуральное.
Ответ: m = 1, n = 1, p = 3; m = 2, n = 4, p = 3. Решение. Прибавляя к нашей дроби 1,
узнаем, что число d = 7m–p2n — делитель числа 27m, а вычитая — что делитель числа p2n+1.
Поскольку d, очевидно, нечетно, оно является общим делителем 7m и p. Таким образом, возможны два случая: d = 1 или d = p = 7. В первом случае 7m–1 = p2n делится на 3, поэтому
p = 3. При этом если n = 1, то m = 1, а если n > 1, то 7m–1 кратно 4, поэтому m = 2k, (7k–
1)(7k+1) = 32n. В левой части одна из скобок не делится на 4 и поэтому не превосходит 6,
значит, k = 1, откуда m = 2 и n = 4. Во втором случае 7m–72n = 7, поэтому 7m–1–1 = 2n, что невозможно, так как левая часть делится на 3.
7. Из чисел 1, 2, ..., 37 произвольным образом выбраны 11 различных. Докажите, что
из этих 11 чисел можно выбрать четыре таких, что сумма двух из них равна сумме двух
других.
Решение. a+b = c+d  a–c = b–d. Поэтому достаточно найти две пары из четырех различных чисел такие, что разности большего и меньшего чисел в парах одинаковы. Всего различных разностей возможно 36: от 1 до 36, а различных пар из 11 чисел можно составить 55.
Поэтому среди таких пар есть пары с равными разностями. Две пары с равными разностями
не дают решения задачи только если большее число одной пары равно меньшему числу другой, то есть пары составлены из чисел, входящих в трехчленную арифметическую прогрессию. Очевидно, что если есть две различные трехчленные прогрессии или прогрессия и не
входящая в нее пара с равными разностями — все в порядке. Если же нет, то трехчленных
прогрессий k ≤ 18, а четырехчленных прогрессий нет. Поэтому остается 55–2k «одиночных»
пар, у которых разности принимают не более 36–k различных значений. Поскольку при
k ≤ 18 имеем 55–2k > 36–k, среди «одиночных» пар найдутся две с равными разностями.
8. Таблица 5050 заполнена натуральными числами таким образом, что все суммы
чисел по строкам и столбцам равны. Какое наименьшее количество чисел надо изменить,
чтобы все эти 100 сумм стали попарно различны?
Ответ: 66. Решение. Оценка. Построим двудольный граф G, вершинами которого будут столбцы и строчки, а рёбра соответствуют тем числам, которые мы изменяем. Количество рёбер, очевидно, не превосходит количества вершин (то есть 100) минус количество
компонент связности. Рассмотрим компоненты связности этого графа. Пусть одна них состоит из двух вершин. Это означает, что есть строчка и столбец, в которых изменили ровно по
одному числу, причём это число стоит на их пересечении. Но тогда и после изменения суммы чисел в них равны, противоречие. Итак, в графе нет компонент связности из двух вершин. Есть не более одной компоненты связности из одной вершины, так как такая компонента связности соответствует строке или столбцу, в которой не изменено ни одно число. Следовательно, 99 вершин нашего графа разбиваются на компоненты связности из хотя бы трёх
вершин, таких компонент связности не более 33. Итого мы имеем не более 34 компонент
связности, а значит, не менее 100–34 = 66 рёбер. Пример. Выделим один столбец, в которым
ничего не меняется, и разобьём остальные 49 столбцов и 50 строчек на 33 тройки из столбца
и двух строк или строки и двух столбцов — это сделать несложно. Рассмотрим одну тройку
— пусть это строка и два столбца. Изменим числа в двух клетках на их пересечении, приба-
93
вив к ним 10a и 10b. Все 66 показателей степеней во всех тройках сделаем разными, что обеспечит различные изменения.
Старшая группа, первая лига, 4 тур, решения и указания для жюри.
1. Найдите наибольшее и наименьшее возможное значение суммы S = a/b+c/d, где a,
b, c, d — натуральные числа, удовлетворяющие условиям a+c = 20202, b+d = 20200.
Ответ: M = 20201+1/20199 и m = 1+20201/20199=1/141+20201/20059. Решение. Ответ:
Пусть b  d, тогда 1/d  1/b , а тогда если мы будем «перекидывать» 1 из a в b и обратно, то
чем больше a тем S меньше и чем меньше a тем S больше. Таким образом и для максимуму и
для минимума одно из чисел a и c должно быть 1. То есть и максимум достигается на
1/b+20201/d , а минимум на 20201/b+1/d . Максимум. Если d не равно 1, то есть оба и b и d
больше 1, то сумма a/b+c/d ≤ 20202/2<M. Если d = 1, то получаем S=1/20199+20201 = M. Минимум. S = 20201/b+1/d=1/d+20201/(20200–d) = 20200(d+1)/(d(20200–d)), пусть d+1=x , тогда
минимум достигается, когда у (x–1)(20201–x)/x достигается максимум. (x–1)(20201–
x)/x = 20201–x–20201/x , а здесь максимум, когда у x+20201/x минимум. а он достигается, когда x=142 . То есть d=141 , то есть b=20200–141=20059 и минимум 1/141+20201/200592. В
каждом из 100 сосудов лежит по 100 камней. Два игрока ходят по очереди. Каждый игрок
при своем ходе должен взять по одному камню из 98 сосудов. Игрок, после хода которого
два сосуда оказались пусты, выигрывает. Кто выиграет при правильной игре — начинающий или его партнер?
Ответ: Второй. Решение. Пусть второй игрок повторяет ходы первого (то есть каждым
ходом берет камни из тех сосудов, из которых их только что брал первый). Очевидно, при
такой стратегии второго игрока после каждого его хода количество камней во всех сосудах
оказывается четным, а после каждого хода первого игрока количество камней в тех сосудах,
из которых он берет камни, оказывается нечетным (следовательно, не 0). Поэтому первый
игрок выиграть не может. С другой стороны, игра закончится не более чем за 102 хода, поэтому второй выиграет.
3. Биссектрисы углов A и C треугольника ABC пересекаются в точке I. Известно,
что AI = BC и ICA = 2IAC. Найдите угол ABC.
Ответ: 60. Решение. Пусть CD биссектриса угла C, а E такая тока на CD, что AE = AI.
Заметим что AD = DC. Легко видеть, что один из треугольников AID и AED должен быть равен треугольнику CDB (поскольку у них равна пара сторон и угол между одной из этих сторон и прямой содержащей третью сторону). Очевидно, это должен быть треугольник AED.
Поэтому, AED = ABC = 180–3CAB. С другой стороны, в силу равнобедренности треугольника AED, AED = AIE = 3CAB/2. Таким образом 180–3CAB = 3CAB/2. Следовательно CAB = 40, а ABC = 60.
4. Известно, что
a
b
c
(a  b)(b  c)(c  a) 1


.
 . Найдите
ab bc ca
(a  b)(b  c)(c  a) 11
Ответ: Решение.
5. В трапеции ABCD с основаниями BC и AD биссектриса угла CAD проходит через
середину отрезка BD. Известно, что BD = 2AB. Докажите, что AD = 3BC.
Решение. Пусть K, L, M — середины отрезков ABL, AC и BD соответственно. Длины
сторон AD и BC обозначим через a и b. Поскольку LMA = MAD = CAM, получаем, что
AL = LM. А поскольку AB = BM, получаем, что AL — медиана в треугольнике ABM. Но MK
— тоже медиана в этом треугольнике, значит K — точка пересечения медиан. Следовательно, LK/MK = 1:3. Поскольку LK = BC/2, а MK = AD/2, получаем утверждение задачи.
94
6. Найдите все такие натуральные m, n и простые p, что число
7 m  2n p
— тоже
7 m  2n p
натуральное.
Ответ: m = 1, n = 1, p = 3; m = 2, n = 4, p = 3. Решение. Прибавляя к нашей дроби 1,
узнаем, что число d = 7m–p2n — делитель числа 27m, а вычитая — что делитель числа p2n+1.
Поскольку d, очевидно, нечетно, оно является общим делителем 7m и p. Таким образом, возможны два случая: d = 1 или d = p = 7. В первом случае 7m–1 = p2n делится на 3, поэтому
p = 3. При этом если n = 1, то m = 1, а если n > 1, то 7m–1 кратно 4, поэтому m = 2k, (7k–
1)(7k+1) = 32n. В левой части одна из скобок не делится на 4 и поэтому не превосходит 6,
значит, k = 1, откуда m = 2 и n = 4. Во втором случае 7m–72n = 7, поэтому 7m–1–1 = 2n, что невозможно, так как левая часть делится на 3.
7. Из чисел 1, 2, ..., 37 произвольным образом выбраны 13 различных. Докажите, что
из этих 13 чисел можно выбрать четыре таких, что сумма двух из них равна сумме двух
других.
Решение. Из 13 различных чисел можно составить 78 пар. Каждая из 78 их сумм не
превосходит 37+36 = 73. Поэтому найдутся две различные пары с одинаковыми суммами.
Легко видеть, что эти пары состоят из четырех разных чисел.
8. Таблица 5050 заполнена натуральными числами таким образом, что все суммы
чисел по строкам и столбцам равны. Какое наименьшее количество чисел надо изменить,
чтобы все эти 100 сумм стали попарно различны?
Ответ: 66. Решение. Оценка. Построим двудольный граф G, вершинами которого будут столбцы и строчки, а рёбра соответствуют тем числам, которые мы изменяем. Количество рёбер, очевидно, не превосходит количества вершин (то есть 100) минус количество
компонент связности. Рассмотрим компоненты связности этого графа. Пусть одна них состоит из двух вершин. Это означает, что есть строчка и столбец, в которых изменили ровно по
одному числу, причём это число стоит на их пересечении. Но тогда и после изменения суммы чисел в них равны, противоречие. Итак, в графе нет компонент связности из двух вершин. Есть не более одной компоненты связности из одной вершины, так как такая компонента связности соответствует строке или столбцу, в которой не изменено ни одно число. Следовательно, 99 вершин нашего графа разбиваются на компоненты связности из хотя бы трёх
вершин, таких компонент связности не более 33. Итого мы имеем не более 34 компонент
связности, а значит, не менее 100–34 = 66 рёбер. Пример. Выделим один столбец, в которым
ничего не меняется, и разобьём остальные 49 столбцов и 50 строчек на 33 тройки из столбца
и двух строк или строки и двух столбцов — это сделать несложно. Рассмотрим одну тройку
— пусть это строка и два столбца. Изменим числа в двух клетках на их пересечении, прибавив к ним 10a и 10b. Все 66 показателей степеней во всех тройках сделаем разными, что обеспечит различные изменения.
Старшая группа, вторая лига, 4 тур, решения и указания для жюри.
1. Найдите наибольшее и наименьшее возможное значение суммы S = a/b+c/d, где a,
b, c, d — натуральные числа, удовлетворяющие условиям a+c = 20202, b+d = 20200.
Ответ: M = 20201+1/20199 и m = 1+20201/20199=1/141+20201/20059. Решение. Ответ:
Пусть b  d, тогда 1/d  1/b , а тогда если мы будем «перекидывать» 1 из a в b и обратно, то
чем больше a тем S меньше и чем меньше a тем S больше. Таким образом и для максимуму и
для минимума одно из чисел a и c должно быть 1. То есть и максимум достигается на
1/b+20201/d , а минимум на 20201/b+1/d . Максимум. Если d не равно 1, то есть оба и b и d
больше 1, то сумма a/b+c/d ≤ 20202/2<M. Если d = 1, то получаем S=1/20199+20201 = M. Минимум. S = 20201/b+1/d=1/d+20201/(20200–d) = 20200(d+1)/(d(20200–d)), пусть d+1=x , тогда
минимум достигается, когда у (x–1)(20201–x)/x достигается максимум. (x–1)(20201–
95
x)/x = 20201–x–20201/x , а здесь максимум, когда у x+20201/x минимум. а он достигается, когда x=142 . То есть d=141 , то есть b=20200–141=20059 и минимум 1/141+20201/20059
2. В каждом из 100 сосудов лежит по 100 камней. Два игрока ходят по очереди.
Каждый игрок при своем ходе должен взять по одному камню из 98 сосудов. Игрок, после
хода которого два сосуда оказались пусты, выигрывает. Кто выиграет при правильной игре
— начинающий или его партнер?
Ответ: Второй. Решение. Пусть второй игрок повторяет ходы первого (то есть каждым
ходом берет камни из тех сосудов, из которых их только что брал первый). Очевидно, при
такой стратегии второго игрока после каждого его хода количество камней во всех сосудах
оказывается четным, а после каждого хода первого игрока количество камней в тех сосудах,
из которых он берет камни, оказывается нечетным (следовательно, не 0). Поэтому первый
игрок выиграть не может. С другой стороны, игра закончится не более чем за 102 хода, поэтому второй выиграет.
3. На стороне AC прямоугольного треугольника ABC с прямым углом B взята точка
D такая, что CD = AB, а на стороне BC — точка E такая, что DB = DE. Известно, что
CAB = 2ABD. Найдите угол EDC.
Ответ: EDC = 22,5. Решение. Пусть DH — высота треугольника BDE. Тогда
CAB = HDC и ABD = BDH = HDE = EDC. Следовательно, треугольники ABD и
EDC равны по двум сторонам и углу между ними. Значит C = A = 45, а EDC = 22,5.
4. Известно, что
a
b
c
(a  b)(b  c)(c  a) 1


.
 . Найдите
ab bc ca
(a  b)(b  c)(c  a) 11
Ответ: Решение.
5. Дана трапеция ABCD с основаниями BC и AD. Точка K — середина диагонали BD.
Оказалось, что AK — биссектриса угла CAD и AD = 3BC. Докажите, что AC = 2BC.
Решение. Пусть K, L, M — середины отрезков ABL, AC и BD соответственно. Длины
сторон AD и BC обозначим через a и b. Поскольку LMA = MAD = CAM, получаем, что
AL = LM. А поскольку AB = BM, получаем, что AL — медиана в треугольнике ABM. Но MK
— тоже медиана в этом треугольнике, значит K — точка пересечения медиан. Следовательно, LK/MK = 1:3. Поскольку LK = BC/2, а MK = AD/2, получаем утверждение задачи.
6. Найдите все такие натуральные m и простые p, что число
7m  2 p
— тоже
7m  2 p
натуральное.
Ответ: m = 1, p = 3. Решение. Прибавляя к нашей дроби 1, узнаем, что число d = 7m–2p
— делитель числа 27m, а вычитая — что делитель числа 4p. Поскольку d, очевидно, нечетно,
оно является общим делителем 7m и p. Таким образом, возможны два случая: d = 1 или
d = p = 7. В первом случае 7m–1 = 2p делится на 3, поэтому p = 3, откуда m = 1 и n = 1. Во
втором случае 7m–14 = 7, поэтому 7m = 21, что невозможно.
7. Из чисел 1, 2, ..., 37 произвольным образом выбраны 13 различных. Докажите, что
из этих 13 чисел можно выбрать четыре таких, что сумма двух из них равна сумме двух
других.
Решение. Из 13 различных чисел можно составить 78 пар. Каждая из 78 их сумм не
превосходит 37+36 = 73. Поэтому найдутся две различные пары с одинаковыми суммами.
Легко видеть, что эти пары состоят из четырех разных чисел.
96
8. Сколькими способами можно раскрасить клетки прямоугольника 22011 в два
цвета так, чтобы никакие три клетки одного цвета не образовывали уголок из трех клеток?
Ответ: Решение.
Младшая группа, высшая лига, 4 тур, решения и указания для жюри.
1. Найдите наибольшее число, делящееся на 72, которое можно получить из числа
123...20092010 (все натуральные числа от 1 до 2010 записаны подряд) вычеркиванием некоторых цифр.
Ответ: 12346…2009200 (вычеркнуты первая пятерка и последняя единица). Решение.
Сумма всех цифр числа из условия дает при делении на 9 остаток 6. Поэтому сумма вычеркнутых цифр должна быть не меньше 6. Одной из вычеркнутых должна быть стоящая на
предпоследнем месте единица: число, оканчивающееся на 10 или 1, не делится на 8. Поэтому
вычеркивать надо две цифры (в других случаях у числа будет меньше разрядов, и оно заведомо не будет наибольшим). Стало быть, надо вычеркивать пятерку. Если вычеркнуть
первую пятерку, в пятом по старшинству разряде будет шестерка, иначе — пятерка. Отсюда
получаем ответ.
 Ответ без обоснования — 0 баллов.
2. Таблица 5050 заполнена натуральными числами таким образом, что все суммы
чисел по строкам и столбцам равны. Какое наименьшее количество чисел надо изменить,
чтобы все эти 100 сумм стали попарно различны?
Ответ: 66. Решение. Оценка. Построим двудольный граф G, вершинами которого будут столбцы и строчки, а рёбра соответствуют тем числам, которые мы изменяем. Количество рёбер, очевидно, не меньше, чем количество вершин (то есть 100) минус количество
компонент связности. Рассмотрим компоненты связности этого графа. Пусть одна них состоит из двух вершин. Это означает, что есть строчка и столбец, в которых изменили ровно по
одному числу, причём это число стоит на их пересечении. Но тогда и после изменения суммы чисел в них равны, противоречие. Итак, в графе нет компонент связности из двух вершин. Есть не более одной компоненты связности из одной вершины, так как такая компонента связности соответствует строке или столбцу, в которой не изменено ни одно число. Следовательно, 99 вершин нашего графа разбиваются на компоненты связности из хотя бы трёх
вершин, таких компонент связности не более 33. Итого мы имеем не более 34 компонент
связности, а значит, не менее 100–34 = 66 рёбер. Пример. Выделим один столбец, в которым
ничего не меняется, и разобьём остальные 49 столбцов и 50 строчек на 33 тройки из столбца
и двух строк или строки и двух столбцов — это сделать несложно. Рассмотрим одну тройку
— пусть это строка и два столбца. Изменим числа в двух клетках на их пересечении, прибавив к ним 10a и 10b. Все 66 показателей степеней во всех тройках сделаем разными, что обеспечит различные изменения.
 Оценка — 6 баллов, пример — 4 балла.
3. Для любых целых чисел a, b, c и d докажите неравенство a 2+b2+c2+3d2  (2d–
1)(a+b+c)+3d.
Решение. Соберем все члены неравенства в его левой части. Тогда его, как легко проверить, можно привести к виду (a–d)(a–d+1)+(b–d)(b–d+1)+(c–d)(c–d+1)  0. Теперь достаточно заметить, что при целых a, b, c и d все три произведения в левой части неотрицательны.
4. Из чисел 1, 2, ..., 37 произвольным образом выбраны 11 различных. Докажите, что
из этих 11 чисел можно выбрать четыре таких, что сумма двух из них равна сумме двух
других.
97
Решение. a+b = c+d  a–c = b–d. Поэтому достаточно найти две пары из четырех различных чисел такие, что разности большего и меньшего чисел в парах одинаковы. Всего различных разностей возможно 36: от 1 до 36, а различных пар из 11 чисел можно составить 55.
Поэтому среди таких пар есть пары с равными разностями. Две пары с равными разностями
не дают решения задачи только если большее число одной пары равно меньшему числу другой, то есть пары составлены из чисел, входящих в трехчленную арифметическую прогрессию. Очевидно, что если есть две различные трехчленные прогрессии или прогрессия и не
входящая в нее пара с равными разностями — все в порядке. Если же нет, то трехчленных
прогрессий k ≤ 18, а четырехчленных прогрессий нет. Поэтому остается 55–2k «одиночных»
пар, у которых разности принимают не более 36–k различных значений. Поскольку при
k ≤ 18 имеем 55–2k > 36–k, среди «одиночных» пар найдутся две с равными разностями.
5. Дана трапеция ABCD с основаниями BC и AD. Точка K — середина диагонали BD.
Оказалось, что AK — биссектриса угла CAD и AD = 3BC. Докажите, что AC = 2BC.
Решение. Продолжим биссектрису AK до пересечения с прямой BC в точке M. Тогда
BMA = MAD = CAM, так что AC = CM. Кроме того, BDA = DBM, BKM = DKA и
KB = KD, так что треугольники AKD и MKB равны, откуда BM = AD = 3BC. Значит,
AC = CM = BM–BC = AD–BC = 2BC.
6. В каждом из 100 сосудов лежит по 97 камней. Два игрока ходят по очереди. Каждый игрок при своем ходе должен взять по одному камню из 98 сосудов. Игрок, после хода
которого два сосуда оказались пусты, выигрывает. Кто выиграет при правильной игре —
начинающий или его партнер?
Ответ: Первый. Решение. При каждом ходе игрок не берет камней из двух сосудов.
Будем называть эти сосуды пропущенными. Очевидно, менее чем за 97 ходов ни один сосуд
не опустеет и игра не закончится. Покажем, как первому игроку выиграть 97-м ходом. Пусть
первый игрок повторяет ходы второго (то есть после каждого хода второго берет камни из
тех сосудов, из которых их только что брал второй). При такой стратегии первого игрока его
ход не увеличивает количество сосудов, которые хоть раз были пропущены. Поэтому за 97
ходов пропущены окажутся не более 98 сосудов (два будут пропущены начальным ходом
первого игрока и еще 96 при 48 ходах второго и повторяющих их ходах первого). Это значит,
что есть по крайней мере два сосуда, из которых брали по камню при каждом ходе. Теперь
эти сосуды пусты, и первый игрок победил.
7. Аня, Боря, Вика и Галя навещали Диму. Договорились они прийти в один день, но в
разное время и пробыли у Димы по 15 минут. Аня посетила Диму в 8 часов, Боря пришел в 9
часов, Вика — в 10 часов, а Галя пришла в 11 часов, только неизвестно кто из них приходил
утром, а кто вечером. Известно, что: 1) К Диме кто-то приходил между Аней и Борей; 2)
Какая-то из девочек пришла к Диме до Ани; 3) Вика не заходила к Диме между Борей и Галей. Определите кто в какое время приходил к Диме.
Ответ: Боря был в 9 утра, Галя — в 11 утра, Аня — в 8 вечера и Вика — в 10 вечера.
Решение. Поскольку кто-то приходил к Диме перед Аней, она была у него вечером. Но ктото заходил к Диме между Аней и Борей, значит, Боря был утром. Далее, Вика не могла прийти к Диме сразу после Бори, так как в этом случае она оказывается между Борей и Галей. Таким образом, она пришла вечером. Наконец, кто-то был у Димы между Борей и Аней. Это
могла быть только Галя. Стало быть, порядок, в котором были гости у Димы такой: Боря, Галя, Аня и Вика. Значит, Боря был в 9 утра, Галя — в 11 утра, Аня — в 8 вечера и Вика — в 10
вечера.
8. Найдите все простые числа p такие, что число (p2–23)(9p+5) — точный квадрат.
Ответ: 5. Решение. Проверяем, что при p, не делящемся на 5, число (p2–23)(9p+5) дает
при делении на 5 только остатки 2 и 3, которые квадраты давать не могут.
98
Младшая группа, первая лига, 4 тур, решения и указания для жюри.
1. Найдите наибольшее число, делящееся на 72, которое можно получить из числа
123...20092010 (все натуральные числа от 1 до 2010 записаны подряд) вычеркиванием некоторых цифр.
Ответ: 12346…2009200 (вычеркнуты первая пятерка и последняя единица). Решение.
Сумма всех цифр числа из условия дает при делении на 9 остаток 6. Поэтому сумма вычеркнутых цифр должна быть не меньше 6. Одной из вычеркнутых должна быть стоящая на
предпоследнем месте единица: число, оканчивающееся на 10 или 1, не делится на 8. Поэтому
вычеркивать надо две цифры (в других случаях у числа будет меньше разрядов, и оно заведомо не будет наибольшим). Стало быть, надо вычеркивать пятерку. Если вычеркнуть
первую пятерку, в пятом по старшинству разряде будет шестерка, иначе — пятерка. Отсюда
получаем ответ.
 Ответ без обоснования — 2 балла.
2. Таблица 4848 заполнена натуральными числами таким образом, что все суммы
чисел по строкам и столбцам равны. Какое наименьшее количество чисел надо изменить,
чтобы все эти 96 сумм стали попарно различны?
Ответ: 64. Решение. Оценка. Построим двудольный граф G, вершинами которого будут столбцы и строчки, а рёбра соответствуют тем числам, которые мы изменяем. Количество рёбер, очевидно, не меньше, чем количество вершин (то есть 96) минус количество
компонент связности. Рассмотрим компоненты связности этого графа. Пусть одна них состоит из двух вершин. Это означает, что есть строчка и столбец, в которых изменили ровно по
одному числу, причём это число стоит на их пересечении. Но тогда и после изменения суммы чисел в них равны, противоречие. Итак, в графе нет компонент связности из двух вершин. Есть не более одной компоненты связности из одной вершины, так как такая компонента связности соответствует строке или столбцу, в которой не изменено ни одно число. Пусть
такая компонента есть. Тогда остальные 95 вершин нашего графа разбиваются на компоненты связности из хотя бы трёх вершин, таких компонент связности не более 31, а всего компонент не более 32. Если же компоненты из одной вершины нет, тоже получается, что компонент не более, чем 96:3 = 32. Итого мы имеем не менее 96–32 = 64 рёбер. Пример. Разобьём все столбцы и строки на 32 тройки из столбца и двух строк или строки и двух столбцов —
это сделать несложно. Рассмотрим одну тройку — пусть это строка и два столбца. Изменим
числа в двух клетках на их пересечении, прибавив к ним 10 a и 10b. Все 66 показателей степеней во всех тройках сделаем разными, что обеспечит различные изменения.
 Оценка — 6 баллов, пример — 4 балла.
3. Для любых целых чисел a, b и c докажите неравенство a2+b2+2c2  (2c–1)(a+b)+2c.
Решение. Соберем все члены неравенства в его левой части. Тогда его, как легко проверить, можно привести к виду (a–c)(a–c+1)+(b–c)(b–c+1)  0. Теперь достаточно заметить,
что при целых a, b, c и d оба произведения в левой части неотрицательны.
4. Из чисел 1, 2, ..., 37 произвольным образом выбраны 13 различных. Докажите, что
из этих 13 чисел можно выбрать четыре таких, что сумма двух из них равна сумме двух
других.
Решение. Из 13 различных чисел можно составить 78 пар. Каждая из 78 их сумм не
превосходит 37+36 = 73. Поэтому найдутся две различные пары с одинаковыми суммами.
Легко видеть, что эти пары состоят из четырех разных чисел.
5. Дана трапеция ABCD с основаниями BC и AD. Точка K — середина диагонали BD.
Оказалось, что AK — биссектриса угла CAD и AD = 3BC. Докажите, что AC = 2BC.
99
Решение. Продолжим биссектрису AK до пересечения с прямой BC в точке M. Тогда
BMA = MAD = CAM, так что AC = CM. Кроме того, BDA = DBM, BKM = DKA и
KB = KD, так что треугольники AKD и MKB равны, откуда BM = AD = 3BC. Значит,
AC = CM = BM–BC = AD–BC = 2BC.
6. Дан прямоугольник 22012 (2 строки, 2012 столбцов). Два игрока ходят по очереди. Первый своим ходом вычеркивает две соседние по вертикали клетки, а второй вычеркивает две соседние по горизонтали клетки. Вычеркивать ранее вычеркнутые клетки нельзя.
Игрок, не имеющий хода, проигрывает. Кто выигрывает при правильной игре?
Ответ: Второй. Решение. Второй мысленно делит прямоугольник на 1006 квадратов
22 и каждым своим ходом из первых 503-х выбрасывает две клетки каждый раз из нового
квадрата. Очевидно, первый не сможет этому помешать, так как каждым своим ходом «портит» не больше одного нового квадрата. Следующими 503 ходами второй выбрасывает
оставшиеся клетки из «занятых» им квадратов. Играя так, он сумеет сделать не меньше 1006
ходов. Но общее число пар ходов в партии не больше 2012:2 = 1006, так что последний ход
останется за вторым.
7. Аня, Боря, Вика и Галя навещали Диму. Договорились они прийти в один день, но в
разное время и пробыли у Димы по 15 минут. Аня посетила Диму в 8 часов, Боря пришел в 9
часов, Вика — в 10 часов, а Галя пришла в 11 часов, только неизвестно кто из них приходил
утром, а кто вечером. Известно, что: 1) К Диме кто-то приходил между Аней и Борей; 2)
Какая-то из девочек пришла к Диме до Ани; 3) Вика не заходила к Диме между Борей и Галей. Определите кто в какое время приходил к Диме.
Ответ: Боря был в 9 утра, Галя — в 11 утра, Аня — в 8 вечера и Вика — в 10 вечера.
Решение. Поскольку кто-то приходил к Диме перед Аней, она была у него вечером. Но ктото заходил к Диме между Аней и Борей, значит, Боря был утром. Далее, Вика не могла прийти к Диме сразу после Бори, так как в этом случае она оказывается между Борей и Галей. Таким образом, она пришла вечером. Наконец, кто-то был у Димы между Борей и Аней. Это
могла быть только Галя. Стало быть, порядок, в котором были гости у Димы такой: Боря, Галя, Аня и Вика. Значит, Боря был в 9 утра, Галя — в 11 утра, Аня — в 8 вечера и Вика — в 10
вечера.
8. Найдите все простые числа p такие, что число (p2–23)(9p+5) — точный квадрат.
Ответ: 5. Решение. Проверяем, что при p, не делящемся на 5, число (p2–23)(9p+5) дает
при делении на 5 только остатки 2 и 3, которые квадраты давать не могут.
Младшая группа, вторая лига, 4 тур, решения и указания для жюри.
1. Найдите наибольшее число, делящееся на 72, которое можно получить из числа
123...20092010 (все натуральные числа от 1 до 2010 записаны подряд) вычеркиванием некоторых цифр.
Ответ: 12346…2009200 (вычеркнуты первая пятерка и последняя единица). Решение.
Сумма всех цифр числа из условия дает при делении на 9 остаток 6. Поэтому сумма вычеркнутых цифр должна быть не меньше 6. Одной из вычеркнутых должна быть стоящая на
предпоследнем месте единица: число, оканчивающееся на 10 или 1, не делится на 8. Поэтому
вычеркивать надо две цифры (в других случаях у числа будет меньше разрядов, и оно заведомо не будет наибольшим). Стало быть, надо вычеркивать пятерку. Если вычеркнуть
первую пятерку, в пятом по старшинству разряде будет шестерка, иначе — пятерка. Отсюда
получаем ответ.
 Ответ без обоснования — 2 балла.
2. 100 депутатов Думы пользуются сетью Билайн, а 200 депутатов – сетью Мегафон. За внутрисетевой звонок Билайн берёт 43 копейки, а Мегафон меньше, но целое число
100
копеек. За звонок в другую сеть стоимость звонка возрастает в 3 раза. Все входящие звонки
бесплатные. В течение дня каждый депутат звонит каждому по одному разу и от каждого
один раз получает встречный звонок. Сколько стоят звонки с Мегафона, если его ежедневные доходы с депутатов более чем на 11000 рублей превышают доходы Билайна?
Ответ: 42 коп. за минуту. Решение. Билайн получает доход с внутрисетевых звонков:
4310099 = 4257 руб. Со звонков на Мегафон: 343100200 = 25800 руб. Всего 30057 руб. Если бы цена звонка Мегафона была 42 коп. за звонок, то его доход составил бы
42200199+342100200 = 16716+25200 = 41916, что соответствует условию задачи (разница
превышает 11 тысяч рублей). Если же Мегафон снизит цену хотя бы на копейку, то доход
составит 41200199+341100200 = 16318+24600 = 40918, что уже не соответствует условиям.
 Ответ без обоснования — 0 баллов.
3. Числа a, b, c и d удовлетворяют условию a+d = b+c. Докажите неравенство
(a–b)(c–d)+(a–c)(b–d)+(d–a)(b–c)  0.
Решение. Раскрыв в левой части все скобки, после приведения подобных и деления на
2 получаем ab+ac–ad+bd–bc = a(b+c–d)+b(d–c) = a2+b(b–a) = a2+b2–ab  0.
4. Из чисел 1, 2, ..., 37 произвольным образом выбраны 13 различных. Докажите, что
из этих 13 чисел можно выбрать четыре таких, что сумма двух из них равна сумме двух
других.
Решение. Из 13 различных чисел можно составить 78 пар. Каждая из 78 их сумм не
превосходит 37+36 = 73. Поэтому найдутся две различные пары с одинаковыми суммами.
Легко видеть, что эти пары состоят из четырех разных чисел.
5. На диагонали BD выпуклого четырехугольника ABCD отмечена точка E. Известно,
что AB = CE, ABD = BCE = ECD и DAB = ABC. Докажите, что треугольник BCD
равнобедренный.
Решение. Заметим, что CED = ECB + CBE = EBA + CBE = CBA = BAD.
Следовательно, треугольники ABD и ECD равны по двум углам и стороне. Стало быть,
BD = CD.
6. Дан прямоугольник 22012 (2 строки, 2012 столбцов). Два игрока ходят по очереди. Первый своим ходом вычеркивает две соседние по вертикали клетки, а второй вычеркивает две соседние по горизонтали клетки. Вычеркивать ранее вычеркнутые клетки нельзя.
Игрок, не имеющий хода, проигрывает. Кто выигрывает при правильной игре?
Ответ: Второй. Решение. Второй мысленно делит прямоугольник на 1006 квадратов
22 и каждым своим ходом из первых 503-х выбрасывает две клетки каждый раз из нового
квадрата. Очевидно, первый не сможет этому помешать, так как каждым своим ходом «портит» не больше одного нового квадрата. Следующими 503 ходами второй выбрасывает
оставшиеся клетки из «занятых» им квадратов. Играя так, он сумеет сделать не меньше 1006
ходов. Но общее число пар ходов в партии не больше 2012:2 = 1006, так что последний ход
останется за вторым.
7. Аня, Боря, Вика и Галя навещали Диму. Договорились они прийти в один день, но в
разное время и пробыли у Димы по 15 минут. Аня посетила Диму в 8 часов, Боря пришел в 9
часов, Вика — в 10 часов, а Галя пришла в 11 часов, только неизвестно кто из них приходил
утром, а кто вечером. Известно, что: 1) К Диме кто-то приходил между Аней и Борей; 2)
Какая-то из девочек пришла к Диме до Ани; 3) Вика не заходила к Диме между Борей и Галей. Определите кто в какое время приходил к Диме.
101
Ответ: Боря был в 9 утра, Галя — в 11 утра, Аня — в 8 вечера и Вика — в 10 вечера.
Решение. Поскольку кто-то приходил к Диме перед Аней, она была у него вечером. Но ктото заходил к Диме между Аней и Борей, значит, Боря был утром. Далее, Вика не могла прийти к Диме сразу после Бори, так как в этом случае она оказывается между Борей и Галей. Таким образом, она пришла вечером. Наконец, кто-то был у Димы между Борей и Аней. Это
могла быть только Галя. Стало быть, порядок, в котором были гости у Димы такой: Боря, Галя, Аня и Вика. Значит, Боря был в 9 утра, Галя — в 11 утра, Аня — в 8 вечера и Вика — в 10
вечера.
8. Трехзначные числа abc и cba делятся на простое число p. Докажите, что хотя
бы одно из чисел a+b+c, a–b+c или a–c также делится на p.
Решение. Вычитая из первого трехзначного числа второе, получаем число 99(a–c), которое тоже делится на p. Поэтому на p делится либо a–c (и тогда все доказано), либо 99. В
последнем случае p либо 3 (и тогда a+b+c делится на p), либо 11 (и тогда a–b+c делится на
p).
Группа «Старт», высшая лига, 4 тур, решения и указания для жюри.
1. Дано натуральное число n > 100. Докажите, что найдется число, которое меньше
n, больше единицы, и не имеет с числом n(n–1) общих делителей, кроме 1.
Решение. Предположим, это не так. Заметим, что тогда n делится на 2, на 3 и на 5.
Рассмотрим числа n/2–1, n/2–2. Их общие делители с n и n–1 , не превосходят 4, поскольку
если их удвоить, то они будут отличаться от n и n–1 не более, чем на 4. Но они не делятся на
3 и среди них ровно одно четное, поэтому одно из них не будет иметь общих делителей с
n(n–1).
2. В некоторые клетки таблицы 1313 записали числа. Оказалось, что все суммы чисел по строкам и столбцам попарно различны. Какое наибольшее количество клеток могли
остаться свободными?
Ответ: 152. Решение. Пример. Поставить различные степени двойки в клетки на рисунке справа. Оценка. Докажем, что
расстановки 16 чисел недостаточно. Предположим, удалось расставить 16 чисел. Тогда не более одной линии (скажем, строки)
не содержит ни одного числа. Если какое-то число уникально и
в строке и в столбце, то суммы чисел в этих линиях равны. Имеется не менее 10 столбцов и не менее 9 строк, в которых стоит
ровно 1 число. Заметим, что на их пересечениях не может быть чисел. Но тогда все числа,
стоящие в этих строках и столбцах различны, поэтому их не менее 19. Противоречие.
3. Из чисел 1, 2, ..., 100 произвольным образом выбраны 20 различных. Докажите, что
из этих 20 чисел можно выбрать четыре таких, что сумма двух из них равна сумме двух
других.
Решение. a+b = c+d  a–c = b–d. Поэтому достаточно найти две пары из четырех различных чисел такие, что разности большего и меньшего чисел в парах одинаковы. Упорядочим выбранные числа от меньших к большим. Пусть утверждение задачи не выполнено. Тогда все разности между вторым и первым, четвертым и третьим, …, 20-м и 19-м числами
должны быть различны, и потому их сумма не меньше, чем 1+2+…+10 = 55. По той же причине различны разности между третьим и вторым, пятым и четвертым, …., 19-м и 18-м числами, и сумма этих разностей не меньше, чем 1+2+…+9 = 45. Но тогда сумма этих двух сумм
с одной стороны не меньше 100, а с другой она равна разности двадцатого и первого чисел,
то есть не меньше 99. Противоречие.
4. На каждой клетке доски 88 лежит камень. Камни убывают по весу в каждой
строке (слева направо) и каждом столбце (снизу вверх). Разрешается взвесить на весах (по-
102
казывающих вес груза) любой камень (несколько камней сразу взвешивать нельзя). Покажите, как за 15 таких взвешиваний определить, есть ли на доске камень, весящий 100 грамм.
Решение. Сначала взвесим камень, лежащий в правом нижнем углу. Если он весит
больше 100 грамм, то камень весом 100 грамм точно не лежит в правом столбце и этот столбец можно вычеркнуть. Если же меньше, то нужный камень не лежит в нижней строке и вычеркнуть можно ее. В получившейся таблице из 56 клеток снова проделаем ту же операцию
(т. е. взвесим правый нижний камень и вычеркнем соответствующую линию). Если эту операцию проделать не более 15 раз, то не останется ни одной клетки и тогда камня весом в 100
грамм на доске нет. В противном случае при какой-то операции будет равенство, и нужный
камень на доске есть.
5. На доске написано число 2011. Вася может написанное число умножать на 8 или
делить на 8 (если частное — целое число), либо переставлять в нем цифры (0 на первое место ставить нельзя). Может ли Вася такими операциями получить число 1?
Ответ: Нет. Решение. Рассмотрим остаток от деления данного числа на 9. Изначально
он равен 4. При перестановке цифр этот остаток не изменяется. При умножении и делении на
8 остаток 4 превращается в 5, а остаток 5 — в 4. Тогда остаток никогда не сможет стать равны единице.
6. На доску выписано несколько натуральных чисел, не превосходящих 100. Оказалось,
что числа 1 и 2 выписаны и ни одно из выписанных чисел не равно сумме двух различных выписанных чисел. Какое наибольшее количество чисел могло быть выписано?
Ответ: 35. Решение. Пример: 1, 2, 4, 7, 10, 13, …, 100. Если выписано более 35 чисел,
то в одной из 32 троек (5,6,7); (8,9,10); (11,12,13); …; (98,99,100) будет не менее двух выписанных чисел, тогда можно к меньшему прибавить 1 или 2 и получить большее.
7. Как провести на круглом листе бумаги пять отрезков, каждый из которых соединяет две точки на краю листа, чтобы среди частей, на которые эти отрезки делят лист, оказались хотя бы один пятиугольник и хотя бы два четырехугольника?
Решение. См. рисунок справа.
8. В стране несколько городов. Некоторые пары городов соединены дорогами, причем
из любого города в любой другой можно добраться по этим дорогам. Оказалось, что из двух
городов выходят по две дороги, а из остальных — по три или по одной. Докажите, что
можно закрыть несколько дорог на ремонт таким образом, чтобы из каждого города выходило по три или по одной дороге.
Решение. Заметим, что по условию существует какой-то путь из одного города, из которого выходит 2 дороги в другой такой город. Есть закрыть все дороги этого пути, то изменится четность числа выходящих дорог только в этих города, поэтому количество дорог, выходящих из каждого города, станет нечетным.
Группа «Старт», первая лига, 4 тур, решения и указания для жюри.
1. Назовём натуральное число n, большее 3, сложным, если с каждым натуральным
числом, меньшими n, кроме 1 и n–1, у него есть общий простой делитель. Найдите все
сложные числа, меньшие 1000
Ответ: 6. Решение. Числа от 4 до 11 перебираются. Если же сложное число больше 11,
оно должно делиться на 2, 3, 5, 7 и 11, а произведение этих простых чисел больше 1000.
2. В некоторые клетки таблицы 55 записали числа. Оказалось, что все суммы чисел
по строкам и столбцам попарно различны. Какое наибольшее количество клеток могли
остаться свободными?
103
Ответ: 19. Решение. Пример на рисунке. В отмеченные клетки расставлены различные
степени двойки. Докажем, что расстановки 5 чисел недостаточно. Предположим,
удалось расставить 5 чисел. Тогда не более одной линии (скажем, строки) не содержит ни одного числа. Если какое-то число уникально и в строке и в столбце, то суммы чисел в этих линиях равны. Имеется не менее 4 столбцов и не менее 3 строк, в
которых стоит ровно одно число. Заметим, что на их пересечениях не может быть чисел. Но
тогда все числа, стоящие в этих строках и столбцах различны, поэтому их не менее 7. Противоречие.
3. Определите максимально возможное число, делящееся на 6, которое может быть
получено из числа 123…200920102011 вычеркиванием некоторых из его цифр.
Ответ: 1345…201020 (вычеркнуты первая двойка и две последние единицы). Решение.
Две последние единицы над вычеркнуть обязательно, иначе число будет нечетным. После
этого сумма всех цифр оставшегося числа будет давать при делении на 3 остаток 2. Поэтому
придется вычеркнуть еще одну цифру (если вычеркнуть больше цифр, у числа будет меньше
разрядов, и оно заведомо не будет наибольшим). Если вычеркнуть первую двойку, во втором
по старшинству разряде будет тройка, иначе — двойка. Отсюда получаем ответ.
4. На 49 карточках записаны числа от 1 до 49. Карточки выложены в квадрат 77
числами вниз. Известно, что в каждом горизонтальном ряду числа упорядочены по возрастанию слева направо, а в вертикальном – сверху вниз. Можно ли последовательно открыть
не более 12 карточек, чтобы найти карточку с числом 25?
Ответ: Да. Решение. Сначала перевернём карточку, лежащую в левом нижнем углу.
Если на ней число большее 25, то последнюю строку можно не рассматривать – там нет
нужной карточки. Если меньшее, то можно не рассматривать первый столбец. Переходим к
левому нижнему углу оставшейся таблицы. Исключив 12 линий, мы оставим единственную
карточку, на которой должно быть число 25.
5. В кружке 6 человек и у каждого ровно 3 друга в этом кружке. Верно ли что всех
кружковцев можно рассадить за 3 парты, за каждой из которых сидят 2 друга?
Ответ: Да. Решение. Если в графе знакомств есть треугольник ABC, то каждый из трех
оставшихся дружит ровно с одним из ABC. Такие пары друзей и надо усадить за парты. Если
же треугольника нет, и A дружит с B, С и D, то последние трое между собой не дружат и,
стало быть, все дружат с оставшимися E и F. Тогда подойдут пары AB, CE и DF.
6. Петя и Вася по очереди умножают записанное на доске число на одно из трёх чисел: 2, 3 или 4. Сначала на доске написана единица. Выигрывает тот, кто первым получит
результат больше 250000. Кто может выиграть независимо от игры соперника?
Ответ. Вася. Решение. Стратегия Васи: если Петя умножает число на 2, то Вася на 4,
если Петя на 3, то Вася – на 3, если Петя на 4, то Вася на 2. Таким образом, после двух ходов
происходит умножение числа на 8 или на 9. После шести ходов результат оказывается не
меньше 86 = 262144 > 250000. Перед последним ходом Васи результат не больше
954 = 236196.
7. Как провести на круглом листе бумаги пять отрезков, каждый из которых соединяет две точки на краю листа, чтобы среди частей, на которые эти отрезки делят лист, оказались хотя бы один
пятиугольник и хотя бы один четырехугольник?
Решение. См. рисунок.
8. 100 депутатов Думы пользуются сетью Билайн, а 200 депутатов – сетью Мегафон. За внутрисетевой звонок Билайн берёт 43
копейки, а Мегафон меньше, но целое число копеек. За звонок в другую сеть стоимость
104
звонка возрастает в 3 раза. Все входящие звонки бесплатные. В течение дня каждый депутат звонит каждому по одному разу и от каждого один раз получает встречный звонок.
Сколько стоят звонки с Мегафона, если его ежедневные доходы с депутатов более чем на
десять тысяч рублей превышают доходы Билайна?
Ответ: 42 коп. за минуту. Решение. Билайн получает доход с внутрисетевых звонков:
4310099 = 4257 руб. Со звонков на Мегафон: 343100200 = 25800 руб. Всего 30057 руб. Если бы цена звонка Мегафона была 42 коп. за звонок, то его доход составил бы
42200199+342100200 = 16716+25200 = 41916, что соответствует условию задачи (разница
превышает 11 тысяч рублей). Если же Мегафон снизит цену хотя бы на копейку, то доход
составит 41200199+341100200 = 16318+24600 = 40918, что уже не соответствует условиям.
105
Download